Mathematics MODULE
Mathematics MODULE
ACKNOWLEDGEMENTS .................................................................................................................... IV
UNIT ONE: FURTHER ON SETS ........................................................................................................ 1
1.1. INTRODUCTION TO CONCEPTS OF SETS. ............................................................................. 1
1.2. THE NOTION OF SETS ................................................................................................................ 2
1.3. OPERATIONS ON SETS .............................................................................................................. 4
SOLVED PROBLEM FURTHER ON SET......................................................................................... 8
UNIT TWO: GEOMETRY AND MEASUREMENT ....................................................................... 11
2.1. REGULAR POLYGONS ............................................................................................................. 11
2.2. FURTHER ON CONGRUENCY AND SIMILARITY ............................................................... 13
2.3. FURTHER ON TRIGONOMETRY ............................................................................................ 17
2.4. CIRCLES ...................................................................................................................................... 20
2.5. MEASUREMENTS ...................................................................................................................... 23
SOLVED PROBLEMS ON GEOMETRY AND MEASUREMENTS .............................................. 25
UNIT THREE POLYNOMIAL FUNCTIONS ................................................................................ 27
3.1. INTRODUCTION TO POLYNOMIAL FUNCTIONS ............................................................... 27
3.2. THEOREMS ON POLYNOMIALS ............................................................................................ 31
3.3. ZEROS OF A POLYNOMIAL FUNCTION ............................................................................... 32
4.4. GRAPHS OF POLYNOMIAL FUNCTIONS.............................................................................. 35
UNIT FOUR: EXPONENTIAL AND LOGARITHMIC FUNCTIONS.......................................... 38
4.1. EXPONENTS AND LOGARITHMS .......................................................................................... 38
4.2. THE EXPONENTIAL FUNCTIONS AND THEIR GRAPHS. .................................................. 40
4.3. GRAPHS OF EXPONENTIAL FUNCTIONS. ........................................................................... 40
4.4. LOGARITHMS. ........................................................................................................................... 43
4.5. THE LOGARITHMIC FUNCTIONS AND THEIR GRAPHS. .................................................. 46
4.6. EQUATIONS INVOLVING EXPONENTS AND LOGARITHMS. .......................................... 50
4.7. APPLICATIONS OF EXPONENTIAL AND LOGARITHMIC FUNCTIONS. ........................ 51
UNIT FIVE: SOLVING OF INEQUALITIES ................................................................................... 54
5.1. INEQUALITIES INVOLVING ABSOLUTE VALUE ............................................................... 54
5.2. SYSTEMS OF LINER INEQUALITIES IN TWO VARIABLES .............................................. 57
5.3. QUADRATIC INEQUALITIES .................................................................................................. 59
UNIT SIX. PLANE GEOMETRY ....................................................................................................... 62
6.1. THEOREMS ON TRIANGLES ................................................................................................... 62
I
6.2. SPECIAL QUADRILATERALS ................................................................................................ 65
6.3. MORE ON CIRCLES................................................................................................................... 67
6.4. REGULAR POLYGONS(5) ........................................................................................................ 69
UNIT SEVEN MEASUREMENT ........................................................................................................ 70
7.1. SURFACE AREAS AND VOLUMES OF PRISMS AND CYLINDERS. ................................ 70
7.2. PYRAMIDS, CONES AND SPHERES ....................................................................................... 73
7.3. FRUSTUMS OF PYRAMIDS AND CONES .............................................................................. 77
UNIT EIGHT FURTHER ON RELATION AND FUNCTIONS ..................................................... 81
8.1. REVISION ON RELATIONS ...................................................................................................... 82
8.2. SOME ADDITIONAL TYPES OF FUNCTIONS ....................................................................... 87
8.3. CLASSIFICATION OF FUNCTIONS ...................................................................................... 102
8.4. COMPOSITION OF FUNCTIONS ............................................................................................ 106
8.5. INVERSE FUNCTIONS AND THEIR GRAPHS ..................................................................... 109
REVIEW EXERCISES ON UNIT 8 ................................................................................................ 114
PRACTICE QUESTIONS ON UNIT 8 ............................................................................................ 115
UNIT NINE: COORDINATE GEOMETRY ................................................................................... 120
9.1. STRAIGHT LINES .................................................................................................................... 121
9.2. CIRCLE ...................................................................................................................................... 134
REVIEW EXERCISES ON UNIT 9 ................................................................................................ 142
PRACTICE QUESTIONS ON UNIT 9 ............................................................................................ 143
UNIT TEN STATISTICS AND PROBABILITY............................................................................ 146
10.1. STATISTICS .......................................................................................................................... 147
10.2. PROBABILITY ...................................................................................................................... 182
REVIEW EXERCISES ON UNIT 10 .............................................................................................. 209
PRACTICE QUESTIONS ON UNIT 10 .......................................................................................... 210
UNIT ELEVEN: MATRICES AND DETERMINANTS ................................................................ 215
11.1. MATRICES ............................................................................................................................ 216
11.2. DETERMINANTS AND THEIR PROPERTIES .................................................................. 236
11.3. INVERSE OF A SQUARE MATRIX .................................................................................... 247
11.4. SYSTEMS OF EQUATIONS WITH TWO OR THREE VARIABLES ............................... 254
11.5. CRAMER‟S RULE ................................................................................................................ 266
REVIEW EXERCISES ON UNIT 11 .............................................................................................. 269
PRACTICE QUESTIONS ON 11 .................................................................................................... 271
II
UNIT TWELVE:VECTORS AND TRANSFORMATION OF THE PLANE .............................. 276
12.1. REVISION ON VECTORS AND SCALARS ....................................................................... 277
12.2. REPRESENTATION OF VECTORS .................................................................................... 278
12.3. COMPONENTS OF VECTORS ............................................................................................ 282
12.4. PRODUCTS OF VECTORS ................................................................................................. 288
12.5. APPLICATION OF VECTOR ............................................................................................... 293
12.6. TRANSFORMATION OF THE PLANE ............................................................................... 300
REVIEW EXERCISES ON UNIT 12 .............................................................................................. 325
PRACTICE QUESTIONS ON UNIT 12 .......................................................................................... 329
UNIT THIRTEEN:SEQUENCE AND SERIES ............................................................................... 333
13.1. SEQUENCE ............................................................................................................................ 334
13.2 ARITHMETIC AND GEOMETRIC SEQUENCES................................................................. 335
13.3 THE SIGMA NOTATION AND PARTIAL SUMS................................................................. 337
13.4. INFINITE SERIES ................................................................................................................. 339
SOLVED PROBLEMS ON SEQUENCE AND SERIES ................................................................ 340
UNIT FOURTEEN: INTRODUCTION TO LINEAR PROGRAMMING ................................... 344
14.1. REVISION ON LINEAR GRAPHS ...................................................................................... 345
14.2. GRAPHICAL SOLUTIONS OF SYSTEMS OF LINEAR INEQUALITIES ....................... 347
14.3. MAXIMUM AND MINIMUM VALUES ............................................................................. 351
14.4. REAL LIFE LINEAR PROGRAMMING PROBLEMS ....................................................... 357
REVIEW EXERCISES ON UNIT 14 .............................................................................................. 362
PRACTICE QUESTIONS ON UNIT 14 .......................................................................................... 363
UNIT FIFTEEN:MATHEMATICAL APPLICATIONS IN BUSINESS ...................................... 366
15.1. BASIC MATHEMATICAL CONCEPTS IN BUSINESS .................................................... 367
15.2. COMPOUND INTEREST AND DEPRECIATION .............................................................. 376
15.3. SAVING, INVESTING AND BORROWING MONEY ....................................................... 382
15.4. TAXATION ............................................................................................................................ 387
REVIEW EXERCISES ON UNIT 11 .............................................................................................. 394
UNIT SIXTEEN: INTRODUCTION TO CALCULUS .................................................................. 395
16.1. INTRODUCTION TO DERIVATIVES. ................................................................................. 396
16.2. APPLICATIONS OF DERIVATIVES .................................................................................... 398
16.3. INTRODUCTION TO INTEGRATION .................................................................................. 399
SOLVED PROBLEMS ON INTRODUCTION TO CALCULUS ................................................... 400
III
ACKNOWLEDGEMENTS
I would like to express my sincere gratitude to all those who contributed to the development of this
Grade 9-12 Mathematics module.
First and for most, I extend my heartfelt thanks to my mentor and content editor, Mitiku Daba (Ph.D),
for his invaluable guidance and support in chronological order arrangement of chapters, contents and
sub-contents throughout this process. Their passion for mathematics and dedication for mathematics
and dedication to teaching inspires me all the time.
I also equally wish to acknowledge Feyisa Kebede (Ph.D), curriculum editor for their collaboration
and feedback, which greatly enhanced the content and clarity of the material. Special thanks to Mr.
Ahmed Awol and Gezahagn Kasehey for their assistance in reviewing the module and providing
insightful suggestions.
Additionally, I am grateful to Adama Science and Technology University (ASTU) for their provision of
Laptop computer and environment necessary for writing and development of this module.
Finally, I would like to thank ASTU Special School directors Mr. Garedew Jima (M.A) and Mekonnen
Kebede (M.A) for their unwavering encouragement and support during my studies.
IV
UNIT ONE: FURTHER ON SETS
Sets are important concepts in mathematics. They are useful in defining and solving of mathematical
problems.
Sets and Elements
Sets is a collection of any object, things or numbers of well-defined.
The individual objects in set are called elements of set.
Repeating elements in a set does not add new elements to the set.
For example, the set {a, a, a} is the same as {a}.
A set is known by its elements.
A set is well-defined if and only if it can be decided that a given object is an element of the set.
Notation: we use capital letters to name sets and small letters to represent elements. The symbol „∈‟
stands for the phrase „is an element of‟ (or „belongs to‟). So, x∈A is read as „x is an element of A‟ or „x
belongs to A‟. We write the statement „x does not belong to A‟ as x∉A.
3.1. Description of sets
A set may be described by three methods:
I. Verbal method
This method may be used when there is a large number of elements or when all elements cannot be
named. We may describe a set in words.
Example:
a. The set of all whole numbers less than ten or
{all whole numbers less than ten}.
a. The set of all natural numbers. This can also be written as
{all natural numbers}
II. The listing method
If the elements of a set can be listed, then we can describe the set by listing its elements. The elements
can be listed completely or partially as illustrated in the
Complete listing method
In this method we list all elements of the set. We use this method if the set has small elements of the set.
Power set: Let A be any set. The power set of A, denoted by P(A), is the set of all subsets of A. That is,
P(A) = {S|S ⊆ A}
Proper Subset: Set A is said to be a proper subset of a set B, denoted by A ⊂B, if A is a subset of B and
B is not a subset of A.
That is, A ⊂ B means A ⊆ B but B ⊆ A
Note:1. For any set A, A is not a proper subset of itself.
2. If a set A is finite with n elements, then
I The number of subsets of A is and
II The number of proper subsets of A is
Universal sets
The universal set is the totality of elements under consideration as elements of any set and including all
possible elements of the set.
Equality of sets : Given two sets A and B, if every element of A is also an element of B and if every
element of B is also an element of A, then the sets A and B are said to be equal. We write this as A = B
∴ A = B, if and only if A ⊆ B and B ⊆ A.
Example: a. Let A = {1, 2, 3, 4} and B = {1, 4, 2, 3}.
A = B, since these sets contain exactly the same elements.
b. E = {x∈ℝ/(x – 2) (x – 3) = 0} and F = {x∈ℕ/1 < x < 4}.
By listing completely, the elements of each set, we have E = {2, 3} and F = {2, 3}. We see that E and F
have exactly the same elements. So, they are equal.
Note: If A and B are not equal, we write A ≠ B.
Example: Let C = {–1, 3, 1} and D = {–1, 0, 1, 2}.
C ≠ D, because 2 ∈ D, but 2 ∉ C.
Union of sets: The union of two sets A and B, denoted by A⋃B and read "A union B" is the set of all
elements that are members of set A or set B or both of the sets. That is, A⋃B = {x|x∈A or x∈B}
Example: a. A = {a, b, c, d, e} and B = {c, d, e, f, g}, then
A⋃B = {a, b, c, d, e, f g}
b. {a, b} ⋃ {c, d, e} = {a, b, c, d, e}
{1, 2, 3, 4, 5} ⋃ ∅ = {1, 2, 3, 4, 5}
Properties of the union of sets
For any sets A, B and C
1. Commutative property A⋃B = B⋃A
2. Associative property (A⋃B) ⋃C = A⋃(B⋃C)
3. Identity property A⋃∅ = A
Intersection of sets : The intersection of two sets A and B, denoted by A ⋂B and read as " A
intersection B ", is the set of all elements common to both set A and set B. common to both set A and set
B. That is, A⋂B = {x|x∈ A and x∈ B}.
Two or more sets are disjoint if they have no common element. A and B are disjoint, if and only if A ⋂ B = ∅.
Using the Venn diagram, In the Venn diagram,
is represented by the blue the sets A and B are disjoint shaded region:
Here A ⋂ B = ∅.
Example: a.
Properties of the intersection of sets : For any sets A, B and C and the universal set U
Commutative Property: A⋂B = B⋂A.
Associative Property: (A⋂B)⋂C = A⋂(B⋂C).
Identity Property: A⋂U = A
Solution a
The complement of a set
Let A be a subset of a universal set U. The complement (or absolute complement) of A, denoted by A', is
defined to be the set of all elements of U that are not in A. i.e., A' = {x|x∈U and x ∉A}.
Note that for any set A and universal set U, A′ = U\A
Example: In copies of the Venn diagram on the right shade
Equality of ordered pairs: Two ordered pairs (a, b) = (c, d), if and only if a = c and b = d.
Definition: Given two non-empty sets A and B, the set of all ordered pairs (a, b) where a∈A and b∈B is
called the Cartesian product of A and B, denoted by A × B (read "A cross B"). i.e., A × B = {(a, b) |a ∈A
and b∈B}.
Example 1 If A = { 1, 2, 3} and B = { 4, 5}, then
A × B = {(1, 4), (1, 5), (2, 4), (2, 5), (3, 4), (3, 5)}
Example 2 Let A = { a, b}, then form A × A.
Solution: A × A = {(a, a), (a, b), (b, a), (b, b)}.
Example 3 Let A = {–1, 0} and B = {–1, 0, 1}.
Find A × B and illustrate it by means of a diagram.
Solution: A × B = {(–1, –1), (-1, 0), (–1, 1), (0, –1), (0, 0), (0, 1)}
The diagram is as shown in the figure.
Note: n (A × B) = n (A) × n (B).
Definition : A polygon is a simple closed curve, formed by the union of three or more-line segments, no
two of which in succession are collinear. The line segments are called the sides of the polygon and the
end points of the sides are called the vertices
Types of polygons: Based on the measure of its interior angle of the polygon can be classified as
Convex polygon: a polygon whose all measure of its interior angle is less than
Concave polygon: a polygon at least one of the measures of its interior angle is greater than
Note:
Polygons can be classified according to the number of sides.
The number of vertices, angles and sides of a polygon are the same.
No of side No of interior angles Name of polygon
3 3 Triangle
4 4 Quadrilateral
5 5 Pentagon
6 6 Hexagon
7 7 Heptagon
8 8 Octagon
9 9 nonagon
10 10 Decagon
ANGLES OF POLYGONS
There are two types of angles of polygon: these are
i. Interior angle: is an angle of a polygon formed at the vertex on the inside of the polygon.
ii. Exterior angle: is an angle at a vertex of a polygon that is supplementary to the interior angle at
the vertex. It is formed between one side of the polygon and the extended adjacent side.
Example: In the polygon ABCD ∠DCB is an interior angle; ∠BCE and
∠DCF are exterior angles of the polygon at the vertex C
The sum of the measures of the interior angles of a polygon
Theorem 2:1 Angle sum theorem
The sum of the measures of the three interior angles of any triangle is
Note: The polygons are divided into triangles by drawing a diagonal from one vertex
Example: consider each of the convex polygons with all possible diagonal from one vertex.
No of sides No of triangles formed Sum of interior
of polygon from one vertex angles of polygon
3 1 1x1800 1800
4 2 2x1800 3600
5 3 3x1800 5400
( )
Since the vertex angles at O of each isosceles triangle have equal measures, Namely , it follows that
all the base angles of all the isosceles triangles are also equal.
From this, it follows that the measures of all the angles of the polygon are equal; the measure of an angle
of the polygon is twice the measure of any base angle of any one of the isosceles triangles. So, the
polygon has all of its sides equal and all of its angles equal. A polygon of this type is called a regular
polygon.
Definition 2. 3: A regular polygon is a convex polygon in which the lengths of all of its sides are equal
and the measures of all of its angles are equal.
Note that the measure of an interior angle of an n-sided regular polygon is , where
( – ) is the sum of the measures of all of its interior angles.
( )
Then the measure of each interior angle of a regular n-sided polygon is
A polygon is said to be inscribed in a circle if all of its vertices lie on the circle.
Example: Find the measure of an interior and exterior angle of a regular polygon with:
a. 10 sides b. 20 sides c. 12 sides
Solution: Then the measure of each interior angle of a polygon is given as
(n 2)1800 8(1800 )
a. The measure of an interior angle of a 10-sided regular polygon is 1440
n 10
(n 2)1800 18(1800 )
b. The measure of an interior angle of a 20-sided regular polygon is 1620
n 20
(n 2)1800 10(1800 )
c. The measure of an interior angle of a 10-sided regular polygon is 1500
n 12
And the measure of each exterior angle of a n side of regular polygon is given as
a. The measure of each exterior of 10 side is
Note
A regular n-side polygons has n-line of symmetry
A regular polygon of odd number of sides has every line of symmetry, passes through the vertex.
Circumscribed (inscribed) regular polygons
A polygon whose sides are tangent to a circle is said to circumscribe the circle.
An inscribed polygon is a polygon all of whose sides are a chords of a circle.
The circle is called circumscribed about the polygon if all sides polygons are chords of a circle.
Inscribe polygon is the same term as circumscribed circle.
EXAMPLE 2
2 2 5
180 0
Since 36 0 , where 5 is the number of their sides, we can generalize the above formulae for any n-
5
1800
sided regular polygon by replacing 360 by , as follows
n
Theorem 5.3
Formulae for the length of side s, apothem a, perimeter P and area A of a regular polygon
with n sides and radius r are
1. s rsin 3. 𝑎 = r cos
n n
2. P = nrsin 4. A= 𝑎P
n
Example 2
a. Find the radius of an equilateral triangle with perimeter 24 units.
b. Find the radius of a regular hexagon with perimeter 48 units.
c. Find the area of a regular hexagon whose radius is 5 cm.
d. Show that the length of each side of a regular hexagon is equal to the length of the radius of the
hexagon.
48 12r sin 30 0
3 8 3
4 r r units 48 6r r 8 units
2 3
C. To find the area of the regular hexagon, we use the formula
A= P , where is the apothem and P is the perimeter of the regular hexagon.
√
Therefore, A= P . / . nrsin / ( )( x x sin ) units.
n n
D. We know that the length of a side s of an n-sided regular polygon is given by
180 0
s 2r sin Where r is the radius of the polygon
n
180 0 1
. If n = 6 then, s 2r sin 2r sin 30 0 2rx r s r
6 2
Therefore, the length of a side s of a regular hexagon is equal to the radius r of the hexagon.
B C E F
The six congruent part of the triangle are the three corresponding angles and the three corresponding size
of and denoted by
These are
Example:1
1. Given that if ̅̅̅̅ ̅̅̅̅ , then prove ̅̅̅̅ ̅̅̅̅
B
Explanation: Two columns proof
Statements Reasons
D ̅̅̅̅ ≡ 𝐵𝐶
1. 𝐴𝐵 ̅̅̅̅
E
2.
3. ̅̅̅̅
𝐴𝐵 ≡ 𝐵𝐶̅̅̅̅
A C 4.
5.
6. Therefore 𝐴𝐶𝐷 ≡ 𝐶𝐸𝐹 by ASA
∴ ̅̅̅̅
𝐴𝐷 ≡ ̅̅̅̅
𝐶𝐸
B. SIMILAR FIGURES
Figures that have the same shape but that might have different sizes are called similar.
In similar figures:
i. One is an enlargement of the other.
ii. Angles in corresponding positions are congruent.
iii. Corresponding sides have the same ratio.
A D P S
2 2
B C 4 4
3
Q
6
The above hexagons are similar, which is denoted by provided that
i. A P, B Q, C R, D S , E T and F U
AB BC CD DE EF FA 2 1
ii.
PQ QR RS ST TU UP 4 2
NOTE:
i. The ratio of corresponding sides of two similar polygons is called the scale factor.
Therefore, the lengths of the sides of the larger polygon are 7.5, 10.5, 12, 16.5, and 28.5 units.
Construction of Similar Figures
Enlargement: An enlargement with center O and scale factor k (where k is a real number) is the
transformation that maps each point P to point P′ such that
Angle: An angle is the union of two rays with a common end point.
In general, we associate each angle with a real number called the measure of the
angle. The two measures that are most frequently used are degree and radian.
i. Measuring angles in degrees
Definition: A degree, denoted by (0), is defined as the measure of the central angle subtended by an arc
1
of a circle equal in length to of the circumference of the circle.
360
Note:
1
A minute which is denoted by ( ' ), is of a degree.
60
1
A second which is denoted by ( '' ), is of a minute, From these we have 1o=3600‟‟ relation.
60
ii. Measuring angles in radians A radian (rad) is defined as the measure of the
central angle subtended by an arc of a circle equal in length to the radius of the
circle.You know that the circumference of a circle is equal to 2πr.
Since an arc of length r along the circle gives 1 rad, a complete rotation of length 2πr generates an angle
of 2π radians. On the other hand, we know that a complete revolution represents an angle of 3600. This
gives us the following relationship:
1 revolution = 3600 = 2π radians
Using similarity ratio of sides of triangles it is possible to drive the trigonometrical ratio on the triangles.
From the similarity ratio
The fractions or ratios in each of these proportions are called trigonometric ratio
The angles 00, 300, 450, 600 and 900 are called special angles, because they have these exact
trigonometric ratios, its values are given as follows.
How can you use the trigonometric table to find the sine, cosine and tangent of obtuse angles?
To find trigonometric value of an obtuse angle we see as follows. y
Let Be an obtuse angle and
Be supplementary angle of
Then and are equal in magnitude different direction
Because OP‟ is obtained by reflecting OP along y-axis. x
1800 1800 , becuase , sup lemantary angle B 0 C
Then
180 0 1800 becuase
19
2. To find cosine of as expressed on the coordinate P(a, b) , to relate with acute angle is shown as:
a a
cos And cos Is an acute angle in the 1st quadrant, which is
a b 2 2
a b 2 2
3. Similarly, to find tangent of whose coordinate is P(a, b) , relating with acute angle is shown as:
a a a
tan And tan , which implies tan tan tan tan(1800 )
b b b
Generally: For an obtuse angle we have
2.4. CIRCLES
Symmetrical Properties of Circles
Circle is defined as the set of points in a given plane, each of which is at the same distance from a fixed
point of the plane. The fixed point is called the center and the distance is called radius of the circle
r
O
Note
A line segment through the center of a circle with end points on the circle is called a diameter
A chord of a circle is a line segment whose end points lie on the circle.
A line that divided the figure and one part of the figure is coinciding with the other part divided by the
line, then the line is called symmetric line and the figure is called symmetric figure.
20
21
22
Solution: A minor segment is segment lie between chord ̅̅̅̅ and arc ̂ . A O C
Solution: first we find the measure of central angle m (∠AOB) =2m (∠AKB) =2(300) =600 A B
Then area of the major segment is=area of circle-area of minor arc
r 2 1 2 8 40
A r 2 r sin 16 4 3 4 3 cm 2
3 3
0
360 2
2.5. MEASUREMENTS
Case ii. The area A of any triangle with base b and the corresponding height h is given by A 1 bh
2
Case iii. The area A of any triangle with sides a and b units long and angle C (∠C) included between
1
these sides is A ab sin( C )
2
NOTE: Heron‟s formula
The area A of a triangle with sides a , b and c units long and semi- perimeter
s
1
a b c is given by A s(s a)(s b)(s c)
2
23
Then from this the length of the sides are 12, 8 and 6 inches respectively.
ii. To find it‟s area we use heron‟s formula
24
25
immersed in it completely. Calculate the height by which the water rises in the cylinder.
Solution:
1. . Perimeter of the isosceles triangle = 5 + 5 + 4 = 14 inches and its semi-perimeter is=14/2=7
AT = 2AB + AL=14x6+2( 2 21 )= 84 4 21 sq inches
Where:
Given:
Height h=1 m=100 cm (convert height to centimeters for consistency in units).
Radius r=70 cm
Step 1: Volume Calculation
26
27
28
Function
Degree 3 3 2 3
B. ( )( ) ( ) ( – )
–
=
29
Telegram channel https://t.me/johnson201485
( )( ) ( ) ( – )
=
Function
Degree 4 4 4 3
Note that if the degree of f is not equal to the degree of g, then the degree of ( ) ( ) and ( – ) ( ) is
the degree of ( ) or the degree of ( ), whichever has the highest degree. If they have the same degree,
however, the degree of ( ) ( ) and ( – ) ( ) might be lower than this common degree when they
have the same leading coefficient.
b. Multiplication Of Polynomials: The degree of the product of two non-zero polynomials is the sum of
the degree of the two polynomials in the product. That is let the degree of ( ) is and the degree of
( ) is , then the degree of ( )( ) is .
Note:The product of two polynomials and is found by multiplying each term of ( ) every term of ( )
and finally add their results.
The product of two polynomial functions and is written as , and is defined as:
( )( ) ( ) ( ), for all ∈ .
Example7: For each of the following find ( ) ( ), degree, leading coefficient , leading term ,
constant term and of , and
A. ( ) ; ( )
B. ( ) ( ) – –
Answer:
A. ( ) ( ) ( )( ) ( ) ( ) ( )
=( ) ( ) ( )
=
Function Degree leading leading constant
coefficient term term
2 1 5 -3
1 5 3 3
2+1=3 1*5=5 5*3=15 -12
B. ( ) ( ) ( )( – – ) ( – – ) ( – – )
=( ) ( )
Function Degree leading leading constant
coefficient term term
2 1 0 2
3 2 0 0
2+3=5 1*2=2 0 4
30
Telegram channel https://t.me/johnson201485
Division of Polynomials.: The quotient (division) of two polynomial functions f and g is written as ,
and is defined as: ( )( ) ( ) ( ) Provided that ( ) , for all ∈ .
Example: For each of the following, find the quotient and the remainder:
A. (x – ) (x – ) B. ( x –x x ) (x )
Answer a.
Quotient Remainder
A x x 0
B x x 0
From the above example the remainder is zero. Which means that the divisor is divide the dividend into
quotient.
31
Telegram channel https://t.me/johnson201485
Remainder Theorem
Let ( ) be a polynomial of degree greater than or equal to 1 and let be any real number. If ( ) is
divided by the linear polynomial ( – ), then the remainder is ( ).
Example: In each of the following, express the function in the form ( ) ( – ) ( ) ( )
for the given number c, and show that ( ) is the remainder.
a. (x) x –x x
b. (x) –x x x –
c. When ( ) – – is divided by x + 1, the remainder is 15. What is the value
of ?
Answer:
a. (x) (x – ) (x x ) With remainder 29 ( ) ( ) – ( )
– Therefore, f (2) = 29 = Remainder.
b. ( ) ( )( – – ) (– ) With remainder –1. (– )
– (– ) (– ) (– ) – – – Therefore, f (–1) = –1 = Remainder.
c. – How? Show it
Remark If ( ) is divided by ( )( ) then the remainder will be linear in , which is of the
form . Thus ( ) ( )( ) ( ) . That is the remainder will be linear equation
because ( ) divided by degree 2
B. Factor Theorem: Let ( ) be a polynomial of degree greater than or equal to 1 and let be any real
number, then
i. – is a factor of ( ), if ( ) , and
ii. ( ) , if – is a factor of ( ).
Example: In each of the following, use the factor theorem to determine whether or not g (x) is a factor of f (x).
a. ( ) ; and ( )
b. ( ) ; and ( )
c. Find numbers and so that – is a factor of ( ) – – and (– ) .
Answer:
a. . / ( ) , then (x) x is not a factor of (x) x x
b. ( ) ( ) ( ) ( ) ; then ( ) ; is not a factor of
( )
c. And – ; how? Show it
3.3. ZEROS OF A POLYNOMIAL FUNCTION
At the end of this sub-unit, students will be able to:
Determine the zero(s) of a given polynomial function.
State the Location Theorem.
Apply the location theorem to approximate the zero(s) of a given polynomial function.
Apply the rational root test to determine the zero(s) of a given polynomial function.
32
Telegram channel https://t.me/johnson201485
Definition: For a polynomial function and a real number , if ( ) , then is a zero of .
Note that if – is a factor of ( ), then c is a zero of ( ).
Example:
a. (y) y – y y c. (x) x
b. (x) x – x – d. Determine the zeros of ( ) – .
e. Find a polynomial function f of degree 3 such that ( ) and the zeros of f are 0, 5 and 8.
Answer:
a. To find the zeros of the function we use different ways. The most important way is factorization
method. Then (y) y – y y y(y – y ) y or
y – y
y – y (y )(y ) y Or y
Then the zeros of (y) y – y y is -1, 0 and 1
b. (x) x – x – ( ) x – ( )
√
( ) ( ) ( ) ( )( )
c. Exercise
d. ( ) – ( ) –
( ) ( )
√
( ) ( ) ( )( )
√ √
Then the zeros of ( ) – are and
Note: A polynomial function cannot have more zeros than its degree.
e. Let ( ) ( – )( – ). Then, ( ) ( – )( – )
, then ( ) ( – )( – )
Zeros and Their Multiplicities
If ( ) is a polynomial function of degree n, n ≥ 1, then a root of the equation ( ) is called a zero
of .
By the factor theorem, each zero c of a polynomial function ( )generates a first-degree factor
( – ) of ( ). When ( )is factorized completely, the same factor ( – ) may occur more than once,
in which case c is called a repeated or a multiple zero of ( ). If ( – ) occurs only once, then c is
called a simple zero of ( ).
Definition If ( – ) is a factor of ( ) but ( – ) is not, then c is said to be a zero of multiplicity
k of .
Example: In each of the following, the indicated number is a zero of the polynomial function f (x).
Determine the multiplicity of this zero.
a. (x) x x – x b. – (x) x x x x
33
Telegram channel https://t.me/johnson201485
Answer:
a. (x) x x – x ( ) ( ). Therefore, 1 is the zero of multiplicity 2.
b. (x) x x x x ( ) . Therefore –1 is the zero of multiplicity 3.
Location Theorem : A polynomial function with rational coefficients may have no rational zeros. Most
of the standard methods for finding the irrational zeros of a polynomial function involve a technique of
successive approximation. One of the methods is based on the idea of change of sign of a function.
Consequently, the following theorem is given.
Theorem: Location Theorem: Let and be real numbers such that . If f is a polynomial
function such that ( ) and ( ) have opposite signs, then there is at least one zero of between
and .
Note: - This theorem helps us to locate the real zeros of a polynomial function. It is sometimes possible
to estimate the zeros of a polynomial function from a table of values.
Example:
a. Use the location theorem to verify that ( ) has a zero between and :
( ) – –
b. Use the Location Theorem to locate each real zero of ( ) between successive integers:
( ) – – –
Answer:
a. ( ) ( ) ( ) ( ) ( ) , and
( ) ( ) ( ) ( ) ( )
Therefore, using location theorem ( ) has zeros between -3 and -2
b.
Function -3 -2 -1 0 1 2 3
( ) – – 68 9 -2 -1 0 13 74
Then ( ) has zeros between -2 and -1
Rational Root Test
The rational root test relates the possible rational zeros of a polynomial with integer coefficients to the
leading coefficient and to the constant term of the polynomial.
Theorem: If the rational number , in its lowest terms, is a zero of the polynomials (x)
n
nx n xn x with integer coefficients, then must be a factor of and must
be a factor of
Example: Find all possible rational zeros:
a. ( ) – – b. ( ) –
1. Find all the rational zeros of the polynomial, and express the polynomial in factorized form:
a. ( )
1a. -1 and 3/2 are rational zeros of p. how? Show it
1b. 0, -1, -5/3 and 3/2 are rational zeros of p. how? Show it
2a. –1 and 2 are the only two rational zeros of p. how? Show it
34
Telegram channel https://t.me/johnson201485
4.4. GRAPHS OF POLYNOMIAL FUNCTIONS
Solution:
a. The graph is not a polynomial function, because there is a gap.
b. The graph is not a polynomial function, because there is a hole.
c. The graph is not a polynomial function, because there have sharp corner.
d. The graph is a polynomial function, because it is smooth and continuous.
Note: To draw the graphs of polynomial function we follow the following steps
A. X-intercept
An x-intercept is a point where a curve crosses the x-axis. The curve crosses or touch the x-axis at any
point where or ( ) . It is called the zeros of the function.
B. Y-intercept: A y-intercept is a point where a curve crosses the y-axis. The curve crosses the y-axis at
any point where .
Example: sketch the graph of the following polynomial function
a. ( ) b. ( )
Answer: To draw the graphs of each of the polynomial function, first we find the zeros polynomial or x-
intercept and y-intercept. Y
a. ( ) 2
The zeros of ( ) 1
( )
-2 -1 1 2 x
( ) ( ) -1
( )( )
-2
35
Telegram channel https://t.me/johnson201485
( )( )( )
Then the zeros of ( ) are -2, -1 and 1. This means
X-intercepts are (-2, 0), (-1, 0) and (1, 0) and The y-intercept is the value of y at x=0, if x=0
( ) , Then y-intercept is (0,-2)
Therefore the graph of ( ) is given as follows
b. Similarly to draw the graph of ( ) , We use the same steps Y
So the zeros of ( ) or x-intercept is (-2, 0), (-1, 0), (1, 0) and (2, 0) 4
And y-intercept is ( ) at then (0, 4) is y-intercept.
2
Then the graphs of the function ( ) is given as
Follows in the right side. -2 -1 1 2
BEHAVIOUR OF THE GRAPH OF POLYNOMIAL FUNCTION: -2
Note: The graphs of polynomial function: -4
With n-degree meets (crosses) the x-axis at most n-times
Crosses the y-axis exactly once.
Has at most n-1 turning point.
The graphs of even-degree polynomial function may not cross the x-axis.
Example:
a. ( ) , ( ) , …does not cross x-axis at all.
The graphs are completely lies above x-axis.
b. ( ) , ( ) … does not cross x-axis at all.
The graphs are completely lies below x-axis.
Example: you can observe from the graphs of above example and .
1. Local maximum and local minimum
The points at which the graph changes direction are called turning point. If the turning point is peak,
then it is called local maximum and if the turning point is valley, then it is called local minimum.
A point of that is either maximum point or minimum point is called local extremum of the
function.
2. FAR LEFT AND FAR RIGHT BEHAVIOUR OF POLYNOMIAL FUNCTION.
The graph of polynomial function may have several up and down fluctuations. How every the graph of
every polynomial function will eventually increase or decrease without bound as the graph moves far to
the left or far to the right from the origin.
So the leading term is said to be dominate a polynomial function ( )
as | | becomes large, because the absolute value of will be much larger than the
absolute value of any other terms.
36
Telegram channel https://t.me/johnson201485
In General: The behavior of the graph of polynomial function as x-decrease without bound to the left or
as x-increase without bound to the right can be determined
By degree (even or odd) and By leading coefficient (positive or negative). So in a polynomial
( ) the dominant term is . The graph of ( ) eventually
rises or falls in the following four cases
Case 1: A positive leading coefficient and even degree ( and even ) 1
The graph is rise to the left and rise to the right without bound
Example: ( ) , ( ) , ( ) , and etc.
Case 2: A negative leading coefficient and even degree ( and even )
The graph is falls to the left and falls to the right without bound
Example: ( ) , ( ) , ( ) , and etc. 3
Case 3: A positive leading coefficient and odd degree ( and odd )
The graph is falls to the left and rise to the right without bound
Example: Example: ( ) , ( ) , ( ) , and etc. 4
Case 4: A negative leading coefficient and even odd ( and odd )
The graph is rise to the left and falls to the right without bound
Example: ( ) , ( ) , ( ) , and etc.
MULTIPLECITY AND X-INTERCEPT.
Let in a polynomial function ( ) , If ( ) is a factor of ( ),
then y
A. If the exponent k is even, then Graph of
i. The graph of ( ), touches the x-axis at x=c. case A
c x
ii. The sign of ( ), does not change from
one side to the other side of x= c.
iii. The graph has a local extremum at x=c. y
Graph of
B. If the exponent k is odd, then
case B
i. The graph of ( ), crosses the x-axis at x=c.
c c x
ii. The sign of ( ), does change from
one side to the other side of x= c.
iii. The graph has no a local extremum at x=c.
Exercise: sketch and state the properties of multiplicity and x-intercept of the following function.
a. ( )
b. ( )
37
Telegram channel https://t.me/johnson201485
UNIT FOUR: EXPONENTIAL AND LOGARITHMIC FUNCTIONS
Unit Outcomes: After completing this unit, students will be able to:
Understand that the laws of exponents are valid for real exponents.
Know specific facts about logarithms.
Know basic concepts about exponential and logarithmic functions.
Solve mathematical problems involving exponents and logarithms.
Main Contents
4.1.Exponents and logarithms.
4.2. The exponential functions and their graphs.
4.3. The logarithmic functions and their graphs.
4.4. Equations involving exponents and logarithms.
4.5. Applications of exponential and logarithmic functions.
A. An Exponent:
At the end of this subunit, students will be able to;
Explain what is meant by exponential expression.
State and apply the properties of exponents (where the exponents are real numbers).
There are many mathematical situations in which we multiply a number by itself repeatedly. Writing
such expressions in exponential form provide a short hand method for representing this repeated
multiplication of the same factor.
Example: A. Exponent
x x x x x x x x x
Base
10 is indicate the number of factor in the product, 2 appears as a factor of 10 times
Exponent
n
x x x x Base
B.
Exponent
n
x x x x Base
2 appears as a factor of n times
a appears as a factor of n times
Note: If n is a positive integer, and then is a product of n factor of .
Such that . In is called base, the exponent and is the power of .
Laws of Exponents
If and are a positive real number, and and are a rational number, then the following properties
hold true.
a. d. ( ) f. h. ⁄ √
b. ⁄
e. . / g. For i. (√ )
c. ( )
38
Telegram channel https://t.me/johnson201485
j. If and are non-zero real numbers then it is always true that for . / . /
k. Any non-zero numbers raised to zero is one. That is , if
Property of equality for exponential equations
l. For b > 0, b ≠1, x and y real numbers,
, if and only if x = y
, (x ≠ 0), if and only if a = b
Rational and Irrational exponents
Note:
A. If and are a positive real number, and and are integers with ,
⁄
(√ ) √ ( )
In √ is not real number if is a negative and is even natural number.
√ is a real number if is a negative and is odd natural number.
B. If is an irrational number and , then is a real number between and for all
possible choices of a rational numbers and such that .
Note:
A. For we have and a real number x:
i. for x ii. for x
B. For we have and a real number x:
iii. for x iv. for x
Example:
i. Use the laws of exponents to simplify each the following exponential expressions:
( )
a. b. c. d.
( )
Solution:
( )
a. c.
( )
b. ( ) ( )
( )
( ) ( ) ( )
d.
ii. Simplify each of the following expressions using one or more of the laws of exponents:
a. ( ) c.
( ) ( )
b. 4 5
Solution:
a. ( ) . / . /
b. 4 5 . / . /
√
( ) ( )
c.
39
Telegram channel https://t.me/johnson201485
4.2. THE EXPONENTIAL FUNCTIONS AND THEIR GRAPHS.
Competencies
At the end of this subunit, students will be able to:
Define an exponential function.
Draw the graph of a given exponential function.
Describe the graphical relationships of exponential functions having bases reciprocal to each other.
Describe the properties of an exponential function by using its graph.
The function defined b
(x) ( ) is called an exponential function with base b and exponent x.
The domain of f is the set of all real numbers.
Note that in the definition of an exponential function, the base is excluded because it yields
( ) Constant function
This is a constant function, not an exponential function.
Example: a The exponential function with base 2 is the function (x) with domain (– )
The values of ( ) for selected values of x follow:
X -3 -2 -3/2 -1 0 1 3/2 2 3
(x)= 1/8 ¼ 1/(2√ ) 1/2 1 2 2√ 4 8
Example The exponential function with base ½ is the function (x) . / with domain (– )
The values of ( ) for selected values of x follow:
X -3 -2 -3/2 -1 0 1 3/2 2 3
(x)=. / 8 4 (2√ ) 2 1 1/2 1/2√ 1/4 1/8
Generally: The function defined by (x) ( ) is exponential functions which have the
following general property.
1. The domain is (– ) 4. It is continuous everywhere.
2. The range is ( ) 5. If b > 1 it is increasing on(– )
3. It passes through ( ) 6. If b < 1 it is decreasing on(– )
Example: sketch the graphs of each of the following exponential function and state its properties:
a. ( ) d. ( ) . / and ( ) . / on the same
b. ( ) . / coordinate.
c. ( ) And ( ) on the same
coordinate.
40
Telegram channel https://t.me/johnson201485
Solution:
a.First, recall that the domain of this function is the set of real numbers. Next, putting x = 0 gives y = 20 =
1, which is the y-intercept and (There is no x-intercept, since there is no value of x for which y = 0) The
values of ( ) for selected values of x is given as follows:
X -3 -2 -3/2 -1 0 1 3/2 2 3
Y
(x)= 1/8 1/4 1/(2√ ) 1/2 1 2 2√ 4 8
4 𝑥
Note that approaches zero as x decreases without bound. 𝑓(𝑥)
There is a horizontal asymptote at y = 0. 2
Furthermore, increases without bound when x increases without bound. 1
Thus, the range of f is the interval ( ) Finally, the graph is given as follows:-2 -1 2 x
b. First, recall again that the domain of this function is the set of real numbers. 1
0
Next, putting x = 0 gives y = (1/2) = 1, which is the y-intercept. Y
(There is no x-intercept, since there is no value of x for which y = 0) 4
The values of ( ) for selected values of x is given as follows: 𝑥
𝑓(𝑥) ( )
2
X -3 -2 -3/2 -1 0 1 3/2 2 3
1
(x)=. / 8 4 (2√ ) 2 1 ½ 1/2√ 1/4 1/8 -2 -1 1 2 X
Note that (1/2)x increases without bound when x decreases without bound.
Furthermore, (1/2)x approaches zero as x increases without bound:
There is a horizontal asymptote at y = 0. Y
As before, the range of f is the interval (0, ). 4 𝑓(𝑥) 𝑥
-2 -1 1 2 x
41
Telegram channel https://t.me/johnson201485
(x)=. / 27 9 √ 3 1 1/3 1/( √ ) 1/9 1/27
The values of the function are greater than 1 for x < 0 and between 0 and 1 for x > 0.
The graph approaches the x-axis as an asymptote on the right and
Increases without bound on the left.
Properties of Exponential Functions
a. The exponential function y = bx (b > 0, b ≠ 1) has the following properties:
1. Its domain is (– , ).
2. Its range is (0, ).
3. Its graph passes through the point (0, 1)
4. It is continuous on (– , ).
5. It is increasing on (– , ) if b > 1 and decreasing on (– , ) if b < 1.
Exponential function with Base “e”
Exponential functions to the base e, where e is an irrational
Then its graph is shown
Number, whose value is 2.7182818…, plays an important role
In both theoretical and applied problems.
For any real number x, the function, f(x) = ex,
Defines the exponential function with base e,
Usually called the natural exponential function.
It can be shown that i . /
Examples: sketch the graphs of the exponential function f(x) = ex and f(x) = e-x
Solution: Since ex > 0 it follows that the graph of y = ex is similar to the graph of y = 2x and Since e–x >
0 it follows that 0 < 1/e < 1 and so f(x) = e–x = 1/ex = (1/e)x is an exponential function with base less than
1. Therefore, it has a graph similar to that of y = (1/2)x.
Consider a few values for x:
X -3 -2 -1 0 1 2 3
( ) 0.05 0.14 0.37 1 2.72 7.39 20.09
( ) 20.09 7.39 2.72 1 0.37 0.14 0.05
y
y 𝑔(𝑥) 𝑒 𝑥
5
5
3
3
𝑓(𝑥) 𝑒𝑥 1
1
-3 -2 -1 1 2 3 x
-3 -2 -1 1 2 3 x
42
Telegram channel https://t.me/johnson201485
Sketch the graph of the exponential function f(x) = ex.
and g(x) = e–x, on the same coordinate.
From above example its graphs is given as follows
( ) ( ) y
The domain is R. The domain is R. 𝑥 𝑓(𝑥) 𝑒𝑥
𝑔(𝑥) 𝑒 5
The range is (0, ∞) The range is (0, ∞) 3
Is an increasing function. Is a decreasing function.
1
The graph is intersects the The graph is intersects the y-
y-axis at (0, 1). axis at (0, 1). -3 -2 -1 1 2 3 x
Greater than 1, if x > 0 Greater than 1, if x < 0
0 < f(x) < 1, if x < 0 0 < g(x) < 1, if x > 0
Generally:
All members of the family f(x) = bx (b > 0, b ≠ 1) have graphs
pass through the point ( 0 , 1)
are above the x-axis for all values of x
are asymptotic to the x-axis
Have domain the set of all real numbers.
Have range the set of all positive real numbers.
4.4. LOGARITHMS.
43
Telegram channel https://t.me/johnson201485
Exponential form Logarithmic form
( )
Generally: the value of is the answer to the question “to what power must b be raised to produces
?
Example: simplify each of the following.
a. b. √ c.
√
Answer:
a. Let then implies .
b. Let √
√ then ( √ ) √ implies .
c. Let then ( ) implies
Laws of logarithms (theorems on logarithm):
To simplify logarithmic expressions and solve logarithmic equations the following theorems are very
important.
Let which is positive numbers and ∈ , then
a. product rule for d. base change of logarithm
logarithm
e. if and only
b. quotient rule for
f.
logarithm g.
c. for any real number k h.
Example: If , then find the value of the following logarithm.
a. c. . / e.
b. f.
d.
g.
Solution:
a.
b. ( )
c. . / ( )
d.
Logarithms in base 10 (common logarithms)
The logarithm of a positive number to base 10 is called a common logarithm. The common logarithm is
usually the most convenient one to use for computations involving scientific notations because we use the
44
Telegram channel https://t.me/johnson201485
base 10 number system and also it has a special notation, which means a common logarithm is usually
written without indicating its base. For example, is simply denoted by .
So if a logarithm is given with no base, we take it to be base 10.
Example1: Find the common logarithm of the following
a. √ b. c. d.
Solution:
a. d.
b. √
c.
Note: If we write a number , as where , then logarithm of x can be read as
from a common logarithm table. The logarithm of m is called the mantissa of the logarithm of x, and c is
the characteristics of logarithm.
Using the logarithm table
The logarithm of any two decimal place number between 1.00 and 9.99 can be read directly from the
common logarithm table (a part of the table is given below for your reference).
So
Read the number at the intersection of row 5.2 and column 6
Reading the number in row 5.2 under column 6, we get 0.7210
45
Telegram channel https://t.me/johnson201485
solution:
a. antilog 0.8998 = 7.94 c. antilog –6 = 0.000001
b. antilog 5.721 = 526000 d. antilog–0.2 = 0.6310
e. The number 0.8175 does not appear in the table. The closest value is 0.8176 and 0.8176 = log 6.57.
∴antilog 0.8175 can be approximated by 6.57 .
f. antilog 3.9058 = antilog (0. 9058 + 3) = 8.05 × 103 = 8050.
In general: Antilog (log c) = c for c>0 and log(antilog(c)=c for any real number c.
Computation with logarithms:
In this section you will see how logarithms are used for computations.
For instance, to find the product of 32 and 128 using logarithm to the base 2, you can do it as follows:
Lex
nti ( x nti ( )
√ √
b. Let ( )
√
x ( )
( ) ( )
x nti ( x) nti ( ) nti ( )
Competencies
At the end of this subunit, students will be able to:
Define logarithmic function.
Draw the graph of a given logarithmic function.
Describe the properties of a logarithmic function by using its graph.
Describe the graphical relationship of logarithmic functions having bases reciprocal to each other.
Describe how the domains and ranges of exponential and logarithmic functions are related.
Describe the relationships of the graphs of exponential and logarithmic functions.
46
Telegram channel https://t.me/johnson201485
Logarithmic function
For x 0 and 0 a 1, y = loga x if and only if x = a y.
The function given by f (x) = loga x is called the logarithmic function with base a.
Every logarithmic equation has an equivalent exponential form: y = loga x is equivalent to x = a y
A logarithmic function is the inverse function of an exponential function.
Generally:
The function defined by ( ) is called the logarithmic function with base b.
The domain of f is the set of all positive numbers.
The range of f is the set of all real numbers.
Example1: sketch the graph of logarithmic function ( )
Solution: Since the logarithm function is the inverse of the exponential function of the same base, its
graph is the reflection of the exponential function in the line y = x.
OR by preparing table value for ( ) it is possible to draw. Y
( ) ( ) 4 𝑥
𝑓(𝑥)
-2 ¼ ¼ -2 2
𝑓(𝑥) 𝑥
-1 ½ ½ -1 1
-2 -1 -1 1 2 x
0 1 1 0
-2
1 2 2 1 -4
2 4 4 2
3 8 8 3
47
Telegram channel https://t.me/johnson201485
Example 3: sketch the graphs of a logarithmic function of y = log2 x and y= log10 x
From these graphs as the base becomes large, Y
It is approaches to y-axis to the left side and 4 𝑓(𝑥) 𝑥
𝑓(𝑥) 𝑥
Approaches to x-axis to the right side. 2
1
-10 -1 -1 1 10 x
-2
-4
Solution: The graphs of this exponential and logarithmic can be drawn by preparing its table value
( )
. /
1/27 -3 -3 1/27 27 -3 -3 27
1/9 -2 -2 1/9 9 -2 -2 9
1/3 -1 -1 1/3 3 -1 -1 3
1 0 0 1 1 0 0 1
3 1 1 3 1/3 1 1 1/3
9 2 2 9 1/9 2 2 1/9
27 3 3 27 1/27 3 3 1/27
48
Telegram channel https://t.me/johnson201485
Generally
Domain of y = logbx is equal to range of y=bx and
Range of y = logbx is equal to Domain of y=bx
Properties of Logarithmic Functions
The logarithmic function
y = logbx (b > 0, b ≠ 1)
has the following properties:
1. Its domain is (0, ).
2. Its range is (– , ).
3. Its graph passes through the point (1, 0).
4. It is continuous on (0, ).
5. It is increasing on (0, ) if b > 1and decreasing on (0, ) if b < 1.
6. The values of the function are negative for 0 < x < 1 and they are positive for x > 1 for b>1 and The
values of the function are positive when 0 < x < 1 and they are negative when x > 1 for 0<b<1
Natural logarithm:
The function defined by f(x) = loge x = ln x (x 0, e 2.718281) is called the natural logarithm
function.
Note: y = ln x is equivalent to e y = x
Examples
x2 x2 1 1
Expand and simplify the expression: ln solution 2 ln x ln( x 2 1) x
ex 2
Example
Sketch the graph of the function y = ln x.
Solution
We first sketch the graph of y = ex.
The required graph is the mirror image of the graph of
y = ex with respect to the line y = x:
49
Telegram channel https://t.me/johnson201485
4.6. EQUATIONS INVOLVING EXPONENTS AND LOGARITHMS.
50
Telegram channel https://t.me/johnson201485
EXERCISE: Show each steps for each of the following questions.
51
Telegram channel https://t.me/johnson201485
We can use logarithms to determine the time it takes for the principal to increase to a given amount.
Finding Term for an Investment to Double
A sum of birr 5000 is invested at an interest rate of 5% per year. Find the time required for the money
to double if the interest is compounded according to the following method.
(a) Semiannual (b) Continuous
A). Semi-annualy : We use the formula for compound interest with P = birr 5000, A(t) = birr 10,000, r =
0.05, n = 2
and solve the resulting exponential equation for t.
2t
0.05
5000 1 10000
2
1.025
2t
2
log1.025 2t log 2
2t log1.025 log 2 (Law 3)
log 2
t 14.04
2 log1.025
The money will double in 14.04 years.
b). Continously : We use the formula for continuously compounded interest with P = birr 5000, A (t) =
birr 10,000, r = 0.05 and solve the resulting exponential equation for t.
52
Telegram channel https://t.me/johnson201485
Solution :We use the formula for continuously compounded interest with P = birr 1000, A (t) = birr
4000, r = 0.04 and solve the resulting exponential equation for t.
b.
53
Telegram channel https://t.me/johnson201485
UNIT FIVE: SOLVING OF INEQUALITIES
Unit Outcomes:
After completing this unit, students will be able to:
Know and apply methods and procedures in solving problems on inequalities involving absolute value.
Know and apply methods in solving systems of linear inequalities.
Apply different techniques of solving quadratic inequalities.
Main content
5.1.Inequalities involving absolute value
5.2.Systems of liner inequalities in two variables
5.3.Quadratic inequalities
Competencies
At the end of this subunit, students will be able to:
Describe sets using interval notation.
Solve inequalities involving absolute value of linear expression.
Introduction
Set: Is a collection of well-defined objects is called set.
Set can be described by:
Verbal method
Listing method (partial listing or complete listing method)
Set builder method
Notation: For real numbers a and b where a < b,
(a, b) is an open interval;
(a, b] and [a, b) are half closed or half open intervals; and
[a, b] is a closed interval, which is it include the boundary elements.
Note: If a and b are fixed real numbers with a < b, then
[a, b] = {x: a ≤ x ≤ b and x∈ℝ} and (a, b) = {x: a < x < b, x∈ℝ} ( ] { } ,
(-∞, a]={x: x ≤ a and x∈ℝ } and (a, ∞)={x: x > a and x∈ℝ }
The symbol "∞" is used to mean positive infinity and "−∞" is used to mean negative infinity.
54
Telegram channel https://t.me/johnson201485
Example: simplify each of the following using intervals
a. { x : x∈ℝ and x ≠ –2 } b. { x: 2x + 3 ≥ –5x} c. {x: 2x –1 < x < 3}
Solution:
a. ( )∪( )
b. x – x x x , )
c. {x: 2x –1 < x < 3} which means, 2x –1 < x, x < 3 and 2x –1 < 3 from this three equation we have x<1
Then the interval form of {x: 2x –1 < x < 3} is ( )
Absolute value:The number that shows only the distance from the point corresponding to zero (and not
the direction) is called the absolute value.
Definition: If x is a real number the absolute value of x, denoted by | | , is defined by
| | { Or | | {
Example:
a. | |
b. | | ( )
Note:
The absolute value of a number is always non negative.
The absolute value of a number is, in other word the distance of a number from zero without
considering the direction
Properties of absolute value :For any two real numbers x and y:
a. | | d. | | | |
b. | | | | e. √ | | | |
c. | | | || |
f. | |
| |
for
| |
55
Telegram channel https://t.me/johnson201485
Example: Find the solution set of the following equation.
a. | x | b. | – x| c. |x– | –
Solution:
a. | x | x r x
x r x
x rx
{ }
b. | – x| x x x
{ }
c. |x– | ,
* + ∅ , because the absolute value of any real number is always non-negative number.
Theorem 3.2 Solution of |x| and |x|
For any real number a > 0,
The solution of the inequality |x| is – a < x < a.
The solution of the inequality |x| is – a ≤ x ≤ a.
In | x| < a, if a < 0 the inequality |x| < a has no solution.
The solution of the inequality |x| , x=0 if a=0
Example:
a. | | c. | x | e. |x – | –
b. | | d. | x | f. |x – | –
Solution:
a. | |
x
x
x
x
56
Telegram channel https://t.me/johnson201485
c. | x | x x x Then the solution set is given as s.
s 2 3
d. | x | , They have no solution, because the absolute value of any real number is greater than
or equal to zero.
e. |x – | – They have no solution, because the absolute value of any real number is greater than or equal
to zero.
f. |x – | – They have no solution, because the absolute value of any real number is greater than or
equal to zero.
Theorem 3.3 Solution of |x| and |x|
For any real number a, if a > 0, then
The solution of the inequality |x| > a is x < – a or x > a.
The solution of the inequality |x| is x ≤ – a or x ≥ a.
The solution of the inequality |x| > a is * + if a=0 and if a<0
The solution of the inequality |x| is if a<0
Example:
a. | x – | b. | | c. |x – | – d. |x – | –
Solution:
a. | x – | x or x x or x x or x
b. | – x | x or x x or x x or x
c. |x – | – , Then the solution set is the set of real number. Because the absolute value of any real
number is always greater than or equal to zero.
d. |x – | – , Then the solution set is the set of real number. Because the absolute value of any real
number is always greater than or equal to zero.
5.2.SYSTEMS OF LINER INEQUALITIES IN TWO VARIABLES
Competency
At the end of this subunit, students will be able to:
solve system of linear inequalities in two variables by using graphical method
57
Telegram channel https://t.me/johnson201485
When two or more linear equations involve the same variables, they are called a system of linear
equations.
An ordered pair that satisfies all the linear equations of a system is called a solution of the system. A
system of two linear equations in two variables often involves a pair of straight lines in the plane. The
solution set of such a system of equations can be determined from the graph and is the set of all ordered
pairs of coordinates of points which lie on both lines.
For instance: {
Note: In a system of equations, if “=” is replaced by “<”, “>”, “≤” or “≥”, the system becomes a system
of linear inequalities.
When two or more linear inequities involve the same variables, they are called a system of linear
inequities. An ordered pairs (region) that satisfies all the linear inequalities called the solution of the
system.
When we solve system of linear inequalities graphically if “≤” or “≥” we use a solid line if “<” or “>” we
use a broken line.
Example1: solve each of the following system of linear inequalities
Solution:
a. Consider the system of inequalities shown below:
The solution of this system is the set of all ordered pairs that
Satisfy both inequalities. This solution can be determined by
Graphing each inequality in the same coordinate plane as shown below.
Recall that the graph of each inequality is called a half-plane.
The intersection of the two half-planes represents the solution
To the system of inequalities. This solution is a region that
Contains the graphs of an infinite number of ordered pairs.
The graphs of
y = x + 2 (y ≥ x + 2)
y = -2x – 1 (y ≤-2x – 1)
58
Telegram channel https://t.me/johnson201485
Are the boundaries of the region and are included in the graph of the system.
b. Solve each system of inequalities by graphing
5.3.QUADRATIC INEQUALITIES
Competencies
At the end of this sub-unit, students will be able to:
Solve quadratic inequalities by using product properties.
Solve quadratic inequalities using the sign chart method.
Solve quadratic inequalities using graphs.
Definition: An inequality that can be reduced to any one of the following forms:
Or
Or
Where and are constants and , is called a quadratic inequality.
Quadratic inequalities can be solved by using different methods. Some of them are:
a. Product Properties or algebraic method.
b. Sign Chart Method
c. Graphical method.
A. Solving Quadratic Inequalities Using Product Properties
The product of two real numbers is positive, if and only if either both are positive or both are negative.
This fact can be used to solve the given inequality.
Product properties:
1. m.n > 0, if and only if i m > 0 and n > 0 or ii m < 0 and n < 0.
2. m.n < 0, if and only if i m > 0 and n < 0 or ii m < 0 and n > 0.
B. Solving Quadratic Inequalities Using the Sign Chart Method.
59
Telegram channel https://t.me/johnson201485
As x is moved along the number line, the quantity is sometimes positive, sometimes zero,
and sometimes negative. To solve the inequality, you must find the values of x for which
is negative. Intervals where is positive are separated from intervals where it is negative
by values of x for which it is zero. To locate these values, solve the equation .
Factorizing the equation and dividing into three interval and determine the sign in those three intervals
give the solution of the required.
C. Solving Quadratic Inequalities Graphically.
In order to use graphs to solve quadratic inequalities, it is necessary to understand the nature of quadratic
functions and their graphs.
If , then the graph of the quadratic function ( ) is an upward parabola.
If , then the graph of the quadratic function ( ) is a downward parabola.
The graph of a quadratic function has both its ends going upward and downward depending on whether
is positive or negative. From different graphs you can observe that the graph of a quadratic function
( )
Crosses the x-axis twice, if – .
Touches the x-axis at a point, if – .
Does not touch the x-axis at all, if – .
To solve a quadratic inequality graphically, find the values of x for which the part of the graph of the
corresponding quadratic function is above the x-axis, below the x-axis or on the x-axis.
Example: solve each of the following quadratic inequality graphically
A. B. C.
Solution: To solve the problem, first let us draw the graphs using the following information
a. and
X-intercept: The zeros of quadratic function which is
( )( +1) Y
Then x-intercept are (-5, 0) and (-1, 0) 𝑓(𝑥) 𝑥 𝑥
Y-intercept ( ) y, then ( ) , (0, 5) is y-intercept
5
Vertex . ( )/ ( )
-1 x
Therefore: The solution set of inequalities of -5
is ( )∪( ) ( )
is ( -∪, )
is ( )
is , -
60
Telegram channel https://t.me/johnson201485
Generally: In a quadratic equation for , the solution of quadratic inequalities
can be determined as follows using the following cases
Case one: If then the equation has no real roots. Moreover, the solution set of
is the set of real number if
is empty set if
is empty set if
is the set of real number if
Case two: If then the equation has exactly one real root which is , then the solution
set of
is the set of real number if is empty set if
is 2 3 if
is the set of real number
is the set of real number if
except if
is empty set if is the set of real number
is 2 3 if except if
Case three: : If then the equation has exactly two distinct real roots, let its roots are d
and e where d<e
is ( -∪ , )if is ( ) if
is ( )∪ ( ) if is , - if
is ( )if is ( -∪ , )if
is , - if is ( )∪ ( )if
61
Telegram channel https://t.me/johnson201485
UNIT SIX. PLANE GEOMETRY
Unit Outcomes
After completing this unit, students will be able to:
Know more theorems special to triangles.
Know basic theorems specific to quadrilaterals.
Know theorems on circles and angles inside, on and outside a circle.
Solve geometrical problems on quadrilaterals, circles and regular polygons.
Main contents:
Theorems on triangles
Special quadrilaterals
More on circles
Regular polygons
c. Bisector of line: A line that divides a line segment into two congruent line segments is called
B a
bisector of the line segment. D
d. Angle bisector: divides an angle into two equal angles. A
Any point of the bisector of an angle is equidistant from the sides of an angle, C
i.e., ≡
e. Perpendicular bisector: When a bisector of a line segment forms right angle with the line segment,
then it is called the perpendicular bisector of the line segment.
f. Incidence theorems: Theorems about collinear points and concurrent lines are called incidence
theorems.
62
Telegram channel https://t.me/johnson201485
Note: special segments on the theorem of triangles
Triangles have four types of special segments: these are
I. Median of triangle
II. Altitude of triangle
III. Angle bisector of triangle
IV. Side bisector of triangle
Special segment (line) of a triangle, which is median, altitude, angle bisector and side bisector of a triangle
are concurrent at a point. The intersection points of:
Median is called centroid
Altitude is called orthocenter
Perpendicular side bisector is called circum-center
Angle bisector is called in-center
A. Medians of triangle
A median of a triangle is a line segment drawn from any vertex to the mid-point of the opposite side.
Every triangle will have three medians.
Three medians of a triangle pass through the same point.
The lines are concurrent.
The point intersection the medians of triangle are called centroid of triangle.
Theorem 6.1 The medians of a triangle are concurrent at a point of the distance from each vertex to the
B. Altitude of a triangle
The altitude of a triangle is a line segment drawn from a vertex, perpendicular to the opposite side, or to
the opposite side produced.
Every triangle has three altitudes
63
Telegram channel https://t.me/johnson201485
In triangle, the three-altitude pass-through the same point, this common point is called ortho-center of
triangle
The three altitudes of a triangle are concurrent. i.e. (intersect at a point)
Three altitudes of:
I. Acute-angle triangle lies inside
II. Right-angled triangle lies the vertex of right angled
III. An obtuse-angle triangle lies outside of triangle
Bisector of triangle:
There are two bisector of triangle:
Angle bisector of triangle
Side bisector of a triangle
The perpendicular bisector of a side of a triangle:
Theorem 6.2
The perpendicular bisectors of the sides of any triangle are concurrent at a point which is equidistant from
A
the vertices of the triangle.
Note: E O D
B C
F
Every triangle has three perpendicular bisector and they intersect each other at a common point.
The perpendicular bisector of the sides and of a triangle intersect at O, This
common point is called circum-center of the triangle
The point O is equidistant from the three vertices A, B, C of triangle because O lies on
perpendicular sector of and of a triangle . Therefore radius
If we draw a circle with center O and radius , it will pass through B and C. The three perpendicular
side bisector of triangle for:
a. An-acute triangle lies inside the triangle. c. Right-triangle lies on hypotenuse of triangle
b. An-obtuse triangle lies outside the triangle.
Theorem 6.3: The altitudes of a triangle are concurrent.
C. Angle bisector of a triangle
An angle bisector of a triangle is a segment that bisects an angle of a triangle has one end-point at a
vertex of a triangle and the other end-point on the triangle.
64
B C
D
Every triangle has three angle bisector and they intersect each other at a common point, this
common point called in-center.
In triangle , and angle bisector, they intersect at a single point O such that
.
D. Altitude theorem: The altitude theorem is stated here for a right angled triangle. It relates the length
of the altitude to the hypotenuse of a right-angled triangle, to the lengths of the segments of the
hypotenuse.
Theorem 6.5 Altitude theorem
In a right-angled triangle ABC with altitude CD to the hypotenuse, AB
Note: The Square of the length of the altitude is the product of the lengths of the segments of the
hypotenuse.
65
Note that if the sides and of trapezium ABCD are congruent, then the trapezium is called an
isosceles trapezium.
B. Parallelogram: A parallelogram is a quadrilateral in which both pairs of opposite sides are parallel.
In Figure the quadrilateral ABCD is a parallelogram, then AB//DC and AD//BC
Properties of a parallelogram and tests for a quadrilateral to be a parallelogram are
Stated in the following theorem:
Theorem 6.7
a. The opposite sides of a parallelogram are congruent.
b. The opposite angles of a parallelogram are congruent.
c. The diagonals of a parallelogram bisect each other.
d. If the opposite sides of a quadrilateral are congruent, then the quadrilateral is a parallelogram.
e. If the diagonals of a quadrilateral bisect each other, then the quadrilateral is a parallelogram.
f. If the opposite angles of a quadrilateral are congruent, then the quadrilateral is a parallelogram.
C. Rectangle A B
A rectangle is a parallelogram in which one of its angles is a right angle.
Some properties of a rectangle
a. A rectangle has all properties of a parallelogram. D C
66
67
Theorem 6.14: The measure of an angle formed by a tangent and a secant drawn to a circle from a point
outside the circle is equal to one-half the difference of the measures of the intercepted arcs.
Given: Secant and tangent intersecting at P.
68
Theorem : If a secant and a tangent are drawn from a point outside a circle, then the square of the length
of the tangent is equal to the product of the lengths of line segments given by
(P ) (P ) (P )
Competencies
At the end of this sub-unit, students will be able to:
Calculate the perimeters of regular polygons.
Calculate the areas of regular polygons.
Introduction: A polygon whose vertices are on a circle is said to be inscribed in the circle.
The circle is circumscribed about the polygon.
A polygon whose sides are tangent to a circle is said to be circumscribed about the circle.
The pentagon PQRST is circumscribed about the circle. The circle is inscribed in the pentagon.
Perimeter of a Regular Polygon
The perimeter of a regular polygon is given as where n is the number of side and s is the length
of the side of regular polygon.
inscribed a polygon.
Area of a Regular Polygon
Theorem 6.16 : The area A of a regular polygon with n sides and radius r is
4 5
Example:
1. Find the area of a regular nine-sided polygon with radius 5 units.
2. Find the area of a regular twelve-sided polygon with radius 3 units
69
70
a. In a prism
Lateral edge
Lateral edges are equal and parallel
Lateral faces are parallelograms Lateral face
b. In right prism
Base Lower base
Altitude is equal to lateral edges and perpendicular to bases.
edge
Lateral faces are rectangles.
c. In oblique prisms
Altitude is shorter than lateral edges
d. A right prism with bases of regular polygon is called Regular prism
e. A right square prism whose altitude equals to length of edge of bases is called a Cube.
f. If bases are n-sided polygon prism then the prism has 3n edges and 2n vertices.
Surface area and Volume of Prisms
For a prism if its height= h
Lateral surface area = AL
Total surface area = AT
Base area = AB
Base Perimeter = p and Volume = V, then
The lateral surface area- the sum of the area of lateral face (AL) = (perimeter) x (height) = ph
Total surface area- the sum of lateral surface and the two bases (AT)= ph+2AB
Volume- the product of base area and height (v)= ABh
Cylinders:It is a circular flat base and a flat top.
The lower is the same as the top base
h
From the base to the top the shapes stays the same.
It is one curved side. It is not a polyhedron as it has a curved surface r
71
( ) ( x ) Sq. units
( ) Sq. units
Sq. units
Cub. Units
√ √
b. . / ( ) . / . √ / .
√ √ √
√ .
72
73
AT = Total surface area the sum of lateral face area and base area
Example:
a. An edge of a right square pyramid is 6cm long, if the length of the slant height is 5cm, then find the area
of its lateral surface, the total surface area and the volume.
b. The volume of a pyramid is . The pyramid has a rectangular base with sides 6 cm by 4 cm. Find the
altitude and lateral surface area of the pyramid if the pyramid has equal lateral edges.
c. Exercise 1: Find the altitude h AL and AT area of a right pyramid with volume of having a rectangular
base of dimensions 3cm by 4cm.
2: The volume a regular square pyramid is If its altitude is 10cm long, find the length of one edge of the
base and its total surface area.
3: If a lateral edge of a regular tetrahedron is x cm, find the measure of Altitude, surface area and volume
4: Show that the volume of a regular square pyramid whose lateral faces are equilateral triangles of side length s, is
√
( )
( ) ( ) √
74
√ √
. / . / ( √ √ ) .
B. Cones:The solid figure formed by joining all points of a circle to a point not on the plane of the
circle is called a cone.
If the base of cone is circular region, the cone is called circular cone.
Right circular cone: is a cone with the foot of its altitude at the Centre of the base
v
Slant height: A line segment from the vertex of a cone to any point on
Altitud Slant
the boundary of the base (circle) is called the slant height. e height(𝑙)
Surface area and volume of a right circular cone: r
The lateral surface area of a right circular cone is equal to half the product of its slant height and the
circumference of the base. That is,
( ) , where √
The total surface area (AT) is equal to the sum of the area of the base and the lateral surface area.
That is,
( )
The volume of a circular cone is equal to one-third of the product of its base area and its altitude.
That is,
, where V denotes the volume, r the radius of the base and h the altitude.
Example:
a. Calculate the total surface area and volume a right circular cone with length of altitude 8cm and
radius 6cm. Solution: h = 8cm, r = 6cm
b. The volume of a right circular cone is . The radius is 4cm.Find the length of the
perpendicular height.
c. Exercise 1: Calculate the total surface area and volume of a circular cone whose altitude and diameter
of base are equally x cm.
2: If the slant height of a circular cone is 13cm and the radius of a base is 12cm, then calculate; AL, AT
and V.
75
b. ( )
C. Spheres:
A sphere is a closed surface, all points of which are equidistant from a point called the Centre or is a
three-dimensional figure made up of all points a given distance from center.
Surface area and volume of a sphere: r
The surface area (A) and the volume (V) of a sphere of radius r are given by
Area of the sphere and Volume of the sphere
Example:
a. The diameter of a sphere is 6cm; find the area and volume of the sphere.
b. The radius of one sphere is twice as long as the radius of another sphere. If the volume of the smaller
sphere is 12 cubic units, then calculate the volume of the larger sphere.
c. If 18cm long wire whose radius of circular thickness is 4cm is melted to form a sphere, find the
surface area and volume of the sphere.
d. Find the surface area and volume a hemispherical metal of radius 12cm.
Exercise 1: A cylindrical container of base radius 8 cm has enough water in it. An iron ball of radius 3
cm is inserted in the cylinder. Assuming that the ball is completely immersed, how high does the
water level rise?
2: If a spherical stone with radius 6ocm is submerged in a cylindrical water tank whose base radius is 2m
then how much is the level of water raised?
Solution:
a. diameter ; x
76
b. Let R be the radius of the larger sphere and r be the radius of the smaller sphere
( )
(√ )
c. To change in shape may change the surface area but doesn„t change the volume.
Then let radius of the sphere, A = area of the sphere radius of cylinder, h = altitude of cylinder
volume of cylinder, volume of sphere
( )
( ) ( )
( ) And
Exercise: If the radius of a sphere is doubled, what effect does this have on its volume and its surface
area?
77
In any pyramid, the ratio of the area of a cross-section to the area of the base is where h is the
altitude of the pyramid and k is the distance from the vertex to the plane of the cross-section.
In both figures
And
So in both conditions we have the above statement;
Where h is the altitude of the pyramid and k is the distance from the
Vertex to the plane of the cross-section.
Surface area and volume of frustum of pyramids:
The lateral surface area (AL) of a frustum of a regular pyramid is equal to half the product of the slant height ( )
and the sum of the perimeter (P) of the lower base and the perimeter (P') of the upper base. That is,
( ) 𝐴𝑏
The total surface area of frustum of regular pyramid is the sum of
Lateral face area and the two upper and lower bases. 𝐴𝑏
( )
78
10
b. ( ) ( ( )) h‟=5 h
h
D
C
c. √ A O
10 B
( √ ) √
√ √ (√ )
(√ )
. / . / ( √ ) √
√
. √ / 4 √ √ √ 5
B. Frustum of cone: A frustum of a cone is a part of the cone included between the base and a
horizontal cross-section made by a plane parallel to the base.
Surface area and volume of frustum of cone.
𝑟
For a frustum of a right circular cone with altitude h and slant height , if the
circumferences of the bases are c and c', then the lateral surface area of
the frustum is given by 𝑟
( ) ( r r) (r r)
The total surface area is the sum of the lateral surface area and the area of the two bases.
79
(r r rr )
Where r is the radius of the bigger (the lower base of the frustum) cone and r' is the radius of the smaller
cone (upper base of the frustum).
Example:
a. Calculate the surface area and volume of frustum of cone with length of bases radii 7cm and 4cm that
has a slant height 5cm.
b. Exercise. A frustum of height 12 cm is formed from a right circular cone of height 16 cm and base
radius 8 cm. Calculate the total surface area of the frustum and volume of the frustum.
Solution: r r n ( ) ( )
Total surface area of frustum of cone is given as follows
(r r) (r r ) ( ) ( )
And
(r r rr ) ( ) ( )
80
81
i.
Domain of R x A: x, y belongs to R for some y B A .
ii.
Range of R x B: x, y belongs to R for some y A B .
NOTE: If R is a relation from A to B, then:
d. R 4 x, y : x B, y A and x - y 0
Example 2: Let A 1, 0, 2, 5 and let R x, y : x, y A and y 2 x 2 is odd . List the elements of R
Domain of R = Range of R A A
82
NOTE: From what we did above, you should have concluded that:
For any relation R ,
Then to determine the inverse R 1 of R we should either interchange the variables in the „variable part‟,
or in the „property part‟, but not both, and then write the rule in the simplest form, if necessary.
Given R : x, y ; P x, y
83
x 1
R 1 : x, y ; y .
2
x 1
of R is given by: R : y, x ; y 2 x 1 or R 1 : x, y ; y
1
Therefore, the inverse .
2
R 1 x, y : 2 y x 1 or R 1 x, y : x 2 y 1
1
Or R x, y : x 1 2 y or R 1
x, y : 2 y x 1 or any other equivalent form
a. R x, y : y is the father of x . d. R x, y : y x 2 4
b. R x, y : x 5 . e. R x, y : 2 y 1, x
c. R x, y : x y 1 . f. R x, y : y x and 2 x y 3
Solution:
a. R x, y : y is the father of x
1
Then R x, y : x is the father of y
Here, domain of R x : x has a father y range of R and
1
b. R x, y : x 5
84
c. R x, y : x y 1
R 1 x, y : y x 1 x, y : x y 1 R , because x y y x .
d. R x, y : y x 2 4
R 1 x, y : x y 2 4 x, y : y 2 x 4 or R
1
x, y : y
x 4 or y x 4 .
e. *( ) ∈ +.
f. R 1 x, y : x y and 2 y x 3 .
8.1.2. Graphs of Inverse Relations
First, let us compare graphs of R and R 1 .For instance consider the following examples.
Example 5: Consider the relation R 3, 1 , 4, 2 , 6, 3 , 5, 1 .
i. Its inverse R 1, 3 , 2, 4 , 3, 6 , 1, 5 .
1
Note that reflecting a coordinate plane along the line with equation y x , maps any point P a, b into
Q b, a . That is P a, b and Q b, a are mirror images one to the other along the line y x . (See figure
10.1, below)
85
The graphs of R and R–1 using the same coordinate axes are given below, figure 8.3.
y=x
When you fold the graph in figure 8.3 below,
y=x2
along the line y = x, you can see that x=y
2
of R-1 : coincide.
Figure 8.3. Graphs of R and R-1
Notice that the boundary parabolas and
Solution: a) *( ) + then R:
To draw the graph of R, first sketch the boundary line :
x-intercept: Then x-intercept is (1, 0)
y-intercept: . The y-intercept is (0, 1)
Thus, the boundary line passes through points (1, 0) and (0, 1).
Take test point P(0, 0) which is below the boundary line .
; P (0, 0) , which is true.
Therefore, the graph of R is the region which lies below or on the boundary line.
86
b) *( ) +
Then, the inverse of R is
*( ) + Or *( ) +
You can draw the graph of as previously done. (See figure 10.5 above)
B, we write f as or →
If is given by a rule that maps x from A to y in B, then we write ( ).
Notation: If x is an element in the domain of a function , then the element in the range that is associated
with x is denoted by ( ) and is called the image of under the function .
This means *( ) ( )+
Example 1: Suppose is the function that gives 5x – 1for any ∈ . The possible ways of
writing this function are
– ( ) – – → –
Example 2: Consider the relation R = {(1, 2), (7, 8), (4, 3), (7, 6)} Since 7 is paired with both 8 and 6,
and 8 6, the relation R is NOT a function.
87
a) b) c)
The graph in figure 8.6a does not define function, because a vertical line intersect the graph twice.
Whereas, The graphs in figures 8.6a and 8.6b do define functions, because any vertical line intersect the
graphs atmost once.
88
a) The sets , , , * +, * +, , (-5, 5), {-2, -1, 0, 1, 2}, {0}, [-5, 5], . /∪
b) The sets , * +, * +, [-5, 5), { -1, 0, 1, 2}, { 5}, [0, 5], , - ( - are not
symmetric sets.
Definition 8.5. Let f : A B be a function and let the domain A be symmetric set. Then is said to be
i) odd function if and only if for any ∈ ( ) ( ).
ii) even function if and only if for any ∈ ( ) ( ).
The evenness or oddness of a function is called its parity.
Example 7:
i) A constant function ( ) is even function, since ( ) ( ) for all ∈ .
ii) f (x) = x3 is odd function, since dom. of and f (–x) = (–x)3 = − x3 = − f (x).
iii) The function f (x) = x2 is even, since f (–x) = (–x) 2 = x2 = f (x).
iv) The function f (x) = x + 1 is neither even nor odd, since
f (–x) = –x + 1 –(x + 1) = – f (x) and f (–x) = – x + 1 x + 1 = f (x).
v) ( ) s is an even function, since ( ) s( ) s ( )
vi) ( ) sin is an odd function, since ( ) sin( ) s ( )
vii) The exponential function ( ) is neither even nor odd, since
( ) . / ( ) and ( ) . / ( )
viii) ( ) for if neither even nor odd, since the given domain , ) is not symmetric set.
Note:
ii. The trigonometric functions ( ) sin , ( ) s , ( ) t n and ( ) t are odd
functions.
iii. The trigonometric functions ( ) s , and ( ) s are even functions.
Remark:
i. If a function is an odd function, then its graph is symmetric w.r.t the y-axis.
ii. If a function is an even function, then its graph is symmetric w.r.t the origin.
NOTE: Using the definition of odd and even functions, we can justify the following:
89
Example 8:
a) The function ( ) and ( ) are an exponential function.
b) The function ( ) ( ) and ( ) ( ) are logarithmic
function.
8.2.2. Power Functions
Definition 10.6: A power function is a function which can be written in the form ( ) , where r
is a real number and a ℝ, is a fixed number.
90
( ) –32 –4 0 4 32
Then the graph of is given below: Figure 8.7a. Graph of 𝒇(𝒙) 𝟒𝒙𝟑
From the graph above, you can see that the graph is concave downward on the interval ( ), and
concave upward on the interval ( ). Point (0, 0) is the point on the graph of where the
concavity of the graph changes. The point (0, 0), where the graph is smooth, and changes its shape
from concave upward to concave downward, is called an inflection point.
ii. A power function ( ) , ∈ , when „r‟ is even positive integer
For instance, consider a power function ( ) .
Here and is even positive integer. Then Domain of = and Range of = [0, )
( ) ( ) ( ). Hence, ( ) is an even function.
Therefore, the graph of is symmetric w.r.t the y-axis. (i.e for every point (x, y) on the graph, (-x, y)
is also on the graph).
To understand the nature of the graph, let us calculate some of its functional values, as shown in the
following table.
Then the graph of is given below:
2 1 0 1 2
( ) 16 4 0 4 16
Note that the graph given above is open upward for all for all ∈ . Hence, the graph has no
inflection point.
iii. A power function ( ) , ∈ , when „r‟ is odd negative integer
91
( ) ( ) . / . / ( ), for all ∈ .
𝟑
Figure 8.8a. Graph of 𝒇(𝒙) 𝟐𝒙
( ) ( ) . / . / ( ), for all ∈ .
𝟐
Figure 8.8b. Graph of 𝒇(𝒙) 𝟐𝒙
92
( ) 1 1
( ) ( ) √ √ √ √ ( ) ( ), for all ∈
8 1 0 1 8
( ) √ 0 1
93
Note: In general, all functions of the form ( ) √ , where n is an odd natural number, have
similar behaviour. They all pass through (−1, −1) and (1, 1). They are also increasing.
𝑓(𝑥) 𝑥𝑛
𝟏
Figure 8.9b. Graph of 𝒇(𝒙) 𝒙 𝒏 , for an odd natural number
n.
ii. ( ) , where is odd natural number
94
𝑓(𝑥) 𝑥 𝑓(𝑥) 𝑥 𝑛
𝟏
Figure 8.10b. Graph of 𝒇(𝒙) 𝒙 𝒏 ,
Figure 8.10a. Graph of 𝒇(𝒙) 𝑥
where n is odd natural number.
Note: In general, all functions of the form ( ) , where n is an odd natural number, have similar
behaviour, as shown in figure 1.10b given above. From the graph we can see that
They all pass through points (−1, −1) and (1, 1).
They are decreasing on the intervals ( ) and on ( ).
Here don‟t say „They are decreasing on the interval ( )∪( ).
Hence, in general, ( ) , for even natural number n, is neither even nor odd function. Thus, power
𝟏
Figure 8.11a. Graph of 𝒇(𝒙) 𝒙𝟐 . 𝟏
Figure 8.11b. Graph of 𝒇(𝒙) 𝒙 𝒏 ,where n is even
natural number.
95
𝟏
Figure 8.12. Graph of 𝒇(𝒙) 𝒙 𝒏 , for an even natural
number
The following figures n. you some of the various possible graphs of power functions with rational
give
exponents .
96
𝒎
Figure 8.13b. Graph of 𝒇(𝒙) 𝒙 𝒏 , for
𝟐
Figure 8.13a. Graph of 𝒇(𝒙) 𝒙 𝟓. even m, odd n and m < n.
𝒎
Figure 8.14. Graph of 𝒇(𝒙) 𝒙 𝒏 , for odd m and even n.
Absolute Value (Modulus) Function: : The absolute value of a real number x is the distance of the point
P with coordinate x on a number line from the origin O.
Definition: For any real number , the absolute value or modulus of , denoted by | |, is defined by
a, if a 0
a .
a, if a 0
97
1. x 0 , for any ∈ . x x
4. , for any x and ∈ .
y y
2. x x , for any ∈ .
5. x y x y , for any x and ∈ .
3. xy x y , for any x and ∈ .
6. x x and x x , for any ∈ .
x 3 2 1 0 1 2 3
f x x 3 2 1 0 1 2 3
y x 0 yx Figure 8.15. Graph of 𝒇(𝒙) | 𝒙 |.
1, for x 0
by y f x sgn x 0, for x 0
1, for x 0
5
a. f 2 b. f
3
c. f 4 2 2 8
98
a. f 2 sgn 2 1, since 2 1.
5 5 5
b. f sgn 1, since 0 .
3 3 3
c.
f 4 2 2 8 sgn 4 2 2 8 0, since 4 2 2 8 0 .
NOTE: The signum function can be described in terms of modulus function as follow:
x
x x 1, for x 0
, if x 0 x
sgn x x , because undefined, for x 0 .
0, if x 0 x
x
1, if x 0
x
Graph of Signum Function: The table of values and the graph of f x sgn x are as given below.
x 3 2 1 0 1 2 3
sgn x 1 1 1 0 1 1 1
a. f x x sgn x b. f x x sgn x
Solution:
a. f x x sgn x .
To sketch the graph of , let us redefine as follow:
x 1, for x 0
f x x sgn x 0 0 0, for x 0
x 1, for x 0
99
x 4 3 2 1 -2/3 0 1 3/2 2 3 4
sgn x 1 1 1 1 1 0 1 1 1 1 1
x sgn x 5 4 3 2 -5/3 0 2 5/2 3 4 5
in ℝ n ( )∪* +∪( )
b. f x x sgn x
x 4 3 2 1 2 0 1 3 2 3 4
3 2 𝑓(𝑥) 𝑥 𝑠𝑔𝑛 𝑥
sgn x 1 1 1 1 1 0 1 1 1 1 1
x sgn x x 4 3 2 1 2 0 1 3 2 3 4
3 2
in ℝ n ( )
Definition: The greatest integer function, denoted by f x x is defined as the greatest integer .
NOTE:
1. The greatest integer is also called the floor of x .
2. The greatest integer is the integer that is immediately to the left of x ( or x itself, if x is an integer).
What is the largest among the integers that is less than or equal to 2.56 ? You can see that it is 2.
Thus, 2.56 2
b.
c. Similarly, x 7.2143 is between 7 and 8 .
7.2143 7
100
d. 0.6 is between 1 and 0 on number line.
NOTE: The greatest integer function which can be written in the form of
Thus domain of .and range of .
x 2 x 1 1 x 0 0 x 1 1 x 2 2 x3 3 x 4
f x 3
Solution:
x 2 x 1 1 x 0 0 x 1 1 x 2 2 x3 3 x 4
f x 2 1 0 1 2 3
The graph of the greatest integer function y x lies on or below the line y x so it provides an integer
floor for x .
The Least integer (ceiling) function, denoted by f x x is defined as the smallest integer .
Definition: The ceiling function also has type of Thus domain of .and range of .
a. x 2.8 b. x 2.8 c. x 12
Solution:
a. 2.8 is between 2 and 3 on number line.
c. 12 12 why?
101
1 5 9
Example 16: Solve the equation x x x 3 .
2 2 2
1
Solution: We can solve this equation more easily by using the substitution x m.
2
Therefore, we have the equation: m m 2 m 4 3 .
Now using the identity x n x n , we have:
Horizontal line
Crossing
point
102
Therefore f x 5x 2 is a one-to-one function.
This means there are numbers ∈ for which x1 x2 f x1 f x2 does not hold.
A horizontal line crosses the graph of y x 2 at two points. Thus, f is not a one-to-one function.
Example 3: Which of the following are one-to-one functions?
a. F x, y : y is the father of x . c. G x, y : x is a dog and y is its nose .
b. H x, y : y x 2
Solution: Only G is one-to-one. Why?
8.3.2. Onto Functions
Definition: A function f : A B is onto (a surjection), if and only if, Range of f B .
a. b.
103
) be given by f x x .
2
b. Let ,
then, f x f y y
2
y . Thus, f is onto.
a. The graph of f x x passes the horizontal line test. Thus, f is one-to-one. The range of f x x
5 5
is . So f is onto.
Therefore f is a one-to-one correspondence.
3x1 1 3x2 1
b. f x1 f x2 3x1 3x2
5 5
x1 x2 for any ∈ is n t n .
5 y 1
For any ∈ f is onto .
, , gives x
3
Therefore f is a one-to-one correspondence.
c. For ∈ ,
f x1 f x2 5x1 5x2
5x1
1
5x2
5x1 x2 1 50
x1 x2 0
x1 x2
104
Thus, f is one-to-one. But f is not onto because negative numbers cannot be images. For instance, take
y 25 . Since 5x 0 , for every ∈ , it is not possible to have ∈ , for which 5x 25 . Thus,
f is not onto.
Therefore, f is not a one-to-one correspondence
d. The graph of f x x passes the horizontal line test. Thus, f is one-to-one. The range of f x x
5 5
5 y 1
For any ∈ , , gives x f is onto . Therefore f is a one-to-one correspondence.
3
5x1
f. For ∈ , f x1 f x2 5x1 5x2 1 5x1 x2 1 50
5x2
x1 x2 0 x1 x2
Thus, f is one-to-one. But f is not onto because negative numbers cannot be images. For instance, take
y 25 . Since 5x 0 , for every ∈ , it is not possible to have ∈ , for which 5x 25 . Thus,
f is not onto.
Therefore, f is not a one-to-one correspondence
g. For x1 , x2 1, , f x1 f x2 ,
x1 1 1 x2 1 1 x1 1 x2 1
2 2 2 2
x1 1 x2 1
2
2
x1 1 x2 1
x1 1 x2 1, x1 1, x2 1 x1 x2 Thus, f is one-to-one.
105
8.4. COMPOSITION OF FUNCTIONS
Given two functions, it is sometimes possible to construct a third function based on the two functions. In
this subunit we will illustrate some of the methods of constructing such a third function.
8.4.1. Combination of functions
NOTE: Recall the following.
f g x f x g x
x 2 1 x 1
x2 x 2
f g x f x g x
x 2 1 x 1
x3 x2 x 1 then Domain of
f f x x2 1
x x 1; x 1
g g x x 1
106
f
Domain of Domain of f Domain of g \ x : g x 0
g
* + * +
Example 2: If f 1, 3 , 2, 4 , 5, 7 , 8, 9 and g 1, 4 , 2, 0 , 3, 5 , 8, 1 , then find
f
f g, f g, f g, and 3 f .
g
Solution:
f g 1, 3 4 , 2, 4 0 , 8, 9 1 1, 7 , 2, 4 , 8, 10
f 3 9
1, , 8,
g 4 1
f
Domain of 1, 8 . Why?
g
107
Solution:
a. gof a g f a g 1 e
b. gof d g f d g 3 h
c. gof c g f c g 2 f
NOTE: The domain of composite function gof is the set of all real numbers x such that x is in the domain of f
Solution:
a. gof 4 g f 4 g 42 1 g 15 3 15 2 47
gog 1 g g 1 g 3 1 2 g 1 3 1 2 1
108
.Solution: gof x fog x g f x f g x
g 4 x 1 f 3x k
3 4 x 1 k 4 3x k 1
12x 3 k 12x 4k 1
2
k 4k 1 3 k
3
Example 8: Let f x x 9 and g x 9 x 2 find gof x and its domain.
2
Solution: gof x g f x f 9
2
9 x2 9 x2
9 x2 9 x2
From this, it might appear that the domain of the composition is the set of all real numbers. This,
however is not true. Because the domain of f is the set of all real numbers and the domain of g is
3 x 3 , then domain of fog x is 3 x 3 .
109
Solution:
x3 x3
a. f x, y : y 4x 3 and f 1 x, y : x 4 y 3 x, y : y y f 1 x .
4 4
b. f 1 x x
y y y
x d. f 1 x x ln 2, y 1
y 1 y 1 y 1
x y 1 y y
x ln 2, y 1
xy x y y 1
xy y x y
ln
2
y 1
y x 1 x e ex
, y 1
y
x ex 2, y 1
y , x 1 y 1
x 1
3y 2
x ex , y 1
Therefore f 1 x y , x 1 y 1
x 1
c. f 1 x x 3 y 8 x 3 y 8 ex y 1 3 y 2
x3 y 8 yex 3 y ex 2
y f 1 x x3 8 ex 2
f 1 x y
ex 3
2x 4 y 1 2
e. f x, y : y and f. f 1 x x , y
1 x 3y 2 3
f 1 x, y : x
2y x 3 y 2 4 y 1
, y 1
1 y
3xy 4 y 2x 1
f 1 x, y : x xy 2 y, y 1
2x 1 4
f 1 x y , x
x 3x 4
f 1
x, y : y , x 2 3
x 2
Note that to find range of a given function use: Range of f Domain of f 1 .
For example, in example above range of in 2 3
Example 2: Show whether or not each of the following pairs of functions are inverses of each other.
3x 5 x4
a. f x 2 x 5 and g x c. f x 2 x 4 and g x
2 2
1 3x 1 x 1
b. f x and g x d. f x 2 x 1 and g x
x 3 x 2
Solution: Functions f and g are inverses of each other if and only if f g x x g f x .
3x 5
a. f g x 2 5 3x
2
f g x 3x x
f and g are not inverses of each other.
110
f g x x
f g x
1 1 1
b. x
3x 1 3x 1 3x 1
3
x x x
f and g are inverses of each other.
x4
c. f g x 2 4 x f and g are inverses of each other.
2
x 1
d. f g x 2 1 x 2
2
f g x x f and g are not inverses of each other.
Example 3: Sketch the graph each of the following function and its inverse on the same coordinate:
a. f x 2x 1 b. f x x , x 0
2
c. f x x 2, x 0
2
Solution:
a. f 1 x x 2 y 1 y f 1 x
x 1 b. f 1 x x
2
c. f 1 x y x 2, x 2
111
Exercise 8.8
1. Determine the inverse of each of the following functions and determine the domain and
range of its inverse. Is the inverse a function?
1
a. f x log3 2 x c. h x 1 x e. f x g. g x 5 x 7
x 1
1
g x 5x 13 d. k x x 2 f. g x 5 x3 1 h. f x
2
b.
e 1
x
2. Are the following functions inverses of each other( in the respective domain )?
x2
a. f x 3x 2; g x d. f x x3 ; g x 3 x
3
b. f x x ; g x x2 e. f x x5 3; g x 5 x 3
x 1 x 1 2 x 1
c. f x 2 x5 1; g x 5 f. f x ; g x
2 x2 x 1
3. Which of the following functions are invertible? If they are not, can you restrict the domain
to make them invertible?
1
a. f x x3 b. g x 4 x
2
c h x x 5 d. f x log x
2
3
4. Which of the following functions are invertible and find its inverse if it is invertable.
a. c. e. c.
b. d. c.
5. For each of the following functions, sketch the graph of f and f 1 on the same coordinate
plane, along y x .
a. f x x3 b. g x x 7 3 c. f x x 5 d. f x 2 x 3 e. h x 3 x 3 f. f x 3 x 7 5
3
3. R b, a : a, b R .
1
9. x x, for x 0 .
x, for x 0
10. x x 2 .
1, for x 0
11. sgn x
0, for x 0 .
1, for x 0
12. The floor function or the greatest integer function f x x maps into .
13. The Least integer (ceiling) function f x x maps into .
14. f is one-to-one, if and only if f x1 f x2 x1 x2 , for any x1 , x2 Domain of f .
15. A numerical function f f is one-to-one, if and only if no horizontal line crosses the graph of f more
than once.
16. f : A B is onto, if and only if Range of f B .
17. f : A B is a one-to-one correspondence, if and only if f is one-to-one and onto.
18. fog x f g x .
19. Domain of fog Domain of g .
20. f 1 is a function, if f is one-to-one.
21. g and f are inverse functions of each other, if and only if g f x x and f g x x .
22. To find f 1
Write y f x .
Interchange x and y in the above equation to obtain x f y .
Solve for y and write y f x .
1
1. Find the inverse of each of the following relation and determine whether the inverse is a function.
a. R 2, 2 , 3,3 , 4, 4 c. R 5, 13 , 6, 10.8 , 3, 11.4 , 10,14
d. For each of the following functions, what is the effect of 4, when f x is compared with f x3 ?
i. f 4 x3 ii. f x3 4
6. Sketch the graph the following functions, find the domain and the range.
a. f x x sgn x b. f x x sgn x
3
c. f x sgn x 2 3
1
7. Check if 2 x x x by taking different values of x .
2
f
a. f g 2 b. gof 3 c. 5 d. f g 10
g
10. Suppose f is a function on the set of integers defined by:
c. g 1of
1
d. fog
1
a. g 1of 1 b. fog 1
1
ln 1 x ln x B. e x 1
1
A. C. ln D. x
x 1 e 1
A.
1
ln 3
x 1
B.
1
ln x 3 1
C. ln 1 x 3
D. 1 e x
3
f x y f x f y C. f x 2 1
1
A. f x2
3
B. f xy f x f y D. f x 1 f x
1
, if x 3
17. Given functions f ( x) x x 3 and g ( x) x 1
2
, then gf (2) is equal to:
x 2 1, if x 3
1
A. B. 3 C. 26 D. 82
4
18. Which of the following represents the graph of the relation R x, y | x y ?
A. Y C.
Y
X
X
Y Y
B. D.
X X
x
19. If f ( x) and g ( x) x 1 , then which of the following is true?
1 x
2x 1
A. gf ( x) C. The domain of gf ( x) x | x 1
1 x
29. The graph shown below is the graph of a certain relation R. What is the range of R 1 ?
Y
A. 2,
B. 1,
C. , 2
X
D. , 1
A. C.
B. D.
9.1.STRAIGHT LINE.
9.2.CIRCLE.
a) b) c)
Figure 9.1
CASE I: When P and Q are on a line parallel to the x-axis (that is, ̅̅̅̅ is a horizontal segment)
As in the Figure 3.1a, since the two points P and Q have the same y-coordinate (ordinate), the distance
between P and Q is given by PQ = |x2 – x1|
For instance; the distance between points A(3, 5) and B(4, 5) is
|x2 – x1| = |4 (3)| = |7| = 7 ; since A and B have the same ordinate.
CASE II: When P and Q are on a line parallel to the y-axis (that is, ̅̅̅̅is a vertical segment)
As in the Figure 3.1b, since the two points P and Q have the same x-coordinate (abscissa), the distance
between P and Q is given by PQ = |y2 – y1|
For instance; the distance between points P(1, 4) and Q(1, 6) is
|y2 – y1| = |6 4| = |10| = 10 ; since P and Q have the same abscissa.
= √( √ (√ )) ( √ ( √ ))
√(√ ) ( √ √ ) √ √
√ √ units
Example 2: What are the possible values of b if the point A(b, 4) is 10 units away from the point B(0, –
2)?
Solution:Given : AB = 10 units where A(b, 4) and B(0, –2).
√( ) ( )
√( ) ( ( )) √
or
Therefore, the possible values of b are and 8.
B. Division of a Line Segment
Given a line segment PQ with end point coordinates P (x1, y1) and Q(x2, y1), let R be a point between P
and Q that divide the line segment ̅̅̅̅ internally in the ratio m : n. Then the coordinates (x0, y0) of R are
given by ( ) . /
. / . /
Example 4: A line segment has end-points P(–4, –3) and Q(2, 5). Find the coordinates of the points that
trisect the segment ̅̅̅̅.
Solution: Let ( ) and ( ) be two points that trisect ̅̅̅̅. This implies that A and B divide ̅̅̅̅
into three congruent smaller segments. That is PA = AB = BQ
First point A divides the ̅̅̅̅ in the ratio 1:2, and hence Q(2, 5)
PA : AQ = 1 : 2
( ) . /
B
( ) ( ) ( ) ( )
. / . /
A
Secondly point B divides the ̅̅̅̅ in the ratio 2: 1, and hence
P(4, 3)
PB : BQ = 2 : 1
( ) ( ) ( ) ( )
( ) . / . /
( ) . /
NOTE:
1. Vertical lines have no gradient.
2. The gradient of a horizontal line is zero.
y y y y
x x x x
NOTE : Three distinct points are said to be LIE ON THE SAME LINE (or COLLINEAR) if the line
that contains any two points also contains the third.
R
C Q
B
A P
Points A, B and C are collinear. Points P, Q and R are not collinear.
RULE :
1. Three distinct points are A, B and C are lie on the same line (or collinear) iff
̅̅̅̅ ̅̅̅̅( ̅̅̅̅)
̅̅̅̅ ̅̅̅̅ . Therefore, points A, B and C don‟t lie on the same line.
√ √ √ √ √ √
b. ̅̅̅̅ √ , and
√ √ √ √
√ (√ ) √ √
̅̅̅̅ √ .
√ √ √ √
Example 9: Find the possible values of k so that the following triplets of points are collinear.
a. ( ) ( ) ( ) b. ( ) ( ) ( )
Solution
a. ̅̅̅̅ and ̅̅̅̅
Here, there are four cases: Let be a line with angle of inclination θ.
y y y y
θ 0
θ 0
θ θ
x x x x
Figure d:
Figure a: Figure b: Figure c:
An inclined line to
A horizontal line. A vertical line. An inclined line to the left.
θ 0 θ 0 the right. 900 < θ 0
Slope m = 0 Slope m = 00 θ 0
m < 0 (negative)
m > 0 (positive)
REMARK: Let be a non-vertical line with an angle of inclination θ. Then slope of = tan θ
0 0
If is vertical line, then θ = 90 and slope of = tan 90 =
Recall the following trigonometric values of special angles:
sin θ cos θ tan θ
00 = 0 rad. 0 1 0
300 = rad. √ √ √
450 = rad. √ √ 1
600 = rad. √ √
900 = rad. 1 0
1200 = rad. √ √
1350 = rad. √ √ 1
1500 = rad. √ √ √
Solution :
a. t n t n √
b. t n t n t n
c. t n t n t n √ √
Example 11: Find the angles of inclinations of the lines having each of the following slope:
a. b. √ c.
Solution :
a. t n 0
b. √ t n √ rad
c. t n 0
y=2
x
A
(iii) TWO-POINT-FORM : The equation of a non-vertical line passing through two points
(x1, y1) and (x2, y2) is given by 𝑦 𝑦 𝑦 𝑦
𝑥 𝑥 𝑥 𝑥
Example 12. Give the equations of the lines passing through the points P(–1, 3) and Q(4, 0).
( ) ( )
( )
𝑥 𝑦
𝑎 𝑏
,
Ax + By + C = 0 Ax + By + C = 0 By = Ax . /,
. / , is the x-intercept.
Example 13. Find the equation of lines passing through (1, 2) and making angle 30° with positive y-axis.
Solution. Put (x1, y1) = (1, 2)
The line makes 30° with +y-axis implies the angle of inclination of the line is
= 300 + 900 = 1200
The slope of the line is t n t n √
Therefore, the equation of the line is y – y1 = m(x – x1)
y–2= √ (x – 1)
y= √ √
The y-intercept is √
√ √
When y = 0, √ √
√
Example 14. Find the slope and intercepts of the line having equation
Solution. First write the given equation in the standard form of equation of line.
. / . /
Or
y–2= ( )
4y – 3x + 1 = 0
Let (h, k) be the points on the line is 5 units away from the point A. then by distance formula
√( – ) ( – ) =5
( – ) ( – ) ------------------------------------- (i)
Since (h, k) is on the line, we have
4k – 3h + 1 = 0
------------------------------------- (ii)
Example 16. Write an equation for the line that passes through the point (6, 2) and is parallel to the line
3x + 4y = 10.
Solution: Let us first figure out the slope of this other parallel line. Transforming the given equation given
to slope-intercept form and read the slope. 3x + 4y = 10
The slope of the given line is .If our line is to be parallel to this line, its slope must be the same.
Now, the new line passes through the point (x1, y1) = (6, 2) and having slope
y ( ) (x )
Perpendicular Lines: If two non-vertical and non-horizontal lines ℓ1 and ℓ2 with slopes m1 and m2,
respectively, are perpendicular to each other, then the product of their slopes is 1.
That is if ℓ1 ℓ2, then m1 . m2 = 1
The converse is also TRUE.
Suppose ℓ1 and ℓ2 be two non-vertical and non-horizontal lines with slopes m1 and m2, respectively.
if m1 . m2 = 1, then ℓ1 ℓ2.
Example 17. Find the equation of the straight line passing through (1, 2) and perpendicular to the line x +
y + 7 = 0.
Solution : Let m be the slope of the line whose equation is to be found out which is perpendicular to the
given line x + y + 7 = 0. The slope of the given line y = (– 1) x – 7 is m = – 1.
Therefore, using the condition of perpendicularity of lines, we have (– ) – or is the
slope of the required new line.
( )
The altitude drawn to side ̅̅̅̅ is perpendicular to side ̅̅̅̅ and so its slope must be the negative reciprocal
of slope of ̅̅̅̅ . That is the slope of the required altitude is
Hence, the required altitude passes through B(1, 7) and having slope .
Therefore, the equation of the required altitude is y – 7 = 2(x – 1) or y – 7 = 2x – 2 or y = 2x +
9.1.2. Angle Between Two Lines on the Coordinate Plane
When two lines are intersecting, the angle between the two lines can be determined using their slopes.
DEFINITION: The angle between two intersecting lines ℓ1 and ℓ2 is defined to be the angle θ measured
counter-clockwise from ℓ1 to ℓ2.
THEOREM: If β is the angle between the two lines ℓ1 and ℓ2 measured from ℓ1 to ℓ2 counter-clockwise
and m1 and m2 their slopes, respectively, then the tangent of β is given by
n n
t n , where .
n n
Example 19. Given points P (2, 3), Q (–4, 1), C(2, 4) and D(6, 5), find the tangent of the angle between
the line that passes through P and Q and the line that passes through C and D when measured from the line
that passes through P and Q to the line that passes through C and D counter-clockwise.
Solution: Let m1 be the slope of the line through P and Q and m2 be the slope of the line through C and D.
Then, and
t n . t n
Example 20. Find the acute angle between the line and √
√ . and
√
√ √
Let be the angle between the two lines. t n
√
√
. /
√
The distance from a point to a line can be obtained using the following theorem.
THEOREM: The distance d from the point (h, k) to the line ℓ : Ax + By + C = 0, where A 0 or B0
| |
is given by √
Example 21. Find the distance of the point (3, 1) from the line y = x + 1.
Solution: First write the line in the general form y = x + 1
Put (h, k) = P(3, 1) and ℓ : x y + 1 = 0 . Then the distance from P to ℓ is
| | | ( ) ( ) | | | √
√ √ ( ) √ √
Distance between two parallel lines :The distance d between two parallel lines y = mx + c1 and y = mx +
| |
c2 is given by √
Example 22. Find the distance between the lines 3x + 4y = 9 and 6x + 8y = 15.
Solution : The equations of lines 3x + 4y = 9 and 6x + 8y = 15 may be rewritten as :
3x + 4y = 9 and 6x + 8y = 15
Since, the slope of these lines are same and hence they are parallel to each other with
and
| |
Therefore, the distance between them is given by
√ ( )
(𝑥 ) (𝑦 𝑘) 𝑟
The above equation is called the STANDARD FORM of the equation of a circle, with centre C (h, k)
and radius r.
Conversely, for r > 0, an equation ( ) ( )
represents a circle with center (h, k) and radius r.
NOTE: If a circle is centered at the origin (0, 0), then and , and the equation becomes
b. ( ) ( ( )) ( ) √
is an equation of a circle with center C( ) and radius r = √ .
Consider a circle with center C (h, k) and radius r having equation ( ) ( )
Expanding the equation gives ------------- (a)
Letting and , then equation (a) can be written as
𝑥 𝑦 𝐷𝑥 𝐸𝑦 𝐹 -------- (b)
𝐷 𝐸 𝐷 𝐸 𝐹
(𝑥 ) (𝑦 )
------------------- (d)
point, since the equation is satisfied by the point . / only. Sometimes the equation is said to be a
point circle.
. / ( )
. / ( ) √
√
Therefore, represent a circle with center . / and
𝓵 𝓵
O O
| | | ( ) ( ) |
√ √ ( )
| |
√
√ √
( ) ( ) √
( ) ( )
Or
Example 30. Find the equation of the tangent to the circle with center at ( )and point of tangency at
( ). 𝓵 P(-1, 3)
Solution : It is given that center : O(2, 0) = (h, k)
point of tangency : P( ) ( ).
The tangent line is passing through P( ) ( ) and O(2,0)
perpendicular to ⃡ . Slope of ⃡ is
Example 31. Give the equation of the line tangent to the circle with equation
(x + 1)2 +(y − 1) 2 =13 at the point P(–3, 4).
Solution : Center of the circle : C(–1, 1) = (h, k) and
Point of tangency : P(–3, 4) = T(xo, yo).
Then the equation of the tangent line is
( )
( )
Therefore, the equation of the tangent is
r is the radius.
1
15. The equation x2 y 2 Bx Cy D 0 represents a circle of positive radius r B2 C2 - 4D and
2
B C
center , if and only if B2 C2 - 4D 0 .
2 2
16. The line that touches a circle at only one point is called a tangent line and its equation is
y y0 x0 h
where x0 , y0 is the point of tangency and h, k is the centre of the circle.
x x0 y0 k
b. Q 1, 1 ; 2 x 2 y 4 0 d. B 2, 0 ; 3x 4 y 0
5. If the length of the perpendicular drawn from the point 15x 8 y 5 0 to the straight line is 10 units,
then find all the possible values of y .
6. Find the distance between the pairs of parallel lines whose equations are given below:
a. y 2x 4and y 2x 1 c. 2x 3 y 2 0 and 2x 3 y 6 0
b. 4 y 3x 1and 8 y 6 x 7 0 d. x 3 y 6 0 and x 3 y 24 0
7. Write the equation of each circle with the given conditions:
a. centre at O 3, 7 and radius r .
9. Find the equation of the circle with radius 5 and concentric with circle x2 y 2 8x 7 .
7 7 9 6
A. 1, B. (1,1) C. ,0 D. ,
5 5 5 5
4.Line l passes through 0, 5 and 5, 0 . What is the angle between the y axis and l in radian
measure?
3
A. B. C. D.
4 3 2 4
5.Consider a circle whose center is on the x axis . If a line given by y x is tangent to the circle at point
2, 2 , what is the equation of the circle?
x2 y2 8 C. x 4 y 8
2 2
A.
B. x 22 y 2 4 D. x 1 y 2 5
2
6.Suppose l1 and l 2 are perpendicular lines intersecting at 2, 1 . If the angle of inclination of l 2 is 45 0 ,
9.A circle of radius 1 unit is internally tangent to the circle x 2 y 2 2 y 8 at 3, 1 . Then, the equation
of the smaller circle is:
x 2 y 1 1 C. x 1 y 2 1
2 2 2
A.
B. x 22 y 12 1 D. x 4 y 1 1
2 2
10. What is the equation of the circle that is tangent to the line 3 y 4 x 5 and has center at 2, 1 ?
2 2
2 11
A. x y 2 C. x 2 y 2 4 x 2 y 16
5 5
B. x 22 y 12 4 D. x 2 4x y 2 2 y 1
11. Which one of the following is equation of a circle whose center is on y axis and radius is 3 ?
x2 y2 6y 0 C. x 2 y 2 3
2
A.
B. x 22 y 2 9 D. x 2 2x y 2 8
12. The equation of the line that passes through 2, 1 and perpendicular to 3x 4 y 6 is:
A. 4x 3 y 5 B. 4 x 3 y 5 C. 4 x 3 y 11 D. 4 x 3 y 11
13. If the graphs of the equations ax by c 0 and Ax By C 0 are perpendicular, then which of
the following is necessarily true?
A. aA bB 0 B. aB bA 0 C. aA bB 0 D. aB bA 0
14. Let C be the circle whose equation is x 2 y 2 2x 2 y 2 0 . Let L be the line whose equation is
y x 4 . Which one of the following is true?
A. L is tangent to C . C. C is below L .
B. L intersects C at two points. D. C is above L .
15. If the end points of a diameter of a circle are 2, 3 and 5, 1 , then which one of the following
equations represent the circle?
2
B. x 2 y 2 7 y 4 x 13 0 D. x 2 y 2 7 x 4 y 13 0
16. Which of the following equations represents the circle passing through 1, 1 and 1, 1 ?
A. x 12 y 12 8 C. x 1 y 1 2 2
2 2
B. x 12 y 12 8 D. x 1 y 1 2 2
2 2
17. What is the equation of the tangent line to the circle with equation x 3 y 4 20 at P(1, 0) ?
2 2
x 1 D. y 2x 1
1 1
A. y x B. y C. y 2x 1
2 2
18. Consider a circle whose equation is given by x 3 y 4 20 . Then what is the equation of the
2 2
Main Contents:
10.1. STATISTICS
10.2. PROBABILITY
Key terms
Summary
Review Exercises
The word statistics comes from the Italian word "statista" meaning stateman. It was used to signify the application
of recorded data for purposes of the state. When statistics is used in its plural sense, it means a body of numerical
facts and figures. The numerical facts are called statistical data, or simply data. When it is used in its singular form,
statistics is a branch of mathematical science, and is concerned with the development and application of methods
and techniques for the collection, organization, analysis and interpretation of quantitative data. We will confine
ourselves to this second meaning of statistics through this unit.
10.1.STATISTICS
10.1.1. Types of Data
ACTIVITY 10.1
1. Classify the following data as qualitative or quantitative:
a. beauty of a picture d. number of children living in a house
b. size of your shoe e. colour of your skin
c. type of a car f. blood type(group)
2. Classify the following variables as discrete or continuous:
a. size of a shirt d. number of rooms in a house
b. number of members of a football club e. heights of students in a class
c. price of a kilo of sugar f. life-time of an electric bulb
The first step in applying statistical methods is the collection of data; this is the process of obtaining counts or
measurements. The data obtained can be classified into two types: qualitative or quantitative data.
Qualitative data (or categorical data), is obtained when a given population or sample is
Definition 10.1: classified in accordance with an attribute that cannot be measured or expressed in numbers,
while Quantitative data is that obtained by assigning a real number to each member of the
population, under study.
Honesty, height, weight, intelligence, income, efficiency, width, sex, pressure, distance, religion, social status.
Solution: Honesty, intelligence, efficiency, sex, religion and social status are qualitative, while height, weight,
income, width, pressure and distance are quantitative.
[If I.Qs (intelligent quotients) are used to measure intelligence, then it will be quantitative.]
Definition 10.2: A number, which is used to describe the attribute and which can take different values is
called a variable.
For example, in your class the height, weight or age of different individuals varies, and can be expressed in
numbers. Therefore, these quantities (height, weight, age,…) are variables.
NOTE:
Variables are denoted by letters such as x, y, z,...
A variable may be either discrete or continuous.
Example 2: Which of the following are discrete variables? Which are continuous?
Number of students in a class, weight of students, length of a road, number of chairs in a room, temperature of a
room, the number of heads in a sequence of coin tosses, the result of rolling a die, the number of patients in a
hospital and number of houses along a street.
Solution: Number of students in a class, number of chairs in a room, the number of heads in a sequence of coin
tosses, the result of rolling a die, the number of patients in a hospital and number of houses along a street are
discrete. They can have whole number values only. On the other hand, weight of students, length of a road and
temperature of a room are continuous variables. They can take fractional or decimal values. For instance, weight of
students could be given by values like 50.1kg , 49.73kg ; length of a road could be given by values like 6.5km,
2.63km, while temperature of a room could be given by values like 20o c, 14o c .
NOTE:
1. Discrete data have no decimal places, the average of these values can be fractional. For example,
families can have only a discrete number of children: 1, 2, 3, etc. However, the average number of
children per family can be 2.2.
Frequently, you‟ll use bar charts to graph discrete data because the separate bars emphasize the distinct
nature of each value. However, it‟s appropriate to use other graphs as well.
2. We can use histograms and scatterplots to graph continuous data. These graphs are designed to
handle values that fall on a continuous spectrum and have decimal places.
Example 3: Consider the following data. It represents the number of patients that a doctor visits per day for 150
working days.
Solution: The data given is raw data or ungrouped data. To summarize the raw data into a grouped frequency
distribution, follow these steps:
NOTE:
From the formula, the class interval, w, is calculated as 5.8. For practical purposes, it will be convenient to
choose the class interval to be a whole number. For this case, you can take class interval as 6. (This is obtained
by rounding 5.8 to the nearest whole number). Therefore (See the grouped frequency distribution below).
In the above frequency distribution, you are considering frequencies of each class. But, in reality you may be
interested to know about other issues such as how many days the doctor visited fewer than 8 patients. To answer
such a question, the frequency distribution given above may not always be suitable. For such a purpose, you need
to construct what is called a cumulative frequency distribution.
A cumulative frequency distribution is constructed by either successively adding the frequencies of each class called
“less than cumulative frequency” or by subtracting the frequency of each class from the total successively called
“more than cumulative frequency”.
The cumulative frequency distribution of the above data of patients that a doctor visits per day is as follows.
The first and the last elements of a given class interval are called class limits.
Definition 10.5:
Exercise 0.1
Prepare a grouped frequency distribution, using 7 classes. Answer the following questions.
1. The average of the upper and lower class limit is called the class mark or class midpoint.
The class mark serves as representative of each data value in a class (or the class itself).
Definition 10.6:
2. The correction factor is half the difference between the upper class limit of a class and
the lower class limit of the subsequent class.
Solution: In this distribution, the correction factor i 1 26 25 0.5 or 1 51 52 0.5 or 1 76 75 0.5
2 2 2
Why do you need the correction factor?
Previously, you saw that a cumulative frequency distribution of discrete values may help to answer some questions. But, there
could be more questions to answer. For example, in Example 5 above, suppose you are asked „To which class does a mark of
9.5 belong? Or, how many students have scored less than 9.5?‟ To solve such problems, you have to smoothen the distribution
and fill the gaps. In order to smoothen, you add the correction factor to the upper limits of each class and subtract from the
lower class limits of each class to get what are called class boundaries.
Then the class 25.5–50.5 includes variable values that are 25.5 and above, but below 50.5.
Steps to construct a frequency distribution:
1. Find the highest and lowest values. 3. Select the number of classes desired
2. Find the range (i.e., highest value – lowest value).
4. Find the class interval by dividing the range by the number of classes and rounding up.
5. Select a starting point (usually the lowest value); add the class interval to get the lower limits.
6. Find the upper class limits. 8. Find the frequencies.
7. Tally the data. 9. Find the cumulative frequency.
Example 6: Construct the frequency distribution table for the following data using 6 classes.
12 18 27 31 40 42 14 20 27 32 40 51
56 40 32 27 20 14 21 29 14 34 40 60
16 23 31 36 45 65
Solution:
Find the highest and lowest values=Highest value 65 and lowest value 12
Calculate the range of the data set=Range 65 12 53
Select the number of classes desired= Number of classes desired is 6.
Class interval
53
8.83 . Round
8.83 to 9 which is called class width.
6
Now construct a frequency distribution table.
Data (Interval) Frequency Cumulative frequency
12 20 8 8
21 29 6 14
30 38 6 20
39 47 6 26
48 56 2 28
57 65 2 30
i. What is the class interval? iv. What is the frequency of the first class?
ii. What is the lower class limit of the second class? v. Complete the following table about Tree type A.
iii. What is the upper class limit of the second class?
vi. Make a similar table for Tree type B.
vii. Draw histograms to illustrate both frequency distributions.
10. The following are yield in quintals of wheat harvested by thirty farmers per hectare.
Prepare a grouped frequency distribution, using 11 classes. Answer the following questions.
a. What is the lower class limit for the third class?
b. What is the lower class boundary for the seventh class?
c. Determine the correction factor for this frequency distribution.
d. What is the class mark of the second class?
e. Find the difference between the class marks of the eighth and ninth classes.
The mean of a set of data is equal to the sum of the data items divided by the number of
items contained in the data set. If are values, then their mean is given by
Definition 10.6:
Solution:
x1 x2 x3 ... xn 3 7 5 13 2
a. x 2.4
n 5
b. Let us prepare a frequency distribution table.
x 2 6 7 8 10
f 3 5 9 2 6
Example 8: A group of 5 water tanks in a farm have a mean average height of 4.7 metres. If a sixth water tank
with a height of 2.91 metres is erected, what is the new mean average height of the water tanks?
Solution: Let x1 , x2 , x3 , x4 , x5 be the five water tanks and x6 be the sixth water tank.
x1 x2 x3 x4 x5
From the given we have 4.7 x1 x2 x3 x4 x5 5 4.7 23.5
5
x1 x2 x3 x4 x5 x6
The new mean average height of the water tanks is given by
6
x1 x2 x3 x4 x5 2.91 23.5 2.91 26.41
but x1 x2 x3 x4 x5 23.5 4.402
6 6 6
Therefore, new mean average height of the water tanks is 4.402meters .
NOTE:
To calculate the mean for two graphs of data, first we need to calculate the number of values of each group and
also the total number of values.
Generally, to find the mean of two groups of data,
Example 9: One group of 8 students has a mean average score of 67 in a test. A second group of 17 students has
a mean average score of 81 in the same test. What is the mean average of all 25 students?
Mean1 Number of values of 1st group Mean 2 Number of values of 2nd group
Solution: Mean
Total Number of values obtained from the groups
67 8 8117 536 1377
Mean 76.52
25 25
Solution: If you have to use what you know so far to calculate the mean, we need to know the total number of
students that took the test and the total number of marks that they scored.
The total number of students is 100, but we have a problem when it comes to the total number of marks. Since you
have grouped data, you cannot obtain individual marks. For instance, 13 students scored between 26 and 30. But,
there is no way you can tell the total mark of the 13 students.
The way out of this problem is to approximate each student‟s mark by the middle mark of the class interval, as in
the following table:
f1 f 2 ... f n n
f
i 1
i
Example 12: The following is the frequency distribution of a grouped data. Find the mean of the data.
Class Interval Frequency f
37 2
8 12 2
13 17 10
18 22 6
Solution:
Class Class mid- Frequency fxc
Interval point xc f
37 5 2 10
8 12 10 2 20
13 17 15 10 150
Therefore, the mean
18 22 20 6 120
fx 300
x 15 . f 20 fx 300
c
f 20 c
The procedure for finding the mean for grouped data assumes that all of the raw data values in each class are equal
to the class mark of the class. In reality, this is not true. However, using this procedure will give us an acceptable
approximation of the mean, since some values usually fall above the class mark and others fall below the class mark
for each class.
a. Find the average number of quintals of fertilizer distributed to the farmers from the raw data.
b. Prepare discrete frequency distribution and calculate the mean.
4. Using the data given in Question 3 prepare two grouped frequency distributions, using 6 and 9 classes.
Answer the following questions.
i. Find the mean of each. iii. Write your generalizations.
ii. Are the four means you calculated equal?
5. A school has three classrooms for grade 11, namely, 11A, 11B and 11C . The number of students in these
classrooms is 28, 20 and 22, respectively. All the students took examination and the average score of the
students of 11A, 11B and11C is 60, 70 and 70, respectively. What is the average score in this examination for
Solution:
a. Arranging in an increasing order gives 3, 5, 6, 6, 7, 8, 10. Since the number of observations is 7 and this
n 1 7 1
th th
number is odd, therefore, md item item 4 item which shows the median is 6.
th
2 2
b. First you have to arrange in increasing order giving, 49, 50, 59, 60, 63, 72. Since n = 6, which is even, you
th th th th
n n 6 6
item 1 item item 1 item
will use the second formula: md 2 2
2 2
2 2
3 item 4 item 59 60 119
th th
md 59.5
2 2 2
Exercise 10.4
1. Consider the following data which shows the amount of milk in liters sold by a farmer in one month.
a. Find the median from the raw data. b. Prepare a frequency distribution table.
2. The following data shows score of fifty students in Mathematics exam
NOTE: First use the correcting factor to prepare a cumulative frequency table.
Example 14: Find the median of the following distribution.
Class Interval Frequency ( )
1 4 8
58 12
54
Solution: The correcting factor is 0.5 (uniform for all 9 12 3
2
classes). 13 16 25
From this, you can prepare the class boundary column and the 17 20 13
cumulative frequency column as follows. 21 24 7
1 4 0.5 4.5 8 8
58 4.5 8.5 12 20
9 12 8.5 12.5 3 23
13 16 12.5 16.5 25 48
17 20 16.5 20.5 13 61
21 24 20.5 24.5 7 68
f 68
2
Therefore the median class is 12.5 16.5
n
Thus, BL 12.5, 34, f c 25, i 16.5 12.5 4, cfb 23 .
2
Therefore, n
cf
b
34 23
md 12.5 1.76 14.26
md BL 2 i 12.5 4
fc 25
The median is 14.26 .
Example 15: The following is the height of 30 students in a class. Find the median height.
146 145
Solution: The correcting factor is 0.5 (uniform for all classes).
2
From this, you can prepare the class boundary column and the cumulative frequency column as follows.
The median class is that class containing the 30 item 15th item . It is the second class.
th
2
Therefore the median class is 145.5 151.5.
n
Thus, BL 145.5, 15, f c 9, i 151.5 145.5 6, cfb 7 .
2
n
Therefore, cf b 15 7
md 145.5 5.333 150.83
md BL 2 i 145.5 6
fc 9
The median height is 150.83 .
2. The mark that students scored in an examination is grouped in class intervals as shown in the following
table. What is the median of the mark?
55 64 8
65 74 12
75 84 20
85 94 6
95 104 4
3. The amounts of drops of water in drip irrigation were registered from 80 sample drip holes in one day and the
data are as follows.
a. Find the mean and median score of the students. b. Compare the mean and the median.
The mode (mo ) In statistics, the word mode represents the most frequently occurring value in a data set.
The Mode of a set of data is the value in the data which appears most frequently in the
Definition 10.7: set of values.
NOTE: The mode can usually be determined by observation. It is possible for distribution to have more than
one mode or no mode at all.
Solution:
a. Every member appeared only once. Hence there is no mode for this distribution.
b. In this observation, the most frequent value is 2. Therefore, the mode is mo 2 since it appears three
times. This data has only one mode and is called unimodal.
xample 17: Given that 1, 2, x,5, y,8 . Find the values of x and y if the mode of the resulting numbers is 5 and the
mean is 4 .
1 2 x 5 y 8
Solution: The mean of the numbers is 4 4
6
x y 16 24
x y 8 …….(1)
Therefore, x 3 and y 5 .
Exercise 10.6
1. The following represent days in a month at which salary was paid for forty-two consecutive months.
a. Determine the mode shoe size in the shop? b. What does this mode describe?
3. Calculate the mean and the mode.
1. Identify the modal class. It is the class with the highest frequency.
d1
2. Determine the mode using the following formula: Mode mo BL i
d1 d 2
where, BL lower class boundary of the modal class.
d1 the difference between the frequency of the modal class and the frequency of the preceding class (pre-modal
class).
d2 the difference between the frequency of the modal class and the frequency of the subsequent class (next
class).
i the size of the class interval.
Example 18: The following table gives the age distribution in a certain class. Compute the modal age (in years).
Solution: The modal class is the 3rd class because its frequency is the largest.
BL 19.5, d1 10 6 4, d2 10 2 8, i 24 19 5
d1
mo BL i
d1 d 2
4
mo 19.5 5
48
20
mo 19.5 19.5 1.67 21.17 years
12
mo 21.17 years
2
Example 19: The following is a distribution of the size of farms (in 1000m ) in a woreda. Find the mode of the
distribution.
d1
mo BL i
d1 d 2
2 5
mo 44.5 10 mo 19.5 45.21
2 26 7
1. The daily profits (in Birr) of 100 shops are distributed in the following table. Find the modal value.
2. The amounts of drops of water in drip irrigation were registered from 80 sample drip holes in one day and
the data are as follows.
4 4
This means Q1 is half way between the 2 nd and 3rd items .
4 4
It is half way between the 7 x7 and 8th x8 items .
th
Solution: Arranging in increasing order of magnitude, we get, 5, 9, 21, 25, 29, 31, 38, 42, 46, 50.
th
in in
10 10 1
Di item
2
1
23
th
10 10 item D2
D2 item=2.5 item 15 and
th
2 2
7 10 7 10
th
1
item 7 8 item 7.5th item 40
th
D7 10 10
2 2
3. Percentiles
Percentiles are values that divide a data set into a hundred equal parts. There are ninety nine percentiles, namely,
P1 , P2 , P3 ,..., P99 . Percentiles are not the same as percentages. If a student gets 85 correct answers out of a possible
100, he obtains a percentage score of 85. Here there is no indication of his position with respect to other students.
th
On the other hand if a score of 85 corresponds with the 96 percentile, then this score is better than 96% of the
students under consideration. Were your average and percentile in your grade eight exams the same?
Steps to calculate percentiles for ungrouped data:
1. Arrange the data in increasing order of magnitude.
2. If the number of observations is:
t n 1
th
a. odd, Pt item.
100
th
tn tn
100 100 1
item
b. even, Pt .
2
Example 22: Find P42 and P75 for the following data. 25, 38, 42, 46, 50, 31, 29, 21, 9, 5.
Solution: Arranging in increasing order of magnitude, we get, 5, 9, 21, 25, 29, 31, 38, 42, 46.
Number of observation is odd.
t n 1
th
So, Pi item.
100
42 9 1
th
42 10
th
100 100
75 9 1
th
75 10
th
100 100
Note that P75 40 . That is, 75% of the data values are less than P75 and the rest are above it.
kn
cfb
Qk k th quartile BL 4 i , k 1, 2, 3 and
f k
BL lower class boundary of the k th quartile class
Range of marks 20 25 26 31 32 37 38 43 44 49
Frequency 15 30 25 20 10
4
kn
4 cfb
Qk BL i
fk
1100
15 25 15 6 25.5 10 6 25.5 2 27.5
Q1 25.5 4 6 25.5
30 30 30
Therefore, Q1 27.5
th
kn
Solution: n 40 and Qk is the item
4
kn
4 cfb
Qk BL i
fk
1 40
5 10 5 5 24.5 5 5 24.5 25 =28.07
Q1 24.5 4 5 24.5
7 7 7 7
2 40
th
4
2 40
12 20 12 5 29.5 8 5 29.5 5 34.5
Q2 29.5 4 5 29.5
8 8 8
3 40
th
4
3 40
20 30 20 5 34.5 10 5
Q3 34.5 4 5 34.5
18 18 18
Q3 34.5 2.78 37.28
2. Deciles: The j th deciles for grouped frequency distributions is calculated in a similar way as follows.
Steps to calculate deciles for grouped data:
th
jn
1. Find the class where the j th quartile belongs, which the class that contains the item .
10
jn
cfb
Use the formula D j j decile BL 10 i, j 1, 2,...,9
th
2.
f c
Where BL lower class boundary of the j th decile class. and n f
cfb the cumulative frequency before the j th decile class.
fc the number of observations (frequency) in the j th decile class.
Solution:
3 36
th
10
7 36
16
So, D 59.5 10 .
7 10 59.5 5.41 64.91
17
9 36
th
n f and cfb the cumulative frequency before the j th percentile class.
Solution:
20 36
th
50 36
th
68 36
th
Class interval 45 55 55 65 65 75 75 85 85 95
Frequency 26 38 33 16 7
th
Find the first quartile Q1 and the 75 percentile P75 of the measurement.
4
1120
26 30 26 10 54.5 1.05 55.55
Q1 54.5 4 10 54.5
38 38
Therefore, Q1 55.55
75 120
th
Exercise 10.7
1. Find Q1 , Q2 and Q3 for each of the following data sets:
a. 78, 68, 19, 35, 46, 58, 35, 35, 31, 10, 48, 28
b.
c.
b. From the above data, if students in the top 25% are to be awarded a certificate, what is the minimum mark for
a certificate?
c. If students whose scores are in the bottom 25% of the marks are considered as failures, then what is the
maximum failing mark?
d. Find the percentile of the farmers who received more than 20 quintals.
e. If a farmer receives more than 75 percentile, find the minimum amount of quintals of fertilizer s/he receives.
3. Find Q2 , Q3 , D4 , D8 , P12 , P24 , P87 for each of the following data sets:
a.
b.
Class Interval
Frequency
Range for grouped data is defined as the difference between upper class boundary of the
highest class Bu(H) and the lower class boundary of the lowest class Bl(L). That is,
Definition 10.9: R=Bu(H)- Bl(L).
Example 29: Consider the following data, what is the range of this distribution?
Solution: From the grouped frequency distribution, the range is calculated using;
R Bu H Bl L 22.5 4.5 18
Advantages and limitations of range
Advantage of Range
It is simple to compute
It is fixed value
Limitation of Range
It only depends on extreme values.
It doesn‟t consider variations of values in between.
It is highly affected by extreme values.
It does not tell us anything about the variability of other data.
2. Variance and standard deviation
The standard deviation is the most commonly used measure of dispersion. The value of the standard deviation tells
how closely the values of a data set are clustered around the mean. In general, a lower value of the standard
deviation for a data set indicates that the values of the data set are spread over a relatively small range around the
mean. On the other hand, a large value of the standard deviation for a data set indicates that the values of that data
set are spread over a relatively large range around the mean.
Variance is a measure of dispersion that takes into account the spread of all data points in a data set. It‟s the
measure of dispersion the most often used, along with the standard deviation, which is simply the square root of
the variance. The variance is mean squared difference between each data point and the centre of the distribution
measured by the mean.
Definition 10.10: Variance is the average of the squared deviation of each item from the mean.
x x
2
x1 x x2 x ... xn x
2 2 2 i
Variance ( s 2 ) i 1
n n
where, x mean , s variance and n number of values
2
The quantities x x is in the above formula are the deviations of x from the mean.
Steps to calculate variance for ungrouped data:
a. Calculate the mean of the distribution.
b. Find the deviation of each value from the mean and square it.
c. Add the squared deviations.
d. Divide the sum obtained in step c by n .
Definition 10.11: The positive square root of variance is called standard deviation.
Properties of Standard Deviatio: It describes the square root of the mean of the squares of all values in a
data set and is also called the root-mean-square deviation.
The smallest value of the standard deviation is 0 since it cannot be negative.
When the data values of a group are similar, then the standard deviation will be very low or close to zero.
But when the data values vary with each other, then the standard variation is high or far from zero.
n
x x
2
i
Standard deviation sd variance sd i 1
n
Example 30: Find the variance and standard deviation of the data 20, 16, 12, 8, 18, 5, 9, 24.
20 16 12 8 18 5 9 24
Solution: The mean x 14
8
x x
2
302
x x
2
x x x2 x ... xn x
2 2 2 i
302
Variance ( s 2 ) 1 i 1
s2 37.75
n n 8
Standard deviation sd variance
n
x x
2
i
sd i 1
37.75 6.14
n
Example 31: Find the variance and standard deviation of the data 6, 7, 10, 11, 11, 13, 16, 18, 25.
6 7 10 11 11 13 16 18 25
Solution: The mean x 13
9
n
x x x x ... x x
xx x x x x
2 2
x
2 2 2 i
6 7 49 Variance ( s ) 2 1
2 n i 1
n n
7 6 36 280
10 3 9 s2
31.11
9
11 2 4
Standard deviation sd variance
11 2 4
13 0 0 n
x x
2
i
16 3 9
sd i 1
31.11 5.58
18 5 25 n
25 12 144 If x1 , x2 , x3 ,..., xn are values with corresponding frequencies
x x 280
2
f1 , f 2 , f3 ,..., f n , the variance is given by
xi x
2
fi
f1 x1 x f 2 x2 x f3 x3 x ... f n xn x
2 2 2 2
Variance ( s 2 ) i 1
n
n
fi
i 1
Example 32: Find the variance and standard deviation of the following data.
f x x
2
i i
146.63
Variance ( s 2 ) i 1
n s2 6.11 and sd variance 6.11 2.47
f i 1
i
24
f i 1
i
mark).
Steps to calculate variance from a grouped frequency distribution:
a. Find the class mark for each class.
b. Find the mean of the grouped data.
c. Find the deviation of each class mark from the mean and square it.
d. Find the sum of the squared deviations, then Divide the sum obtained in step d by f i .
Example 33: The following set of raw data shows the lengths, in millimeters, measured to the nearest mm, of 40
leaves taken from plants of a certain species. This is the table of frequency distribution.
Length (mm) Frequency fi
25 29 2
30 34 4
35 39
n
7
Solution: The mean x
fx i ci
1720
40 44 10 i 1
n
43
f
40
45 49 8 i 1
i
50 54 6
55 59 3
fi xi
xi 43 xi 432 f i xi 43
2
f 40 f x 1720
i i f x x
i i
2
2460
f x x
2
2460
Variance ( s 2
) i i
61.5
f i 40
f x x
2
2460
Standard deviation sd 61.5 7.84
i i
f i 40
Example 34: Find the variance and standard deviation of the following distribution.
f x x
2
250
Variance ( s 2
) i i
16.67
f i 15
f x x
2
250
Standard deviation sd 16.67 4.08
i i
f i 15
b.
c.
10.2. PROBABILITY
1. An Experiment is an activity (measurement or observation) that generates results (outcomes).
2. An Outcome (Sample point) is any result obtained in an experiment.
3. A Sample Space (S) is a set that contains all possible outcomes of an experiment.
4. An Event is any subset of a sample space.
Example 1: When a "fair" coin is tossed, the possible results are either head (H) or tail (T). Consider an
experiment of tossing a fair coin twice.
a. What are the possible outcomes? c. Give the event of H appearing on the second throw.
b. Give the sample space. d. Give the event of at least one T appearing.
Solution:
NOTE: In tossing a coin, if the coin is fair, the two possible outcomes have an equal chance of occurring. In
this case, we say that the outcomes are equally likely.
probability of the event A HH,TH , you have to count the number of outcomes in event A (which is 2) and
n A 2 1
P A
nS 4 2
Now, if the experiment is tossing a coin five times, what is the total number of possible outcomes? If an event E is
defined by "3 heads and 2 tails", then how do you find n E ? From this, you can observe that counting plays a
very important role in finding probabilities of events. In this section, you shall see some mathematical techniques
which will help you to simplify counting problems. When the number of possible outcomes is very large, it will be
difficult to find the number of possible outcomes by listing. So you have to investigate different counting
techniques which will help you to find the number of elements in an event and a possibility set.
Fundamental principles of counting
There are two fundamental principles that are helpful for counting. These are the multiplication principle and the
addition principle.
Multiplication principle
Before we state the principle, let us consider the following example.
Example 3: Suppose Nuria wants to go from Harrar via Dire Dawa to Addis Ababa. There are two minibuses from
Harrar to Dire Dawa and 3 buses from Dire Dawa to Addis Ababa. How many ways are there for Nuria to travel
from Harrar to Addis Ababa?
Example 7: A corporation has a board of directors cosisting of 10 members. The board must select from among its
members a chairperson, vice chairperson, and secretary. In how many ways can this be done?
Solution:
1. Select chairperson- there are 10 ways (people) to do this.
2. Select a vice chairperson- there are 9 ways ( 9 people left after chairperson is chosen) to do this.
3. Select a secretary- there are 8 ways to do this.
Therefore, the total nmber of ways to complete the task is 10 9 8 720 ways.
Addition principle
NOTE: Two events are said to be mutually exclusive, if both cannot occur simultaneously.
In tossing a coin, Head and Tail are mutually exclusive events because they cannot appear at the same time.
Example 5: A question paper has two parts where one part contains 4 questions and the other 3 questions. If a
student has to choose only one question, from either part, in how many ways can the student do it?
Solution: The student can choose one question in 4 3 7 ways.
The fundamental counting principles can be extended to any number of sequences of events.
Example 6: A question paper has three parts: language, arithmetic and aptitude tests. The language part has 3
questions, the arithmetic part has 6 questions and the aptitude part has 5 questions. If a student is expected to
answer one question from each of two of the three parts, with arithmetic being compulsory, in how many ways can
the student take the examination?
Solution: The student can either take language and arithmetic or arithmetic and aptitude. This gives
3 6 5 6 48 possibilities.
Example 7: The HIV/AIDS Club of a certain school has 6 members: A, B, C, D, E, and F. They are going to line
up, from left to right, for a group photo. After lining up in alphabetical order (ABCDEF), Mr. F complains that he
is always last whenever they do things alphabetically, so they agree to line up in reverse order (FEDCBA) and take
another picture. Then Ms. D complains that she's always stuck next to Mr. C, and that she never gets to be first in
line. Finally, in order to avoid bruised egos, they all agree to take pictures for every possible left-to-right line-up of
the six people. How many different photos must be taken?
Solution: To arrange the six people in a line we need to make six dependent decisions:
1. Choose first person (6 options);
2. Choose second person (5 options)
3. Choose third person (4 options)
4. Choose fourth person (3 options)
5. Choose fifth person (2 options)
6. Choose last person (1 option)
According to the Fundamental Counting Principle the number of outcomes is 6 5 4 3 2 1 720 .
Exercise 10.9
1. In an experiment of selecting a number from 1 – 10, which of the following cannot be an event?
a. The number is “even and prime”. c. The number is multiple of 3.
b. The number is “even and multiple of 5”. d. The number is zero.
2. In an experiment of tossing three coins at a time,
a. Determine the sample space. b. Find the probability of getting two heads.
3. A box contains 2 Red and 3 Black balls. If two balls are drawn at random,
a. Determine the possible outcomes
b. Find the probability of getting 2 Red balls.
c. Find the probability of getting 1 Red and 1 Black balls.
10!
Example 8: Calculate a. 4! b. 6! c.
6!
Solution:
a. 4! 4 3 2 1 24 10! 10 9 8 7 6!
c. 10 9 8 7 5040
b. 6! 6 5 4 3 2 1 720 6! 6!
Permutation
The number of permutations of a set of n objects taken all together is denoted by P n, n or n Pn and is equal to
n! .
Thus, P n, n n !
Example 9:
a. Give all the permutations of three letters A, B and C.
b. Suppose we have 5 people to be seated in only 3 seats. In how many ways can they sit?
Solution:
Example 10: Find the number of permutations that can be made out of the letters of the word
"MATHEMATICS". In how many of these permutations
a. do the words start with C?
b. do all the vowels occur together?
c. do the words begin with H and end with S?
Solution:
a. In word "MATHEMATICS" there are 2 M‟ s, 2 A‟s and 2 T‟s
10!
So, there are ways that words start with C.
2!2!2!
4!7!
b. There are ways that all vowels occur together.
2!2!2!
9!
c. There are ways that words begin with H and end with S.
2!2!2!
Exercise 10.10
1. How many four – digit numbers can be formed from the digits 1, 3, 5, 7, 8 and 9 where a digit is used at
most once?
a. if the numbers must be even? b. if the numbers are less than 3000?
2. Two men and a woman are lined up to have their picture taken. If they are arranged at random, find the
number of ways that
a. the woman will be on the left in the picture.
b. the woman will be in the middle of the picture.
3. In a library there are 3 Mathematics, 4 Geography and 3 Economics books. If each of them will be put on
a shelf and each type of a book are identical, in how many ways can these books be arranged?
4. Verify that n Pn1 n Pn .
If n objects are to be arranged on a circle (along the circumference of a circle), then the number of circular
Example 11:
a. 7 people are to sit around a circular table. In how many different ways can these people be seated?
b. In how many ways can 6 boys and 6 girls sit around a table of 12 seats, if no two girls are to sit
together?
c. In a playground, 3 sisters and 8 other girls are playing together. In a particular game, how many
ways can all the girls be seated in a circular order so that the three sisters are not seated together?
Solution:
a. Consider the following.
The number of ways these 7 people sit around a round table is 7 1! 6! 720 ways.
and .
To arrive at a formula for n Cr , observe that the r objects in n Pr can be arranged among themselves in r!
ways.
n!
Pr n r ! n!
Hence, C n, r n
.
r! r! n r !r !
Therefore, the number of possible combinations of n objects taken r at a time is given by the formula
n n!
C n, r , 0r n.
r n r !r !
From this, you can see that the number of ways that a committee of two members can be selected from three
3! 3!
individuals is given by C 3, 2 3 ways.
3 2 !2! 1!2!
Example 12: Compute the following. a. C 8, 6 b. C 10, 4
Solution:
8! 8! 8 7 6! 8 7
a. C 8, 6 28 .
8 6 !6! 2!6! 2!6! 2
a.
n
b. n n n 1
n
Example 13: Show each of the following.
r nr
r r 1 r
Solution:
n n! n n! n!
n r n n r ! n r !
a.
n r n n r ! n r ! r ! n r !
n! n
C n, r
n r !r ! r
n n n! n!
b.
r r 1 n r !r ! n r 1! r 1!
n n n! n!
r r 1 n r !r ! n 1 r ! r 1!
n! rn !
n!
rn !
n r !r ! n 1 r !r ! n r !r ! n 1 r n r !r r 1!
n! n!
n r !r ! n 1 r n r ! r 1!
n ! n 1 r rn !
n 1 r !r !
n !n n ! rn ! rn !
n 1 r !r !
n !n n ! n ! n 1 n 1! n 1
n 1 r !r ! n 1 r !r ! n 1 r !r ! r
Example 14:
a. In an examination paper, there are 12 questions. In how many different ways can a student choose
eight questions in all, if two questions are compulsory?
b. In how many different ways can three men and three women be selected from six men and eight
women?
c. In how many ways can Bekele invite at least one of his friends out of 5 friends to an art exhibition?
d. A committee of 7 students has to be formed from 9 boys and 4 girls. In how many ways can this be
done when the committee contains
i. exactly three girls? ii. at least three girls? iii. 2 girls and 5 boys?
Solution:
6 8
6 men and 8 women is given by 20 56 1120 ways (by the Multiplication principle).
3 3
c. At least one means that he can invite either one, two, three, four or five. Therefore, the total number
of ways in which he can invite at least one of his friends is given by (Addition principle)
ii. At least 3 girls are included means the committee will consist of either 3 girls and 4 boys or 4 girls
and 3 boys.
∴ Total number of ways of forming a committee is given by
C 4, 3 C 9, 4 C 4, 4 C 9, 3 4 126 1 84 588 ways.
iii. Two girls and 5 boys can be selected in C 4, 2 C 9, 5 6 126 756 ways.
Exercise 10.11
2. Show that C n, 0 1 .
3. Show that C 5, 2 C 5, 3 .
4. If C n, 6 C n, 4 , then find n .
5. In how many ways can a committee of 5 be selected from 10 people willing to serve?
6. A committee of 5 students has to be formed from 9 boys and 9 girls. In how many ways can this be
done when the committee consists of
10. Let A be the set of 4-digit numbers a1a2 a3 a4 where a1 a2 a3 a4 , then find n A .
11. Suppose that 20 pillars of the same height have been erected along the boundary of a circular
stadium. If the top of each pillar has been connected by beams with the top of all its non-adjacent pillars, then
find the total number of beams.
10.2.2.Binomial Theorem
Binomial theorem
x y C n, 0 x n C n, 1 x n 1 y C n, 2 x n 2 y 2 C n, 3 x n 3 y 3 ... C n, n y n
n
n n n n n
x n x n 1 y x n 2 y 2 x n 3 y 3 ... y n
0 1 2 3 n
x y C 4, 0 x 4 C 4, 1 x 41 y C 4, 2 x 4 2 y 2 C 4, 3 x 4 3 y 3 C 4, 4 y 4
4
a.
ax b C 3, 0 ax C 3, 1 ax b C 3, 2 ax b 2 C 3, 3 b 3
3 3 2
b.
ax 3 ax b 3 ax b 2 b 3 a3 x3 3a2bx2 3ab2 x b3
3 2
0 1 2 3
5 5
2 x 3 3
4 5
4 5
4 4 4 3 4 2 4 4
x 2y x x 2 y x 2 y x 2 y 2 y
4 2 3 4
e.
0 1 2 3 4
x 4 4 x3 2 y 6 x 2 2 y 4 x 2 y 1 2 y
2 3 4
x4 8x3 y 24 x2 y 2 32xy3 16 y 4
Example 16: Find the coefficient of
x 2 y3 in expansion of x y . c. a b in expansion of 2a 5b .
5 6 5 11
a.
x 7 in expansion of x 1 .
39
b.
Solution:
5 5 5 5 5 5
x y x5 x 4 y x3 y 2 x 2 y 3 xy 4 y 5
5
a.
0 1 2 3 4 5
5 5! 5 4
Thus, the coefficient of x 2 y 3 is 10 .
3 3!2! 2
39 39!
15,380,937 .
7
b. The coefficient of x is
7 32!7!
92, 400,000a6 b5
Exercise 10.12
1. Expand each of the following using the Binomial Theorem:
6
1
3x 4 y
5
c. a
6
a. e. x 2 2
a
7 9
a b y
2 x 3 d. f. 3x
4
b.
b a 3
2. Without writing all the expanded terms, answer the following
NOTE: Outcomes of a random experiment are said to be equally likely when there is no reason to
expect any one of the outcomes in preference to another. That is, each element has equal chance of
being chosen.
Example 18: If a fair die is thrown, any one of the outcomes 1, 2, 3, 4, 5, 6 has an equal chance of appearing
at the top. Therefore, they are considered as equally likely.
NOTE: In a random experiment, the outcomes which insure the happening of a particular result are
said to be favourable outcomes to that particular result.
Example 19:
a. A fair die is thrown. How many favourable outcomes are there for getting an even number?
b. In picking a playing card from a pack of 52 cards, what is the number of favourable outcomes to
getting a picture card?
Solution:
a. There are 3 favourable outcomes. These are 2, 4 and 6.
b. There are 12 favourable outcomes - 4 Jacks, 4 Queens and 4 Kings.
10.2.4.Events
Recall that any subset of a sample space is called an event and is usually denoted by E . An event is a
collection of sample points.
Types of events
a. Simple Event (Elementary Event) is an event containing exactly one sample point.
Example 21: In a toss of one coin, the occurrence of tail is a simple event.
b. Compound Event When two or more events occur simultaneously, their joint occurrence is known
as a compound event, an event that has more than one sample point.
Example 22: When a die is rolled, if you are interested in the event "getting even number", then the event
will be a compound event, i.e. { 2, 4, 6}.
We can determine the possible number of events that can be associated with an experiment whose sample
space is S . As events are subsets of a sample space, and any set with m elements has 2m subsets, the number
of events associated with a sample space with m elements is 2m . (Sometimes this is called the exhaustive
number of events).
Example 23: Suppose our experiment is tossing a fair coin. The sample space for this experiment is S = {H,
T}. Thus, this sample space has a total of four possible events that are subsets of S. The list of the possible
events is { }, {H}, {T}, and {H, T}.
Occurrence or Non-occurrence of an event
During a certain experiment, there are two possibilities associated with an event, namely, occurrence or non-
occurrence of the event.
Example 24: If a die is thrown, then S = {1, 2, 3, 4, 5, 6}. Let E be the event of getting odd number, then
E 1, 3, 5 . When we throw the die, if the outcome is 3, as 3 E , then we say that E has occurred. If in
another trial, the outcome is 4, then as 4 E , we say that E has not occurred (not E ).
c. Complement of an Event E , denoted by E ' (not E ) consists of all events in the sample space that
are not in E .
Example 25: Let a die be rolled once. Let E be the event of a prime number appearing at the top i.e.
Algebra of events
NOTE:
Since events are sets (subsets of the sample space) one can form union, intersection and complement of them.
The operations obey algebra of sets, like commutativity, distributivity, De Morgan‟s laws and so on.
d. Exhaustive Events are events where at least one of them must necessarily occur every time the
experiment is performed.
Example 26: If a die is thrown give instances of exhaustive events.
Solution: The sample space is S = {1, 2, 3, 4, 5, 6}. From this, the events {1}, {2}, {3}, {4}, {5}, {6} are
exhaustive events. The events {1, 2}, {3, 4}, {4, 5, 6} are also exhaustive events for this experiment.
More generally, events E1 , E2 ,..., En form a set of exhaustive events of a sample space S where
e. Mutually Exclusive Events are events that cannot happen at the same time.
Example 27: Say whether or not the following are mutually exclusive events.
i. When a coin is tossed once, the events {H} and {T}.
ii. When a die is rolled, E1 getting an even number E2 getting a prime number
Solution:
i. Either we get head or tail but we cannot get both at the same time. Thus, {H} and {T} are mutually
exclusive events.
ii. E1 and E2 are not mutually exclusive because 2 is even and prime at the same time.
i. Ei E j for i j and,
ii. E1 E2 .... En S , then the collection of the events E1 , E2 ,..., En forms a mutually exclusive
and exhaustive set of events.
Example 28: If a die is thrown, the events {1}, {2}, {3}, {4}, {5}, {6} are mutually exclusive and exhaustive
events. But, the events {1, 2}, {3, 4}, {4, 5, 6} are not because {3, 4}∩{4, 5, 6} ≠ ∅.
g. Independent Events: Two events are said to be independent, if the occurrence or non- occurrence of
one event does not affect the occurrence or non-occurrence of the other.
Example 29: In a simultaneous throw of two coins, the event of getting a tail on the first coin and the event
of getting a tail on the second coin are independent.
Example 30: If a card is drawn from a well shuffled pack of cards and is replaced before drawing a second
card, then the result from drawing the second card is independent of the result of the first drawn card.
h. Dependent Events Two events are said to be dependent, if the occurrence or non-occurrence of one
event affects the occurrence or non-occurrence of the other.
Example 31: If a card is drawn from a well shuffled pack of cards and the card is not replaced, then the result
of drawing a second card is dependent on the first draw.
10.2.5.Probability of an Event
1. 0 PE 1
2. P S 1 , S is the sample space (the sure event)
3. If E1 and E2 are mutually exclusive events, then P E1 E2 P E1 P E2
NOTE:
P is a function with domain the set of subsets of S (Sample space) and its range is the set of real numbers
between 0 and 1 (both inclusive). Thus we note the following:
a. The probability of an event is always between 0 and 1.
b. If (the impossible event), then , and if E = S (the certain event), then P(S) = 1.
Example 34: A box contains 6 red balls. One ball is drawn at random. Find the probability of getting a.
a red ball b. a white ball
Solution:
a. The box contains all red balls. Hence we are sure that red will occur. Then, the probability of getting
n R 6
a red ball is one. Thus, P R 1
nS 6
Example 36:Which of the following cannot be valid assignments of probabilities for outcomes of sample
Solution:
a. is not valid assignment because the sum of the probabilities is not 1.
b. is valid; all the properties in the axiom above are satisfied.
c. is not valid because probability cannot be negative.
If m and n are probabilities of the occurrence and non-occurrence of an event respectively, then the ratio
m : n is called the odds in favour of the event. The ratio n : m is called the odds against the event.
Example 37: The odds against a certain event are 5 : 7 . Find the probability of its occurrence. Solution: Let
E be the event. Then, we are given that number not E 5 and number E 7 .
n E n not E n E 5 7 12 .
7
PE
12
In the last section, you have seen different types of events and approaches to probability. We will now discuss
some essential rules for probability and probabilities of the different types of events.
Example 38: Find the probability of obtaining a 6 or 4 in one roll of a die.
Solution: In one roll of a die, the sample space is S = {1, 2, 3, 4, 5, 6}. Obtaining 6 or 4 gives the event E = {4,
6}.
number of outcomes favouring E 2 1
Thus, P 4 or 6 P E .
number of possible outcomes 6 3
Trying to calculate probabilities by listing all outcomes and favourable outcomes may not always be
convenient. For more complex situations, there are rules we can use to help us calculate probabilities.
so that P E1 E2 P E1 P E2 .
Example 39:
a. Find the probability of obtaining a 6 or 4 in one roll of a die.
b. Find the probability of getting Head or Tail in tossing a coin once.
c. A die is rolled once. Find the probability that it is even or it is divisible by 3.
Solution:
a. Let E1 be event of getting 6, E2 be event of getting 4. Then E1 and E2 are mutually exclusive
1 1 2 1
events P E1 E2 P E1 P E2 .
6 6 6 3
This shows the addition rule of probability with two events. What do you think the rule will be for three or
more events? The rule can be extended for a finite number of events, but becomes increasingly complicated.
For example, for three events it becomes:
NOTE:
by P E2 | E1 and is called the conditional probability of E2 given that E1 has already occurred. If the
occurrence or non-occurrence of E1 does not affect the probability of E2 , or if E1 and E2 are independent,
Multiplication rule of probability: If E1 and E2 are any two events, the probability that both events occur,
P E1 E2 P E1 P E2 \ E1 , whenever P E1 0 .
P E2 P E1 \ E2 , whenever P E2 0 .
NOTE: If and are independent, then .
Hence, for independent events and .
Example 41:
a. A box contains 3 red and 2 black balls. One ball is drawn at random, is not replaced, and a second
ball is drawn. Find the probability that the first ball is red and the second is black.
b. A die is rolled and a coin is tossed. Find the probability of getting 3 on the die and a tail in the coin.
c. A bag contains 3 red, 4 blue and 3 white balls. Three balls are drawn one after the other. Find the
probability of getting a red ball on the first draw, a blue ball on the second draw and a white ball on the third
draw if
i. each ball is drawn, but then is replaced back before the next draw.
ii. the balls are drawn without replacement.
Solution:
a. Let E1 getting red in the first draw. And E2 getting black in the second draw.
3 2 6 3
P E1 E2 P E1 P E2 | E1 .
5 4 20 10
b. Let E1 getting 3 on the die and E2 getting tail on the coin.
1 1 1
Since the two events are independent, P E1 E2 P E1 P E2
6 2 12
c. Let E1 getting red, in the first draw,
E2 getting blue in the second draw, and E3 getting white in the third draw.
i. The balls are replaced after each draw. The events are independent.
3 4 3 36 9
P E1 E2 E3 P E1 P E2 P E3 .
10 10 10 1000 250
ii. The balls are not replaced, so events are dependent .
3 4 3 1
P E1 E2 E3 P E1 P E2 | E1 P E3 | E1 and E2 .
10 9 8 20
1
Therefore, the probability that both outcomes are heads is .
4
Example 43: Suppose that a group of 10 students contain eight boys (B) and two girls (G). If two students
are chosen randomly without replacement, find the probability that the two students chosen are both boys.
Solution: P B1 and B2 P B1 P B2 | B1 108 79 56
90 45
28
.
28
Hence, the probability that the two students chosen are both boys is .
45
Example 44: A bag contains 5 red balls, 4 blue balls, and 3 white balls. Two balls are drawn one after the
other, without replacement.
a. Find the probability that both are red.
b. Draw tree diagram representing the experiment.
5 4 20 5
Solution: P R and R .
12 11 132 33
x i
Mean x i 1
for raw data
n
n
fx i i
i 1
n
for discrete data
f
i 1
i
fm i i
i 1
n
for grouped data ( m class mark)
f
i 1
i
x1 x x2 x x3 x .... xn x 0 .
8. If a constant c is added to each value of a population function, then the new mean x ' is equal to the old
mean plus c .
x
9. Median of ungrouped data is given by
n 1th
Md item, if n is odd
2 After data is arranged in
n
th
n
th
increasing or decreasing
item 1 item order of magnitude.
2
2
, if n is even
2
x x
2
i
Variance i 1
n
n
x x
2
i
Standard deviation (S) is the positive square root of variance, S Variance i 1
.
n
18. Probability of an event E is defined as follows If an experiment results in n equally likely outcomes and
m
m n is the number of the ways favourable for event E , then P E .
n
19. Multiplication Principle
If an event can occur in m different ways and for every such choice another event can occur in n different ways,
then both events can occur in the given order in m n different ways.
20. Addition Principle
If an operation can be performed in m different ways and another operation can occur in n different ways and
the two operations are mutually exclusive, (the performance of one excludes the other) then either of the two can
be performed in m n ways.
21. f n is a natural number, then n factorial, denoted by n! , is defined by
n 1!
1. If 5 , then find n .
n!
2. How many three – digit numbers can be formed from the digits 2, 5, 7, 9
a. if each digit is used once only? b. if each may be used repeatedly?
3. A box contains 12 bulbs with 3 defective ones. If two bulbs are drawn from the box together, what is the
probability that
a. both bulbs are defective? c. one bulb is defective?
b. both are non-defective?
7 3 4 6
a. the coefficient of a b b. the coefficient of a b .
5. A committee of 5 members is to be selected from 7 men and 8 women. In how many ways can this be done so
as to include
a. 2 women? b. at least 2 men? c. at most 4 women?
6. A box contains 3 red and 8 white balls. If one ball is drawn from it, find the chance that the ball drawn is red.
7. From a pack of 52 playing cards, three cards are drawn one after the other without replacement. What is the
probability that Ace, King and Jack will be obtained respectively?
8. Suppose a pair of dice is thrown. What is the probability that the sum of the scores is 5?
9. The marks for passing in the examination in a subject is 33.4 out of 9 students who appeared at the examination
have failed and the marks of remaining students are 78, 40,97, 65, 50 . Find the median of the marks.
1. A school has three classrooms for grade 11, namely, 11A, 11B and 11C . The number of students in these
classrooms is 28, 20 and 22, respectively. All the students took examination and the average score of the
students of 11A, 11B and11C is 60, 70 and 70, respectively. What is the average score in this examination
A. 3 B. 3 C. 2.3 D. 3.6
4. Which of the following is true about the Arithmetic Mean of a given data?
a. It can be obtained even in the absence of some of the values in the data.
b. It can also be used for qualitative data.
c. There can be two means for a given data.
d. It is affected by extreme values.
5. A three-digited library identification card is to be printed from the numbers 0, ,1, 2, 3, 4, 5 in such a way that
the first is non-zero and no number is to be repeated. How many such card can be printed?
A. 100 B. 120 C. 150 D. 180
6. A student needs to be select 3 books from 3 different mathematics, 3 different physics and 1 history book. What
is the probability that one of them is mathematics and the other two is either physics or history books?
3 9 15 18
A. B. C. D.
35 35 35 35
7. Items produced by a certain company are subjected to two kinds of defects D1 and D2 . Out of the total
production, if 5% have defect D1 , 10% have defect D2 and 2% have both defects, then what is the
10. The following table shows the frequency distribution of a group data in which the frequency of the class
interval, f1 , is missed.
Class interval 37 8 12 13 17 18 22
Frequency f1 9 4 1
If the mean of the data is 10 , what is the frequency f1 of the first class interval?
A. 4 B. 6 C. 7 D. 8
11. Below are the marks of grade 12 students in Mathematics out of 100. What is the variance of the frequency
distribution given below?
Marks 20 40 40 60 60 80 80 100
Number of students 4 10 4 2
A. 107.2 B. 4 C. 304 D. 17.5
12. Let A: getting head in one toss of a coin and B: getting number 5 in one roll of a die. Which one of the
following is true?
A. P A B 16 B. PA B 101 C. P A B
7
12
D. P A PB
1
10
13. Three men, M 1 , M 2 and M 3 are firing at a target independently and have probability 0.20, 0.25 and
0.30 respectively, of hitting the target. What is the probability that at least one of them hits the target? A.
0.30 B. 0.42 C. 0.58 D. 0.75
If Qi , Di and Pi are respectively the i quartile, deciles and percentile of a data arranged in an
th
20.
26. Three persons P1 , P2 and P3 are firing at a target independently and have a probability 0.7, 0.5 and 0.4 ,
respectively, of hitting the target. What is the probability that at least one of them hits the target?
A. 0.95 B. 0.85 C. 0.91 D. 0.99
27. The following is a simple frequency distribution table of a data with variable X.
X 3 5 6 7
Frequency 2 5 2 1
product, 5% have defect D1 , 10% have defect D2 , and 2% have both defects. What is the probability that a
C. PE1 E 2 E3
1
A. The number of elements in E1 E 2 is zero.
6
B. PE1 E 2 D. PE1 E 2 E3
1 5
2 6
Main Contents:
11.1. MATRICES
11.2. DETERMINANTS AND THEIR PROPERTIES
11.3. INVERSE OF A SQUARE MATRIX
11.4. SYSTEMS OF EQUATIONS WITH TWO OR THREE VARIABLES
11.5. CRAMER’S RULE
Key terms
Summary
Review Exercises
11.1. MATRICES
DEFINITION 11.1 Let R be the set of real numbers and m and n be positive integers.
A rectangular array of numbers in of the form,
Number of coins
10 cent coins 2 6 4
50 cent coins 3 2 0
25 cent coins 4 0 5
Solution
a. 2 6 4
A 3 2 0
4 5
0
b. The columns represent the number of the various kinds of coins each student has.
c. The rows represent the number of coins of a certain fixed value that the students have.
d. a31 = 4. It means Chaltu has four 25-cent coins in her pocket. a23 = 0. This means Solomon has no 50-cent
coins.
Let A (aij )32 where aij 2 j i . Determine A.
2
Example 3.
Solution. A (aij )32 shows that i 1, 2 and j 1, 2 and 3 .
NOTATION
For a matrix A (ai j )m n , the ith row of A is denoted by Ai and the j th column of A is denoted by A . That is if
j
The PRINCIPAL OR MAIN DIAGONAL of a square matrix A = (aij)nn of order n is the list
( ) ( )
of its n diagonal elements.
The other elements with i j are called the OFF-DIAGONAL ELEMENTS of A.
For instance;
2 10
i) A is a square matrix of order 2.
5 8
The list of the main diagonal elements are and
1 5 3
0 3
ii) B 0 1 is a square matrix of order 4.
2
1 1 2 0 0
7 0 1
1
The list of the main diagonal elements are ,
4. A matrix with all entries 0 is called a ZERO or NULL MATRIX which is denoted by 0.
For instance;
i) C 0 0 0 is a zero matrix of order 1 by 3.
ai j 0 for all i j.
For instance;
0 0 0 0 0 0 0 0 0
0 0
A , B 2 0 0 , C 0 0 0 and D 3 0 0
5 0 3 0 0 0 0 0 2 1 0
8. An UPPER TRIANGULAR MATRIX is a square matrix whose elements below the main diagonal are all zero.
That is a square matrix A ai j
nn
is an upper triangular matrix if
ai j 0 for i j.
For instance;
1
2 0 0 3 0 1
2
0 5
M , S 0 2 0 , and T 0 1 0
0 0 0 0 2 0 0 9 4
0 0 2
0
are upper triangular matrices.
NOTE: A square matrix A ai j nn
is said to be A STRICTLY UPPER TRIANGULAR MATRIX if
ai j 0 for all i j.
0 1 3
0 1 4 0 0 0
0 2 0 0 0 5
For instance; A , B 0 0 8 , C 0 0 0 and
0 0 0 0 0 0 0 0 0 0 0 3
0 0 0 0
Note that all diagonal matrices are triangular matrices.
Equality of matrices
DEFINITION 11.2: Two matrices A (ai j ) m n and B (bi j ) m n of the same order are said to be
EQUAL, written as A = B, if their corresponding elements are equal, i.e. ai j bi j for all 1 i m and 1 j n
.
Suppose Fatima wants to know the total production of sport shoes in each price category. Then the total production
In category 1 : for boys (80 + 90) , for girls (60 + 50)
In category 2 : for boys (75 + 70) , for girls (65 + 55)
In category 3 : for boys (90 + 75) , for girls (85 + 75)
This can be represented in the matrix form as
This new matrix is the sum of the above two matrices. Thus the sum of two matrices is defined as follow:
DEFINITION 11.3
Let A (ai j ) m n and B (bi j ) m n be two matrices. Then the sum of A and B, denoted by A + B, is an m n matrix
obtained by adding the corresponding elements, while the difference of A and B, denoted by A – B, is an m n
matrix obtained by subtracting the corresponding elements i.e.,
i) A B (ai j )m n (bi j )m n (ai j bi j )m n , and
4 5 2 1
Example 6. If A , then find a) 5A b) A c) 0.A d) 1.A
0 2 1 2
4 5 2 5 (4) 5 5 5 2 20 25 5 2
Solution a) 5 A 5
0 2 1 5 0 5 (2) 5 (1) 0 10 5
1 1 1 5 2
(4) (5) ( 2) 2
1 1 4 5 2 2 2 2
b) A 2 2
2 0 2 1 1 1
(1) 0
2 1 1
(0) (2) 1
2 2 2 2
4 5 2 0(4) 0(5) 0( 2) 0 0 0
c) 0. A 0
0 2 1 0(0) 0( 2) 0( 1) 0 0 0
4 5 2 1(4) 1(5) 1( 2) 4 5 2
d) 1. A 1
0 2 1 1(0) 1(2) 1(1) 0 2 1
Properties of Scalar Multiplication of Matrix
If A and B are matrices of the same order and r and s are any scalars (i.e., real numbers), then:
a. r(A + B ) = rA + rB c. (rs)A = r(sA) e. 0A = 0e.
b. (r + s) A = rA + sA d. 1A = A and 0A = 0 f. 1A = A
1 1 3 3 2 1
7. Let A 0 2 5 and B 1 5 2 , then find
2 0 3 2 0 2
a) B c) 3B 2 A e) A 3B
b) 2 A B d) a matrix C such that B 2C A
1 2 5 3 2 7 2 3 0
8. Let A , B and C . Then find
0 3 3 1 2 1 1 1 4
a) C 2 A b) B (C A) c) (3B A) 2C
9. Graduating students from a certain high school sold cinema tickets on two different occasions, in two kebeles,
in order to raise money that they wanted to donate to their school. The following matrices show the number of
students who attended the occasions.
st nd
1 occasion 2 occasion
Kebele 1 Kebele 2 Kebele 1 Kebele 2
Boys 175 221 Boys 120 150
Girls
190 150 Girls 199 181
a. Give the sum of the matrices.
b. If the tickets were sold for Birr 2.50 a piece on the 1st occasion and Birr 3.00 a piece on
the second occasion, how much money was raised from the boys? from the girls? In kebele 1. What is the total
amount raised for the school?
Multiplication of Matrices
To study the rule for multiplication of matrices, first let us define the rule for row and column matrices of orders 1
× p and p × 1, respectively.
Let A and B be a row and column matrices of orders 1 × p and p × 1, respectively, given by
b11
A a11 a12 ... a1 p and B b21 . Then the product AB in the given order is the 1 × 1 matrix given by
...
a p1
b11
b21
AB a11 a1 p
... 11 11
a12 ... a b a12b21 ... a1 p b p1
bp1
0
3
Example 7. If A a14 4 0 2 1 and B (b41 ) , then find AB.
5
7
matrix B bp 1 ,
1. The number of columns of A = the number of rows of B = p
2. The operation is done row by column in such a way that each element of the row is multiplied by the
corresponding element of the column and then the products are added.
2 3
Now, let us consider two matrices A 1 0
and B 1 1 0 4 . Then
0 5 2 5 3 12 4
3 2
1 1
A1 B1 2 3 2(1) (3)(2) (8) , A1 B 2 2 3 2(1) (3)(5) (17)
2 5
0 4
A1 B 3 2 3 2(0) (3)(5) (15) , A1 B 4 2 3 2(4) (3)(1) (11)
3 1
1 1
A2 B1 1 0 1(1) (0)(2) (1) , A2 B 2 1 0 1(1) (0)(5) (1)
2 5
0 4
A2 B 3 1 0 1(0) (0)(3) (0) , A2 B 4 1 0 1(4) (0)(1) (4)
3 1
Then the matrix determined by these products
c11 c12 c13 c14 A1 B A1B A1B A1B 8 17 15 11
1 2 3 4
C (cij ) 24
4
21 22 23 24 2 1 1 0 4
1 2 3
c c c c A B A2 B A2 B A2 B
is called the PRODUCT of A and B.
Under this idea we define multiplication of matrix by matrix is defined as follow.
DEFINITION 11.4
Let A (ai j ) m p and B (bk l ) p n be matrices in ℝ , where , and are positive integers. The product of A and
REMARK
From the definition of product of matrices, you can observe that, if A is an m s matrix and B is a t n matrix,
then the product AB is defined only when s t and the product will be a matrix of order m n . That is, the
product AB is defined only when the number of columns of the first matrix A is equal to the number of rows of the
second matrix P.
In this case, we say that the two matrices are COMPATIBLE for matrix multiplication.
2 0
Example 8. If A 5 3 0 and B 3 5 , then find AB.
1 4
Solution. Since the number of columns of A = number of rows of B, A and B are compatible matrices and the
product AB is a 1 2 matrix given by
2 0
AB 5 3 0 3 5 5(2) 3(3) 0(1) 5(0) 3(5) 0(4) 16 15
1 4
2 0
1 2
Example 9. If A and B 1 5 , then find AB.
7 2 4 3
Solution. Since the number of columns of A number of rows of B, A and B are not compatible matrices and
the product AB is not defined.
1 2
1 0 2 1
Example 10. If M 2 3 and T
, then find MT.
0 2 3 1 5
4
Solution. M is a 3 2 and T is 2 4 matrices. Since the number of columns of M = number of rows of T, then
M and T are compatible matrices and the product MT is a 3 4 matrix given by
given by
( )
Compare the 1st row of A with the 1st column of B, and the 2nd column of A with the 2nd column of B. What do you
observe?
From your responses in Activity 6.4, you have observed that, the rows of A are columns of B
and the columns of A are rows of B.
That is, B is obtained from A by changing rows of A to columns of B and columns of A to rows
of B and this process is called TRANSPOSING A MATRIX.
DEFINITION 11.5:The Transpose of a matrix A aij mn
, denoted by AT or At, is the n × m matrix
obtained by interchanging the rows and columns of A. That is AT B b ji of order n × m such that
b ji aij . Thus,
Example 13. Find the transpose of each of the following matrices.
a) ( A B) ( A (B)) A (B) A B =
T T T T T T
7 0 3 1
1 5 3(1) 3(5) 3 15
b) 3A 3
7 0 3(7) 3(0) 21 0
T
2 4 T 2 3 T 2 4
c) B 3 1 4 1 3
T T
B
1
17 1 17 14
T
Hence, AB
T
14 28 1 28
1. Find a) AT b) BT c) A AT d) B BT
2. What do you observe? Is AT A ? BT B ?
From your responses in Activity 6.5, you can observe that
A AT 0 . That is A A . Such types of matrices are known as symmetric matrices.
T
i)
B BT 0 . That is B B . Such types of matrices are known as skew-symmetric matrices.
T
ii)
DEFINITION 11.5
a) Let ( ) be a square matrix of order . Then A is said to be a SYMMETRIC
MATRIX, if . That is, A is a symmetric matrix, if 𝑗 = 𝑗 for 1 ≤ , 𝑗 ≤
b) Let ( ) be a square matrix of order . Then A is said to be a SKEW-
SYMMETRIC MATRIX, if . That is, A is a skew-symmetric matrix, if 𝑗 = 𝑗
for 1 ≤ , 𝑗 ≤ .
Note that, if a matrix ( ) is skew-symmetric matrix, then for , for all 1 ≤ ≤ .
Example 15. Identify the following matrices as symmetric, skew-symmetric or neither.
2 0 3 0 1 3 2 1 4
1 0
a. A 0 1 5 b. M 1 0 5 c. B d. E 1 0 5
3 5 0 3 5 0 0 1 3 5 1
Solution.
2 0 3 2 0
T
3
a. A 0
T
1 5 0 1 5 A . Therefore, A is symmetric matrix.
3 5 0 3 5 0
0 1 3 0 1 3
T
b. M 1 0 5 1
T
0 5 M . Hence, M is skew-symmetric matrix.
3 5 0 3 5 0
T
1 0 1 0
c. B B . Therefore, B is symmetric matrix.
T
0 1 0 1
2 1 4 2 1 3
T
d. E 1 0 5
1 0 5 . Here, you can see that E E and E E .
T T T
3 5 1 4 5 1
That is E is neither symmetric nor skew-symmetric.
5 4 3 3 5 3 2 1 6
2 0 3 2 7 5 0 7 8
A A 7 1 5 0 1 4 7 0 9 ,
T
5 4 3 3 5 3 8 9 0
Here, you can observe that A AT is symmetric and A AT is skew-symmetric matrices.
7
2 2 1
4 7 2
7
B A A 7 2 1
a. 1 1 1 which is symmetric matrix.
T
1
2 2 2 2
2 1 6
1 1 3
2
7
0 2 4
0 7 8
b. C 1 ( A AT ) 1 7 0 9 7 0 9 which is skew-symmetric matrix.
2 2 2
2
8 9 0
4 9 0
2
c. Adding B and C, we obtain
7 7
2 2 1 0
2
4
2 0 3
7 1 7 9
BC 1 0 7 1 5 A
2 2 2 2
5 4 3
1 1 9
3 4 0
2 2
This shows that A B C A AT A AT .
1 1
2 2
symmetric skew symmetric
3 0 1 3 1 4
T
Now, M 1 5 0
0 5 2 .
T
4 2 8 1 0 8
5 1
3
3 0 1 3 1 4 2 2
1
1
2
M M T 1 5 0 0 5 2
2
5 1 and
1
2
4 2 8 1 0 8 5
1 8
2
1 3
0 2 2
3 0 1 3 1 4
1 1
1
2
M M 1 5 0 0 5 2
T
2
0 1 .
1 0 8 2
4 2 8
3 1 0
2
1 5 1 3
3 2 2 0 2 2
Then M M M M M
1 T T 1
5 1 1
0 1
2 2 2
5
1 8 3
1 0
2 2
symmetric skew symmetric
sin cos
2 0 1
12. If A 1 2 0 and (2 A B)T AT A , then find the matrix B.
0 0 1
4 2 0 1
1 2 3
17. If A 3 2 1 , then find A3 23A 40I 0 .
4 2 1
cos sin sin cos
18. Simplify cos sin
sin cos cos sin
19. A trust fund has Br. 30,000 that must be invested in two different types of bonds. The first bond pays 5%
interest per year, and the second bond pays 7% interest per year. Using matrix multiplication, determine how to
divide Br. 30,000 among the two types of bonds. If the trust fund must obtain an annual total interest of: (a) Br.
1800 (b) Br. 2000
cos sin
Let A . If A A I , then find the value of .
T
20.
sin cos
0 2y z
Find the values of x, y and z if x y z satisfy the equation A A I 3
T
21.
x y z
1 2 0 0
22. For what value of x 1 2 1 2 0 1 2 0
1 0 2 x
3 1
If A , then show that A 5 A 7 I 2 0
2
23.
1 2
1 0 2 x
24. Find x , if x 5 1 0 2 1 4 0
2 0 3 1
If A is a square matrix such that A A , then show that I A 7 A I , provided that I is an identity
2 3
25.
matrix having the same order as A.
a) matrix B obtained by crossing-out the 2nd row and the 1st column from A.
b) det(B) and the value of (1)21 |B|.
c) matrix C obtained by crossing-out the 1st row and the 2nd column from A.
d) det(C) and the value of (1)12 |C|.
From your responses for Activity 6.6, observe that
(i) both . / and . / are 2 2 matrices;
Example 21. Let ( ). Then determine the minors and cofactors of the entries , and
The cofactor of = =( )
In a similar manner,
The minor of = | |,
The minor of = | |,
Example 22. Determine the minors and the cofactors of all the entries of the 3 3 matrix
( ).
In a similar manner | | | | | |
| | | | | |
Then The cofactors of all the entries of A are:
ACTIVITY 11.7
Let ( ) . Then find
Example 23. Let ( ). Then, find the determinant of A using cofactor expansion along the
three rows of A.
Solution:
a. The determinant of A using the cofactor expansion along the first row is given by :
| | ( ) ( ) ( )
. | |/ ( ). | |/ . | |/
( ) ( )( ( ( )) ( )
b. The determinant of A using the cofactor expansion along the second row is given by :
| | ( ) ( ) ( )
. | |/ ( ). | |/ . | |/
( )( ) ( )
c. The determinant of A using the cofactor expansion along the third row is given by :
| | ( ) ( ) ( )
. | |/ . | |/ . | |/
( ) ( ) ( )
Thus, the determinant of A using the cofactor expansion along any of its three rows is the same,
that is, | | .
In the similar manner, if you calculate the determinant of A using the cofactor expansion along any of its three
columns, you obtain the same result | | .
Therefore, the determinant of a square matrix is independent of the choice of rows and columns.
Solution. As the second column has two zero entries, more zero entries than the other columns or any rows, you
can use a cofactor expansion along the second column. Thus,
| |
| | | | | | | | ( )
REMARK: In the above discussion, the determinant of a 3 by 3 matrix is calculated by the use of minors and
cofactors which is a general rule for calculating determinants of any squared order. But, for only 3 by 3 matrix,
the following approach can also be used to calculate determinants.
Step 2. Multiply and add the triple numbers presented left to right.
( )( ) ( )( ) ( )( )( )
Step 3. Multiply and add triple numbers presented right to left.
( )( ) ( )( ) ( )( )( )
Step 4. Subtract the result of step 2 from result of step 3. The resulting number is the determinant. Therefore, the
determinant of A is |A| = 2 21 = 23
Properties of Determinants
ACTIVITY 11.8
For a matrix ( ) , do each of the following.
1. | | | | and | | | |
2. ( ) and | | | | ( ) ( )
3. ( ) ( ) and | | | |
For instance, consider a matrix ( ) . then using cofactor expansion along the
| | | | ( ) ( ) ( )
6. If A has two identical rows (or columns), then the determinant of A is zero.
| | | | ( ) ( ) ( )
7. If some or all elements of a row (or a column) of a determinant are expressed as sum of two (or more)
terms, then the determinant can be expressed as sum of two (or more) determinants. That is, for instance;
| | | | | |
( ) ( )
a) | | b) | | c) | |
d) | | e) | |
Hence, | | | | | |
| | | | | |
times the 1st row of A. No other operation has been done on A. Hence,
| | | |
( )
| | | | | | ( )| |
( )| | | | ( )
6 1 3 2
Example 29. Without expansion, show that 2 0 1 4
0.
3 6 1 2
4 0 2 8
Solution. Using property (3) of Theorem 6.1, In the L.H.S, taking 2 common from fourth row,
6 1 3 2 6 1 3 2 6 1 3 2
2 0 1 4 2 0 1 4 2 0 1 4
2 2(0) 0 ,
3 6 1 2 3 6 1 2 3 6 1 2
4 0 2 8 2( 2) 2(0) 2(1) 2(4) 2 0 1 4
since rows 2nd and 3rd are identical in the last determinant.
Example 30. Let A and B be two square matrices of order 3 with |A | = 4 and |B| = 5. Then compute a) AB
b) 2 AT B c) A3
A3 A A A A 43 64
3
c)
Using Property (8) of Theorem 6.1 and the associative property of matrix multiplication, if A
Am A
m
is a square matrix and m is a positive integer, then
Example 31. Let A be two square matrices of order 3 with |A | = 2 . Then compute A5 .
A5 A (2)5 32
5
Solution.
Solution. | | | |
Solution. | | ( )| | | | | |
( )( ( ) ( )) ( ( ) ( )) ( ( ) ( ))
( )( ) ( ) ( )
( ) ( ) ( )
or
| | | | | |
| | | | ( )
⏟ ⏟
( ) ( )
| | | | | | | | | |
, ( ) ( ) ( )- , ( ) ( ) ( )-
, ( ) ( ) ( )- , ( ) ( ) ( )-
( ) ( ) ( )
sin s 3 1 4
a) | | b) | sin sin | c) 6 2 8
s sin 1 5 7
2. Show that
1 1 2 1
3 1 4 5
a) | | b) | | ( )( )( ) c) 60
7 6 1 2
1 1 3 4
1 1 1
3. Find if a) | | | | b) | | | | c) t 1 t 0
1 t 2 3
4. Find equation of the line joining (3, 1) and (9, 3) using determinants.
5. Find the area of a triangle having vertices P(1, 5), Q(2, 0) and R(1, 3).
2 1 3
6. The 3×3 matrix A is defined in terms of the scalar constant k by A k 2 4 .
k 2 3 k 7
If A 8 , then find the possible values of k.
7. Verify each of the following statements is TRUE. Assume that all letters represent non-zero
real numbers.
1 a bc
x tw x t x w a rb b a b
a) b) c) 1 b c a 0
y su y s y u c rd d c d
1 c ab
1 5
Example 41. Find the adjoint of the square matrix E .
3 1
Solution. Let E (ei j ) and ci j = the cofactor of the entry ei j . Then
c11 (1)11 1 1 , c12 | 5 | 5 , c21 | 3 | 3 and c22 |1| 1
1 5
Hence, the matrix of cofactors of E is C .
3 1
1 5 1 3
T
6 3 2
Hence, the matrix of cofactors of A is C 2 1 0 .
2 2 2
6 2 2 6 3 2
T
Therefore the adjoint of A is given by adj A C 3 1 0 2 1 2
T
2 2 2 2 0 2
ACTIVITY 11.10
Consider a matrix . /. Then find a) |A| b) adj A c) A(adj A) d) (adj A)A e) |A| I2
f) what do you observe from the above results?
From your responses in Activity 6.10, observe that (adj A) A | A | I 2 A(adj A)
This property of adjoint matrix is holds true for a square matrix of order n.
Statement (ii) above implies that if A 0 , then A is invertible or non-singular and the inverse of A is given by
1
A1 (adj A) .
| A|
Example 43. Let A be a square matrix of order 3 and |A| = 2. Then find adj A .
Solution. From the relation (adj A) A | A | I3 , taking determinant of both sides, we have
(adj A) A | A | I3
adj A A | A |3 I3 …………… (since | AB || A || B | and | k I 3 | k | I 3 | )
3
adj A 22 4
ACTIVITY 11.11
a b
Consider a square matrix of order 2 given by A . Then find
c d
d) adj A
T T
a) A b) adj A c) adj ( AT )
e) what do you observe from the above results?
From your responses in Activity 6.10, observe that
Now, you are at an appropriate stage of computing the inverses of invertible square
matrices.
From the definition of inverse matrix and property (ii) of adjoint matrices, if A 0 , then
1 1
(adj A) A I n A(adj A) .
| A| | A|
1
This shows that A is invertible and its inverse is given by A1 (adj A) . This proofs the following theorem of
| A|
inverse.
THEOREM 11-2 : A square matrix A is invertible, if and only if A 0 . If A is invertible, then,
1
A1 (adj A)
| A|
0 2 3
Example 43. Find the inverse of the square matrix A 1 3 3 , if invertible.
1 2 2
Solution. First check whether A is invertible.
0 2 3
3 3 1 3 1 3
A 1 3 3 0 (2) (3) 1
2 2 1 2 1 2
1 2 2
| A | 0 . This implies that A is invertible.
The cofactors of A are
0 1 1
Then the matrix of cofactors of A is C 2 3 2 .
3 3 2
0 1 1
T
0 2 3
The adjoint of A is adj A C 2 3 2 1 3 3
T
3 3 2 1 2 2
Therefore, the inverse of A is
0 2 3 0 2 3 0 2 3
1 1
A1 (adj A) 3 3 (1) 1 3 3 1 3 3
| A| 1 1 2 2 1 2 2
1 2 2
1 2 1
Example 43. Show that matrix A 3 4 5 is singular.
4 2 6
1 2 1
4 5 3 5 3 4
Solution. A 3 4 5 1 2 (1) 1(24 10) 2(18 20) 1(6 16) 0
2 6 4 6 4 2
4 2 6
Hence, A is singular (has no inverse).
a b
REMARK: Given a 2 2 square matrix A .
c d
d b
1. The adjoint of A is given by adj A .
c a
1 1 d b
A1 (adj A)
A ad bc c a
2. If A 0 , then A is non-singular and
Example 44. Check whether each of following matrices are non-singular.
4 3 1 2
a) A b) B
1 3 3 6
Solution.
4 3
a) A 4(3) (3)(1) 9 0 .
1 3
a b e f ae bg af bh
AB
c d g h ce dg cf dh
1 cf dh (af bh)
( AB)1 …………………………> (*)
| AB | (ce dg ) ae bg
1 d b and 1 h f
A1 B 1
| A | c a | B | g e
1 h f 1 d b 1 h f d b
B 1 A1 .
| B | g e | A | c a | A || B | g e c a
1 dh cf (bh af ) …………………………> (**)
| A || B | (dg ce) bg ae
Comparing (*) and (**) , we obtain the relation ( AB)1 B1 A1
1
REMARK: If A is an invertible matrix of order n, then A1 A
1 1
Proof: A is an invertible matrix implies that A 0 . Now, from A (adj A) , we have
A
…………………….. since kA k | A |
n n
1 1
A1 ( adj A)
A
adj A
A
n 1
…………………….. …………… since adj A | A |
n
1 n 1
A
A
1 1
A
A
Solution. A1 1 1 .
| A| 5
Exercise 11.3
1 0 2 11 2 2
1. Show that matrices 2 1 3 and 4 0 1 are inverse to each other.
4 1 8 6 1 1
2. Find the inverses of each of the following matrices, if exist.
4 0 3 2 1 0
2 6 4 2
a) b) c) 1 2 1 d) 2 2 3
1 2 12 6 0 3 2 1 3 1
3. If A is a square matrix of order 3 and | A | = 5, then find |5A|.
4. If A is a square matrix of order 3 and | A | = 10, then find A(adj A) .
4 k 3
5. Find k if A 7 3 6 is singular matrix.
2 3 1
2 2 4
6. Show that A 5 x 2 2 is non-singular for all values of real numbers x.
1 x
3
4 4 4
7. Given the 3×3 matrix A 1 0 1 . Determine the values of the constant , so that A I is
7 6 7
singular.
8. The 3×3 matrices A and B are defined in terms of a scalar constant k by
cos sin 0
9. Given A sin cos 0 , show that A1 AT .
0 1
0
k 9 2 1 3 2
10. A 1 k 0 and B k 2 1 . Find the possible values of k so that the product AB is
5 1 1 4 1 1
singular.
3 1 1
11. It is given that A and B are 3×3 matrices that satisfy det( AB) . If det( A ) , then find
4 20
det( B1 ).
Solution.
2 3 1 4
a) . A R2 R1 B
1 4 2 3
2 3 R1 3 R1 3(2) 3(3) 6 9
b) A C
1 4 1 4 1 4
2 3 2 3 2 3
c) A R2 R2 2 R1 D
1 4 1 2(2) 4 2(3) 3 2
These three new matrices B, C and D are said to be equivalent matrices.
REMARK: In this book, we use only elementary row operations in solving system of linear equations.
DEFINITION 11.11
Two matrices are said to be ROW (OR COLUMN) EQUIVALENT, if and only if one is obtained from the other
by performing successive applications of a finite number of elementary row operations.
Thus, matrices A, B, C and D obtained in Example 46 are row equivalent matrices, denoted by
A B C D.
2 4 6
Example 47. Let A 4 5 6 . Apply appropriate elementary row operations to change A an upper
2 7 12
triangular matrix.
2 4 6 1 2 3
Solution. A 4 5 6 R1 R1
1 1
2 4 5 6 (Multiplying R1 by 2 .
2 7 12 2 7 12
R2 R2 4 R1
1 2 3
0 3 6 (Replacing R2 by R2 4R1 and R3 by R3 2R1
R3 R3 2 R1
0 3 6
1 2 3
R3 R3 R2 0 3 6 (Replacing R3 by R3 + R2 )
0 0 0
1 3 2 1 3 2 1 3 2
b) B 0 1 2 R1 R1 0 1 2 R3 R3 4 R1 0 1 2
4 1 2 4 1 2 0 11 6
1 3 2 1 3 2
R3 R3 11R2 0 1 2 R3 281 R3 0 1 2 (This is REF of B)
0 0 28 0 0 1
1 3 2 1 0 2 1 0 0
R2 R2 2 R3 0 1 0 R1 R1 3R2 0 1 0 R1 R1 2 R3 0 1 0 I3
0 0 1 0 0 1 0 0 1
Which is the RREF of B.
Exercise 11.4
1. Reduce each of the following matrices into row echelon form.
5 0 1 1 1 3 6
1 1 1 5
a) A b) B 1 1 0 c) C 5 3 2 4
4 8 1 6 0 1 4 1 3 4 11
2. Reduce each of the following matrices into row reduced echelon form.
0 2 2 0 1 1 0 1
a) A
2 1 b) B 2 3 0 c) C 1 2 2
1 1 4 3 1
1 3
4
0 2 2 0 3
x
31 1 32 2
a a x a x ..... a x b
form
33 3 3n n 3
…………………………..> ( * )
.....................................................
.....................................................
am1 x1 am 2 x2 am3 x3 ..... amn xn bm
where ai j ; i 1, 2,...., m and j 1, 2, ..., n are real numbers
If b1 b2 b3 ... bm 0 , then the system is said to be HOMOGENEOUS
Example 53.
2 x y 2
i) is a system of two linear equations in two variables x and y.
5 x 3 y 0
3x1 x3 0
ii) is a homogeneous system of two linear equations in three variables x1 , x2 and x3 .
3x1 x2 5 x3 0
3x 4 y z 2w 0
iii) x y 3z 3 is a system of three linear equations in four variables x1 , x2 and x3 .
5 x y z 2w 1
REMARK:
The system of linear equations (*) can be written in matrix form as
AX B …………> (**)
a11 a12 a13 ... a1n
x1 b1
a21 a22 a23 ... a2 n
a a a ... a3n x2 b2
where A ai j 31 32 33 , x and b
mn X 3 B 3
........................... ... ...
........................... ... ...
am1 am 2 am3 ... amn xn bm
Matrix A is called THE COEFFICIENT MATRIX of the system.
a11 a12 a13 ... a1n b1
a21 a22 a23 ... a2 n b2
The matrix form a a32 a33 ... a3n b3
( A | B ) 31
...........................
...........................
am1 am 2 am 3 ... amn bm
obtained by adjoining B to A is called the AUGMENTED MATRIX of the system.
z 1
Now, starting from the bottom, we substitute back into the variables and find the solution.
From the last equation, you have z 1
Consider the second equation, y 32 z 12 , with z 1 from above and solving for y gives
y 32 z 12 23 (1) 12 24 2
Use the first equation x 2 y z 2 , and z 1 and y 2 from above to solve for x .
Then, solving for x gives you that x 2 y z 2 x 2 2 y z 2 2(2) (1) 1
0 3 1 3 3
the number of non-zero rows is less than the number of variables. Therefore, the given solution has infinite number
of solutions.
x 2 y z 0
The given system is equivalent to the system of equations
y 3 z 1
1
The variable which corresponds to leading 1‟s are x and y. This variables are known as DEPENDENT
VARIABLES. The other variable Z known as FREE VARIABLES.
The solutions of such types of system are written as “the dependent variable interms of the free variables.
Now, from the second equation, solving for y in terms of z , you get y 13 z 1 y 13 z 1
1 3 1 1
Solution. The augmented matrix of the system is 2 1 4 1
6 7 8 7
1 3 1 1 1 3 1 1 1 3 1 1
R2 R2 2 R1 1
2 1 4 1 0 7 6 1 R2 7 R2
1
0 1 7 7
6
6 7 8 7 R3 R3 6 R1 0 11 2 13 0 11 2 13
1 3 1 1 1 3 1 1
1
R2 R2 0 1 76 71 R3 R3 11R2
1
7 0 1 7 7
6
0 11 2 13 0 0 80 80
7 7
1 3 1 1
R3 R3 0 1 76 71 (the first 3 columns are REF of A)
7
80
0 0 1 1
1 0 1 2 1 0 0 1
R2 R2 76 R3
0 1 0 1 R1 R1 R3 0 1 0 1
R1 R1 3R2 0 0 1 1
0 0 1 1
This implies that the given system is equivalent to the system of equations
x1 1
x2 1
x 1
3
Hence, the solution of the given system has a unique solution : x1 1, x2 1, x3 1 .
Therefore; the solution set is S.S = {(1, 1, 1)}
NOTE:
The system of n n linear equations AX = B has unique solution if A 0
x 2 y 3z 5
Example 60. For what values of k does the system 2 x y z 8
kx y 2 z 14
a) has a unique solution? b) has no solution? c) Has an infinite solution?
0 1 2k 2 3k 14 5k 0 0 3 k 6821k
5
1 2 3 5
R3 (2k 1) R2 R3 0 1 1 52
0 0 3 k 6821k
5
a) From the last matrix, the system has unique solution if 3 k 0 k 3
An alternative way is to use the fact that the system has a unique solution is if A 0
1 2 3
1 1 2 1 2 1
2 1 1 1 2 (3)
1 2 k 2 k 1
k 1 2
(2 1) 2(4 k ) 3(2 k ) 5k 15
| A | 5k 15 0 k 3
68 21k
b) The system has no solution if 3 k 0 and 5 0 k 3 and k 68
21
68 21k
c) The system has infinite solution if 3 k 0 and 5 0 k 3 and k 68
21
a 2 2 matrix obtained from A by replacing its 2nd column by the constant column matrix B.
This rule of solving system of equation is called CRAMER‟S RULE.
This rule is valid for any system of linear equations in so far as the coefficient matrix A is square matrix having
non-zero determinant. But, the repeated calculation of determinants may be tiresome.
CRAMER‟S RULE FOR 2 2 SYSTEM OF LINEAR EQUATIONS:
a x b1 y c a1 b1
Consider a system of linear equation 1 with A 0.
a2 x b2 y d a2 b2
a b1 c b1 a c
Let D 1 , Dx and Dy 1 .
a2 b2 d b2 a2 d
Dx Dy
Then the system has unique solution given by x , y
D D
2 x y 3
Example 60. Solve the system of equations using Cramer‟s Rule.
3 x 5 y 1
2 1 3
Solution. The coefficient matrix A is A and the column matrix of constants is B .
3 5 1
2 1
Since D | A | 2(5) (1)(3) 7 0 , Cramer‟s Rule is applicable.
3 5
3 1 2 3
Hence, Dx 3(5) (1)(3) 12 and Dy 2(1) (3)(3) 7
1 5 3 1
Dx 12 Dy 7
This implies that x , y 1
D 7 D 7
12
Therefore, the solution of the system is x
, y 1
7
NOTE: Under similar conditions, the rule holds for 3 3 system of linear equations too.
Dx Dy Dz
system has exactly one solution. The solution of the system is given by x , y , z
D D D
a11 a12 a13 b1 a12 a13 a11 b1 a13 a11 a12 b1
where D a21 a22 a23 , Dx b2 a22 a23 , Dy a21 b2 a23 and Dz a21 a22 b2 .
a31 a32 a33 b3 a32 a33 a31 b3 a33 a31 a32 b3
x 4 y z 1
Example 61. Solve the system of equations 2 x y z 0 using Cramer‟s Rule.
x y z 1
1 4 1 1
Solution. The coefficient matrix A is A 2 1 1 and the column matrix of constants is B 0 .
1 1 1 1
1 4 1
D A 2 1 1 1(1 1) 4(2 1) (1)(2 1) 5 0
1 1 1
This implies that the system has unique solution.
1 4 1 1 1 1 1 4 1
Dx 0 1 1 1, Dy 2 0 1 2 , Dz 2 1 0 4
1 1 1 1 1 1 1 1 1
Exercise 11.5
1. Using the Gaussian Reduction Procedure, solve the system of linear equations
x y z x y z x y z
a) { x y z b) { x y z c) { x y z
x y z x y z x y z
2. Using the Gaussian Jordan Method, find the solution set of the equation
x y z x y z
x1 x2 2
a) b) { x y z c) {x y z
3x1 2 x2 0 x y z x z
a) { b) ( )( ) ( )
x ny
9. Determine the values of m and n for which the system { has
x y
a) A unique solution b) No solution c) Infinite many solution
REVIEW EXERCISES ON UNIT 11
INSTRUCTION: Workout each of the following questions correctly
a) b) ( )
2. Construct a 4 5 matrix ( ) where i
3. Given ( ) ( ) , then
b) c) d) ( )
13. Given a square matrix ( ). Then describe C as a sum of symmetric and skew-
symmetric matrices.
s sin
14. If . / and ( 𝑗 ) , then find the value of k.
sin s
a. ( ) c. ( ) e. ( )
b. ( ) d.
⁄
( )
19. Suppose that A and B are 3 3 matrices, I is identity matrix of order 3 such that AB = 2I. If
|B| = 6, then find |AT|.
20. If . / and . /, then find ( ) .
30. Consider the system { . If the determinant of the coefficient matrix is 2, then
0 4 1
9. If 2 1 5 and B is a 3 3 matrix such that det2 B 40 , then det AB is equal to:
0 3 2
A. 200 B. 200 C. 50 D. 50
x y z 1
10. What should be the value of k so that the system of equation x 5 y 4 z 1 has one solution?
2x 2 y z k
A. 0 B. 1 C. 4 D. 4
11. Suppose that A and B are 3 3 matrices, I is the identity matrix of order 3 such that AB 2 I . If
det B B 6 . What is detAT ?
1 4
A. B. C. 12 D. 48
3 3
1 2
12. Suppose A . If X is a 2 2 matrix such that AX AT 2 A , then what is the value of X ?
2 3
3 1 3 3 3 6 3 0
B. C. D.
3
A.
1 3 3 6 9 0 3
x y z 1
13. Consider the system x 2 y 4 z 0 . If the determinant of the coefficient matrix is 2 , then what is the
5 x y z 0
solution of the system of equations?
19 11 19 11 3 19 11 3 19 9
A. 3 , , B. 3, , C. , , D. , ,
2 2 2 2 2 2 2 2 2
6, 2, 8 B. 0, 1 5
T
AX b ? A. C. 3, 1, 4 D.
T T
,
a b
x 3y 2z 6
24. The solution of the system of linear equation of 2 x 4 y 3z 8 is:
3x 6 y 8 z 5
A. x 1, y 3, z 2 C. x 1, y 3, z 2
B. x 1, y 3, z 2 D. x 1, y 3, z 2
25. For any n n square matrix A , which one of the following is true?
A.
det A det AT , where AT is the transpose of A .
B. If k is a scalar, then detkA k n det A .
C. If B is a matrix obtained from A by interchanging of two rows of A , then detB det A .
D. If A is invertible, then det A det A1 .
x 4 y z 1
26. The solution of 2 x y z 0 using Cramer‟s rule is:
x y z 1
1 2 4
A. x 1, y 2, z 5 C. x , y ,z
5 5 5
1 2 2
B. x , y ,z D. x 5, y 1, z 2
2 3 5
1 3 3 1
27. For what value of x is x 1 A. 1 B. C. D. 1
x x2 2 2
0 3 4
31. Let k 0 8 be skew-symmetric matrix. Then what is the value of k ?
4 8 0
3
A. 3 B. C. 3 D. 4
2
2 x y 3z 4
32. The augmented matrix associated to the system of equations x z 1 is
4 x y z 0
3 2 1 1 3 1 2 4 1 2 3 4 2 1 3 4
A. 1 1 1 3 B. 1 0 1 1 C. 1 1 1 1 D. 1 0 1 1
1 4 1 4 4 4 3 0 1 4 1 3 1 4 0 3
A. *( )+ B. 2. /3 C. 2. /3 D. 2. /3
x y z
34. If the system {x y z has infinitely many solutions, then the values of and are
x y z
A. ∈ C.
B. D. ∈
Main Contents:
12.1. REVISIONON VECTORS AND SCALARS
12.2. REPRESENTATION OF VECTORS
12.3. COMPONENTS OF VECTORS
12.4. PRODUCT OF VECTORS
12.5. APPLICATION OF VECTORS
12.6. TRANSFORMATION OF THE PLANE
Key terms
Summary
Review Exercises
NOTE: If both the initial and terminal points of a vector v lie at the origin, then v is said to be the zero vector
and is given by v = (0, 0) or =. /
PROOF: Take any point O and draw the vectors = u and = v such
that the terminal point of the vector u is the initial point of the vector v, as
show in figure. Then, by definition of vector addition you have:
. . . . . . . . . . . . . . . . . . . . . . . . > (1)
Now, completing the parallelogram OABC whose adjacent sides are and , you
infer that , and .
Using the triangle law of vector addition, you obtain
Hence, from the above given set of vectors, you have observed that the direction of vector u remains the same
when the value of the scalar is positive and the direction becomes exactly opposite when the value of the scalar is
negative. In both cases the magnitude keeps changing depending upon the values of the scalar multiple.
DEFINITION 12.5: If v is a non-zero vector and k is a non-zero number (scalar), then the product kv is
defined to be the vector whose length is | | times the length of v and whose direction is
i) the same as that of v if k > 0 and
ii) opposite to that of v if k < 0.
If k = 0 or v = 0, then kv = 0.
A vector of the form kv is called a SCALAR MULTIPLE of v.
Example 4. Let v be any vector. Then 3v is a vector in the same direction as v and with length 3 times the
length of v.
Example 5. Let v be any vector. Then v is a vector in the opposite direction as v and with length half as
long as the length of v.
Example 7. Sketch the resultant vector of the following force vectors using the tail-to-head method using a
Cartesian plane.
• in the positive y-direction
• F2 = 1,5 N in the positive x-direction
• in the negative y-direction
• in the negative x-direction
Solution.
Exercise 12.2
1. Observe the following vectors. Complete the chart for X-
component and Y- component for all the vectors.
2. A student walks a distance of 3km due east, then another 4km due south. Find the displacement relative to
his starting point.
3. What are the x and y components of the plane‟s velocity?
, and
where ,
Note that the order that we add the vectors does not matter.
Example 9. Suppose two forces = 15 N acts at an angle of 300 to the horizontal and = 27 N acts at an
0
angle of 53 to the horizontal. Find the resultant force, .
Solution. Resolving each vectors into components gives:
| | s ( s ) ( )
| | sin (sin ) ( ) and
| | s ( s ) ( )
| | sin (sin ) ( )
Then the components of the resultant are
| | (sin )
REPRESENTATION OF VECTORS: As seen in definition 8.2, If v is a vector in the plane whose
initial point is the origin, O (0, 0) and whose terminal point is Q( , ) then the coordinate form of a
vector v = ( ) or the column form of a vector v = . /.
For instance, the coordinate form of a vector whose initial point is the origin and whose terminal point is
Q(3, 7) is v = (3, 7) or in column form v = . /.
If v is a vector in the plane whose initial point P( 1 , 1) and whose terminal point Q( 2 , 2), then the
coordinate form of a vector v = ( ) or the column form of a vector v=
. /.
For instance, the coordinate form of a vector whose initial point A( 2, 5) and whose terminal point is
B(3, 1) is ( ( ) ) ( ), or in column form v = . /.
| | √.√ / . / √
NOTE: The unit vectors in the direction of positive x-axis and positive y-axis, respectively, are (1, 0)
and (0, 1). These unit vectors are known as STANDARD UNIT VECTORS in the xy-plane.
Notation: the standard unit vectors in the coordinate plane are denoted by
̂ ( ) and ̂ ( ).
Thus, | ̂ | and | ̂ |
Now consider a vector ( ) in a coordinate plane.
Then by vector addition and scalar multiplication, can be expressed as
( ) ( ) ( ) ( )
( ) ( ) ̂ ̂
Thus, any vector ( ) in the plane can be expressed uniquely in the
form
( ) ̂ ̂
in this case we say that is expressed as a linear combination of ̂ and ̂.
Example 11. Express the following vectors in terms of the unit vectors ̂
and ̂ and findtheir norm. a. (7, –8) b. (–1, 5) c. (0, 3) d. ( , 2)
̂ ̂
| | | ̂ ̂| √( ) ( ) √
√ √ √ √ √
Example 12. For each of the following vectors, find the unit vector in the direction of the vectors.
a. u = (3, –2) b. u = ̂ ̂ c. u = ̂ d. u = ( , 0)
Solution
a. u = (3, –2) . Its nom is |( – )| √ ( ) √ √
( – )
Then the unit vector in the direction of u is | |
̂ ̂
√ √ √
b. u= ̂ ̂. Its nom is | ̂ ̂| √( ) √ √
̂ ̂
Then the unit vector in the direction of u is | |
̂ ̂
√ √ √
c. u= ̂. Its nom is | ̂| √ . / √
| |
(– ) . / ̂ ̂
√ √ √ √ √
b. u= ̂ √ ̂. Its nom is | ̂ √ ̂| √ √ √
.
Example 17. For two vectors u and v, and is the angle between u and v, find u.v,
a. if | | ,| | and c. if | | and | |
b. if | | ,| | and
Solution.
(a) (b)
(c)
(d) (e)
i) When . Then and are parallel and in the same direction. (See Figure (a))
| || | s | || |
ii) When . (i.e. is acute) (See Figure (b)). This implies that s .
√
Now, | || | . /. /
| || | √ √
√
( )
Example 23. Let u and v be vectors with |u| = 3 and |v| = 6 and angle between them is θ = .
a. Find |3u − 2v|. b. Find the cosine of the angle between vectors 3u − 2v and u.
Solution : In this problem, the property | | is important.
| || | ( ) . / . /
a. Using the properties of dot product, you have
| | ( )( )
| | | | ( ) ( ) ( )
Then | | √ √
b. Let be the angle between vectors 3u − 2v and u.
( ) ( ) ( ) | | ( ) ( )
Now, ( ) | || | s
√
45 = ( )( √ ) s s ( )( √ ) √
Example 24. For two non-zero vectors u and v, show that u.v = 0 |u + v|2 = |u|2 + |v|2.
Solution :
i) Let u.v = 0.
| | ( )( ) | | | |
| | | | ; since
| | | | ; since
2 2
ii) Let |u + v| = |u| + |v|2
| | ( )( ) | | | |
|u| + |v|
2 2
| | | |
| | | | | | | |
,
, where t is a scalar.
Then we have the following results.
1. If a direction vectors v of the line ℓ and a point ( ) on ℓ are given, then the VECTOR
EQUATION of the line determined by Po and v is given by
, where t is a scalar
Thus, the vector equation of a line ℓ containing and parallel to vector v is given by
, where t is a scalar
2. If ( ) , ( ) and ( ), then the above equation can be written as
( ) ( ) ( ) , where t is a scalar and ( ) ( ).
( )( ) ( )( ) ( )( )
Example 36. Find the equation of the circle whose end points of a
diameter are P(−1, −1) and Q(4, 11).
Solution : The centre of the circle is ( ) midpoint of
( ) . / . /
The radius of the circle is given by ( ) √( ) ( )
Let R(x, y) be a point on the circle and and be position vectors of R and C, respectively.
Substituting the corresponding components of and in equation (ii), you obtain:
( )( ) ( ). / . / . / . /
Thus, the equation of the circle is
12.5.4 Equation of Tangent Lines to a Circle
A tangent line to a circle is a straight line that touches the circumference of the circle at the point of
tangency. A line tangent to a circle is characterized by the fact that the radius at the point of tangency is
perpendicular (orthogonal) to the line.
Consider a circle with center ( ) and a radius of r units as
shown in the figure below.
Let ( ) be a point of tangency to the circle. Since ( )
is a point on the circle, it fulfilled the equation of the circle: (
) ( )
If ( ) is an arbitrary point on the tangent L, then
Thus, the equation of the tangent line is ( )(
)
( ) ) ( )( )
Adding ( ) ( ) to both sides of the equation above, you obtain
( )( ) (( )( ) ( ) ( )
Factorizing ( ) and ( ) from respective terms, you have
( )( ) ( )( )
( )( ) ( )( )
Thus, the equation of the tangent line L to a circle with center ( ), radius and point of tangency
to the circle at ( ) is given by ( )( ) ( )( )
This equation can be written as dot product :
Example 37. Find the equation of the tangent to the circle x2 + y2 – 4x + 6y + 4 = 0 at (2, 0).
Solution : By completing the square, the equation can be written as ( ) ( )
Example 38. Find equation of tangent line through the point R1(3, 4) at a point on the circle x2 + y2 =
25.
Solution : Since the center of the circle is at the origin, the equation of the tangent is given by
( ) ( )
Exercise 12.5
1. Find the parametric and symmetric equations of the line passing through the point (-3, 5) and
parallel to the line x = 1 + 3t, y = –1 – 2t.
2. Find a) the vector equation b) the parametric equation and c. the standard equation of the
line though (2, –1) and (5, 3).
3. Determine whether each of the following pairs of lines are parallel, perpendicular or neither.
i) L1 : ; L2 :
ii) L1 : ; L2 : ( ) ( ) ( )
iii) L1 : ; L2 :
4. Find the vector equation of a line .
5. Find the line through P = (1, 1, 1) and perpendicular to the line ( ) ( ) ( ).
6. Find the vector equation of line passing through the point having vector 5i + 4j and having
direction vector ( −3, 4).
7. Find the standard form of equation of a line ( ) ( ) ( ) ∈ .
8. Given an equation of a line ℓ by P = (1, 0) + t (2, 2), ∈ ., find out whether the, points
A(1, 0), B(2, 2), C(–5, –6) and D (3, 0) lie on ℓ. For those of them lying on ℓ find the
respective values of the parameter t.
9. The standard equation of a line is . Rewrite the equation in parametric and vector
equation form.
10. Find a vector equation of a line L passing through the point ( ) and parallel to the line
containing points ( ) and ( ). Check whether the point P( ) lies on L.
11. Are the points A, B and C collinear?
a) A(1, –4), B (–2, –3), C(11, –11) b) A(–2, –3), B(4, 9), C (–11, –21)
12. Find the point of intersection of the pair of lines L1 : and
L2 : { ∈
13. Find the cosine of the angle between the lines L1 : , ∈ and
L2 :
14. Find an equation of the circle centred at C (1, −3) and radius 6.
15. Find the equation of the circle whose end points of a diameter are P(2, 2) and Q(5, 3).
16. Find an equation of the circle centred at origin and passes through (1, 3).
17. Find an equation of the circle cantered at C (−4, 2) and passes through (2, −5).
18. Find the equation of the tangent to the circle at the point (−4, 8).
19. Find equation of the tangent line through P(−4, −3) point on the circle .
20. Show that the given point lies on the circle and find the equation of the tangent at the point.
a) x2 + y2 – 2x – 4y – 9 = 0 at P(1, 4) b) (x+2)2 + y2 = 3 at P(–1, 2
Let A(2, –3) and B (5, 4) be points on the plane.. Compare the lengths of and when
a. T((x, y)) = (x, 0) c . T ((x, y)) = (x, –y)
From the above Activity, you have observed that some mappings, called Transformations of the plane
onto itself, of objects preserve shape, size or distance between any two points and some other mapping of
objects are not preserve shape, size or distance between any two points. Due to this reason,
transformations are classified as either rigid motion or non-rigid motion. But in this section, we will
focus only on rigid motions.
DEFINITION 12.10
A motion is said to be RIGID MOTION, if it preserves distance. That is for two distinct points P and Q,
PQ = P'Q' where P' and Q' are the images of P and Q, respectively. Otherwise it is said to be NON-
RIGID MOTION.
12.6.1 Translation
Translation is a type of rigid motion that occurs when the object simply slides and maintains its direction.
In a translation, everything is moved by the same amount and in the same direction. Every translation has
a direction and a distance.
ACTIVITY 12.3
Consider a triangle ABC. Discuss what will happen to the shape, size and orientation of a triangle when
you slide it to the new position , as shown in the figure below.
From the above activity, you have observed that and have the same size, shape and
orientation. This type of transformation is said to be a TRANSLATION.
An easy way to remember what translation means is that a translation is a change in location.
√(( ) ( )) (( ) ( )) √( ) ( )
P(6, 2) is a point on the given line ℓ. The image P' of P is given by
( )
P( ) → P( ) ( )
Then the image line ℓ' has slope and contains the image point ( ).
Therefore, by slope – point form of equation of a line, the equation of the image line ℓ' is
ℓ' : ( )
ℓ' : ( )
ℓ' :
d. is vertical line. Its image ℓ' is also vertical given by
Thus, the given ellipse has center ( ), and lengths of semi-major and semi-minor axis √
and √
This implies that the image ellipse ha center
Or E' :
Example 45. Determine the equation of the parabola x2 + 6y = 18 under translation by a translation
vector u = (3, 2).
Solution. The translation vector is u = (h, k) = (3, 2).
The curve to be translated is a vertical parabola P : x2 + 6y = 18 which opens downward.
Its standard form is P : ( ) ;
P' : ( ) . /( )
P' : ( ) ( )
P' :
Therefore, the standard form of the image parabola is P' :
Combined Translation
THEOREM 12.6: The composition of two (or more) rigid motions is a rigid motion.
Notation: Let S and T be two rigid motions. Then
i. The rigid motion S followed by T, denoted by , is given by: ( )( ) ( ( ))
ii. The rigid motion T followed by S, denoted by , is given by: ( )( ) ( ( ))
Example 46. A translation T takes the origin to A(3, –2). A second translation S takes the origin to B(–2,
–1). Find
i. where T followed by S takes the origin, and
ii. where S followed by T takes the origin.
Solution.
Exercise 12.6
1. If a translation T takes the origin to the point A(–3, 2), find the image of the rectangle ABCD with
vertices A(3, 1), B(5, 1), C(5, 4) and D(3, 4).
2. Triangle ABC is transformed into triangle A'B'C' by the translation vector (4, 3).
If A = (2, 1), B = (3, 5) and C = (–1, –2), find the coordinates of A', B' and C'.
3. Find the equation of the image of the circle (x + 1)2 + (y – 3)2 = 5 when translated by the vector ,
where P = (1, –1) and Q = (–4, 3).
4. A translation T takes the origin to A(3, –2). A second translation S takes the origin to B(–2, –1).
Find where T followed by S takes the origin, and where S followed by T takes the origin.
5. If a translation T takes (2, –5) to (–2, 1), find the image of the line ℓ : 2x – 3y = 7.
6. If a translation T takes the origin to (4, –5), find the image of each of the following lines.
a. y = 3x + 7 b. 4y + 5x = 10
7. If the point A(3, –2) is translated to the point A'(7, 10), then find the equation of the image of
a. the ellipse 4x2 + 3y2 – 2x + 6y = 0 b. the parabola y2 = 4x
c. the hyperbola xy = 1 d. the function f (x) = x3 – 3x2 + 4
12.6.2 Reflections
As the name indicates, reflection transforms an object using a reflecting material.
From the above Activity, you have observed that ABC and its reflection A‟B‟C‟ have the same shape
and size, but the figures face in opposite directions. Having these concepts, let us define reflection as
follows.
DEFINITION 12.11
Let L be a fixed line in the plane. A reflection M about a line L is a transformation of the plane onto itself
which carries each point A of the plane into the point of the plane such that L
L L
A
Let be the angle between the x-axis and the line . Let A be the point of intersection of L and ,
and .
Thus, ( ) → ( )
2. In a similar manner, when = , you will have reflection in the y-axis.
Thus, ( ) → ( )
a) ( ) → ( ) c) ( ) → ( )
b) ( ) → ( )
Example 50. Find the image of the point A(5, 1) after it has been reflected along .
Solution. Since reflection along the line maps ( ) onto ( ), we have
( ) → ( )
Example 51. Find the image of the point A(2, 1) after it has been reflected along √ .
Solution. The reflecting line L : √ has the form with
sin . / s . / sin s
√ √
. / . / √
√ √
Thus, the coordinates of the image point of A(2, 1) are . /
Example 52. Find the image of A(0, 3) after it has been reflected along the line .
Solution. The reflecting line L : has the form with
Thus, if is the angle of inclination of L, then t n . is a 1st quadrant angle
sin and s
√ √
. / . / , and
. / . / , and
Let be a line that passes through the given point A and the image point A'. This is to say ⃡ . Since
we have slope of =
Let the lines L and t intersect at point B. then B is the midpoint of A and A'.
therefore, using points A and B, we can determine the coordinates of the image point .
STEP 3: Find the point of intersection B of L and t which serves as the midpoint of ̅̅̅̅̅.
STEP 4: Using A as the mid-point and P as one end point of ̅̅̅̅̅, find the coordinates of P'.
Example 53. Find the image of the point P(1, 3) after it has been reflected along the line .
Solution.
The reflecting line L : has slope and the point to be reflected is P(1, 3).
Let be the line passing through P(1, 3) and image P'(x', y').
The equation of is ( ) ( )
,
This point of intersection . / is the mid-point of P(1, 3) and P'(x', y').
and
Thus, the image of the point P(1, 3) after it has been reflected along is ( ).
NOTE : The steps for determining the image of a point along the reflecting line also works
for the reflecting line .
Example 54. Find the image of the point A(2, 0) after it has been reflected along the line .
Solution: The reflecting line L : has slope and the point to be reflected is A(2, 0).
Let be the line passing through A(2, 0) and image A'(x', y').
The equation of is ( )
Thus, the image of the point A(2, 0) after it has been reflected along is ( ).
Example 55. If a reflection along a line L maps a point ( ) into the point ( ) , then find the
equation of line of reflection.
Solution: By definition of reflection, L is the perpendicular bisector of ̅̅̅̅̅.
( )
Slope of ̅̅̅̅̅ . Hence, slope of
i.e
Solution. You can observe that the reflecting line L and the given line are perpendicular, since the
product of their slops is 1.
Therefore is its own image.
That is the image line is
The equation of is ( )
{ ,
( )
This implies that the point of intersection . / is the mid-point of A(0, 0) and A'(x', y').
Thus, the image of the point A(0, 0) after it has been reflected along is . /.
Therefore, by slope-point form of equation of a line, the equation of the image line of is
. / . / :
= Slope of ̅̅̅̅̅ =
----------- ( i )
The midpoint of (0, 2) and (a', ) which is . / . / lies on the reflecting axis L.
{ ,
( )
It follow that C has coordinates C(1, 5)
Then the image line passes through points ( ) and C(1, 5).
Therefore, by two-point form of equation of a line, the equation of the image line is
( )
is ( ). That is ( ) → ( )
Example 59. Find the image of the point ( ) after reflection in the line .
Solution. Reflection with respect to the vertical line maps the point ( ) ( ) to the point
( ) ( ( ) ) ( ).
Reflection of a point in a horizontal line ∈
The image of a point ( ) after it has been reflecting along a horizontal line
is ( ). That is ( ) → ( )
Example 60: Find the image of the line after reflection in the line .
Solution: First choose a point on the given line to be reflected, say ( ).
( ) → ( ) ( ( ) ) ( )
{ ( ) &
( )
It follow that C has coordinates C(5, 3)
Then the image line passes through points ( ) and C(5, 3).
Therefore, by two-point form of equation of a line, the equation of the image line is
( ) ( )
( )
( )
slope of ̅̅̅̅̅ =
( )
{ &
( )
and n
the image point has coordinates ( ), which is the center of the image circle.
Therefore, the equation of the image circle is given by
Exercise 12.7
1. The vertices of triangle ABC are A (2, 1), B (3, –2) and C (5, –3). Give the coordinates of the
vertices after: a. a reflection in the x-axis b. a reflection in the y-axis
c. a reflection in the line x + y = 0 d. a reflection in the line y = x.
2. Find the image of the point (–4, 3) after a reflection about the line ℓ : y = x −2.
3. If the image of the point (–1, 2) under reflection is (1, 0), find the line of reflection.
4. Find out some of the figures which are their own images in reflection about the line y = x.
5. Find the image of the line ℓ : y = x + 4 after it has been reflected about the line L : y = x – 3.
6. Find the image of the line ℓ : y = 2x + 1 after it has been reflected about the line L : y = 3x + 2.
7. The image of the circle x2 + y2 – x + 2y = 0 when it is reflected about the line L is
x2 + y2 – 2x + y = 0. Find the equation of L.
8. Find image of the circle after reflection in the line .
9. Given an equation of a circle ( ) ( ) , find the equation of the image
circle after a reflection about the line .
10. If T is a translation that sends (0, 0) to (3, −2) and M is a reflection that maps (0, 0) to (2, 4),
find a) T(M(1, 3)) b) M(T(1, 3))
11. In a reflection, the image of the line y − 2x = 3 is the line 2y – x = 9. Find the axis of
reflection.
12. Find the image of the circle after reflection in the line = .
13. Find the image of the circle ( ) ( ) after reflection in the line : = .
15.6.3 Rotation
Rotations are movements around a central point where distance from that point is maintained.
A rotation fixes one point and everything rotates by the same amount around that point. Every rotation
has a center of rotation and an angle of rotation.
ACTIVITY 15.5 Consider a triangle ABC, discuss what will be
happened the shape, size and orientation of a triangle when you
turn ABC counter clockwise it
around a fixed point as shown in the figure below
and try to define what does rotation of figures mean?
As you observed form the figure, when ABC is rotated about a point O or the centre of rotation to be
the origin, through 900 in counter clockwise direction, then and have the same shape and
size.
DEFINITION 12.12
A ROTATION R about a point O through an angle θ is a transformation of the plane onto itself which
carries every point P of the plane into the point of the plane such that
i) and ii) (∠ ) .
O is called the CENTRE OF ROTATION and θ is called the ANGLE OF ROTATION.
The rotation is in the counter clockwise (ccw) direction if θ > 0, and in the clockwise direction (cw) if θ
< 0, as shown in the following figure.
Therefore, the image the point A(2, 0) under rotation through an angle of
about the origin is . This is written as ( ) (√ )
NOTATION: If R is rotation through an angle θ, then the image of A (x, y) is denoted by ( ).
Now, let as derive the formula for the image ( ) of a point ( )
under rotation through an angle of θ about the origin ( ).
Let be the angle between ̅̅̅̅ and the x-axis.
Thus, the angle between ̅̅̅̅̅ and the x-axis is θ.
Let . Hence, , as shown in the figure.
From trigonometric identity, you have
( ) ( ) and
( ) ( ( ) ( ))
You know that ( )
( )
This proves the following theorem.
a) 60° b) 150° c)
Therefore; the image of the point A(2, 0) when it is rotated about the origin through 60°
√ √
is ( ) . /
Therefore; the image of the point A(2, 1) when it is rotated about the origin through 60°
√ √
is ( ) . /
Example 64. Find the image of a point ( ) when it is rotated about the origin through
a) b) 900 ; cw f) 12600
Therefore; the image of the point A(0, 3) when it is rotated about the origin through
√ √
is ( ) . /
b)
2. If , then ( ) ( ).
3. If , then ( ) ( ).
4. If , for ∈ , then ( ) ( ).
B. Rotation when the center of rotation is the any point ( )
Let R be a rotation through an arbitrary point ( ). Then rotation R about the point ( , ) can be divided
the following three transformation.
i) Translation T of the center of rotation ( , ) by vector ( , ). This translation moves
( , ) to the origin (0, 0). Then the image of any point P(x, y) becomes
( ) ( ) ( )
ii) Then rotate ( ) through an angle about the origin
( ) ( ) ( s sin sin s )
= (( ) s ( ) sin ( ) sin ( ) s )
iii) Translation T of the center of rotation (0, 0) by vector ( , ). This translation moves the center of
rotation to the original position ( , ). Then the image of ( ) becomes
( ) ( ) ( )
=( ( ) s ( ) sin ( ) sin ( ) s )
This will prove the following statement.
COROLLARY 12.1: The image of a point ( ) under rotation through an angle of about the an
arbitrary point origin ( ) is given by ( ) (x y ) where
( ) s ( ) sin
( ) sin ( ) s If ( ) ( ),the rotation becomes about the origin.
sin and s
Then the images of A and B under this rotation are obtained as follow
Let ( ) is the image of ( ) ( ) under this rotation, then
( ) s ( ) sin ( ) s ( ) sin
( ) sin ( ) s ( ) sin ( ) s
( ) sin ( ) s ( ) sin ( ) s
then
( ) ( )
Example 68:Find the image of the line under a rotation R through an angle of about
Points ( ) and ( ) are on the line. Then the images of A and B under R are
√ √
( ) (( )) . s sin sin s / . /
( ) (( )) . s sin sin s / ( √ )
√ √
Then, the image line passes through points . / and ( √ ).
Therefore, by two point form of equation of line, the equation of the image line is
√ √
√ √ √
√ √
. /
√ √
√
C. Rotation of a circle about a point (a, b)
To find the image of a circle C under a given rotation R, follow the following steps:
1. Find the centre A and the radius r of the given circle.
2. Find the image of the centre of the circle under the given rotation.
Then the image circle has center and radius r.
Thus, the image circle will be the circle having center and radius r.
Example 69. Determine the image of the circle under a rotation R through an angle of
The center of the circle is O(0, 0) and the radius of the circle is r = 2
Let ( ) be the image of ( ) ( ) under this rotation. Then
( ) s ( ) sin ( ) s( ) ( ) sin( )
Example 70. Determine the image of the circle ( ) under a rotation R through an
angle of about the point ( ).
The center of the circle is O(2, 0) and the radius of the circle is r = 1.
Let ( ) be the image of the center ( ) ( ) under this rotation. Then
√
( ) s ( ) sin ( ) s ( ) sin
( ) sin ( ) s ( ) sin ( ) s √ .
√
Thus, the image of the center O(0, 0) under this rotation is ( √ ).
√
Thus, the image circle has center ( √ ) and radius r = 1.
Exercise 12.8
1. Rectangle ABCD has vertices A (1, 2), B (4, 2), C (4, –1) and D (1, –1). Find the images of
the vertices of the rectangle when the axes are rotated about (0, 0) through an angle
2. Find the point into which the given point is transformed by a rotation of the axes through the
indicated angles, about the origin. a (– ) b (–2, 0); c. (–1, 2);
3. Find an equation of the line into which the line with the given equation is transformed under
a rotation through the indicated angle about the origin.
a. 3x – 4y = 7; acute angle such that t n b. 2x + y = 3;
4. Find an equation of the circle into which the circle with the given equation is transformed
under a rotation through the indicated angle, about the origin.
a. x y ;θ b. x y x y ;θ
Find the image of (1, 0) after it has been rotated −600 about the point (3, 2).
5. If M is a reflection in the line and R is a rotation about the origin through 900, find
a. M(R(3, 0)) b. R(M(0, 2)) c. R(R(1, 1))
6. In a rotation R, the image of A(6, 2) is ( ) and the image of B(7, 3) is (2, 6).
Find the image of (0, 0).
8. Find the image of (0, 1) after it has been rotated 450 about (5, 3).
9. Find an equation of the line into which the line with the given equation is transformed under a
rotation through the indicated angle about (0, 1).
a. – ; b. ;
10. Find an equation of the circle into which the circle with the given equation is transformed
under a rotation through the indicated angle, about the (1, 0).
a. ; b. ( ) ( ) ;
a. ( )(v u) ( ) c. ( ) ( )
b. ( ) (v u)
a. ‖ ‖ b. ‖ ‖ c. ( )( )
4. Given that ‖ ‖ ‖ ‖ . Determine the value of x for which the vectors and
9. ABCD is a rectangle with ‖ ‖ and ‖ ‖ , then calculate the magnitude and direction
of: a. b.
10. Find the component forms of √ ( ) and √ ( ) in , if and makes angle of
11. Find vector equation of the line that passes through the point ( ) and parallel to vector ̇
( )
12. Find the angle between a diagonal of a cube and one of its edges.
13. Given two vectors and such that 𝑗 and | | . If and are perpendicular
vectors, then the value of | | = ________
14. If ( ) and ( ), what is the unit vector in the direction of ?
15. A line given by the vector equation ( ) ( ) ∈ , is tangent to a circle at point
(1, 4). If its center is on the y-axis, then the radius of the circle is _____________
16. Using vector method, find the equation of tangent to the circle – at the point
( ).
17. If 𝑗 and is a unit vector such that | | , then the cosine of the angle
25. Suppose that the magnitude of vectors is 24 and its direction is (from the positive x – axis).
A. √ B. √ C. √ D. √
3. What are the x and y-components of vector with magnitude 15 and direction from +ive x-
axis?
√
A. . / B. . √ / C. ( √ ) D. . √ /
√
A. √ B. √ C. D.
6. Let ABCD be a parallelogram, and and its diagonals. Which of the following is NOT
TRUE?
A. C.
B. D.
7. In the xy-plane, if ( 𝑗) ( 𝑗) ( 𝑗), then the norm of the vector is
A. 11 B. √ C. 3 D. √
8. Which of the following is the reflection of the parabola y x 2 1 in the line y x ?
A. y x 2 1 B. y x 2 1 C. x y 2 1 D. x y 2 1
9. Consider the circle given by x 2 y 2 2 and the line l given by parametric vector equation
x, y 2, 0 t 1, 1 . Which of the following is true?
A. l is tangent to the circle at 1 7 . C. l intersects with the circle at two distinct points.
,
2 2
B. l and the circle have no common point. D. The distance from the center of the circle to l is 2
image of l after a reflection in the line y 2 x 5 followed by a rotation through the angle of 90 0 about
the origin? A. x 2 y 0 B. 2x y 0 C. x 2 y 5 D. x 2 y 5
11. Which one of the following is a vector equation of the line tangent to the circle
x 2 y 2 2 x 7 0 at 1, 2 ?
What is the image of the ellipse 3x 2 y 1 1 under translation that takes 0, 1 to 2, 2
2
12.
followed by the reflection through the y axis?
A. x 22 3 y 22 1 C. x 2 3 y 2 1
2
3x 2 y 2 1 D. 3x 2 y 2 1
2 2 2
B.
13. The equation of the image of the line y x 1 after rotation about the origin through 90 0 in
counterclockwise direction is:
A. y x B. y x 1 C. y x 1 D. y x 1
x 22 y 12 4 , which of the following is the parametric vector equation of the line?
A. x, y 1, 0 t 2, 1 C. x, y 2, 1 t 1, 0
16. What is the image of the line given by x, y 1, 0 t 3, 6, t , under the translation that
What is the image of the ellipse whose equation is 2x 2 y 1 2 under translation that
2 2
17.
x2 2y2 2 B. 2 x 2 y 2 2 C. 2x 4 y 2 D. x 4 2 y 2
2 2 2 2
A.
18. The image of the point 1, 3 when reflected along the line y x is
19. If two lines y x and y x 4 are tangent to a circle at 2, 2 and 4, 0 , respectively, then
what is the equation of the circle?
A. x 22 y 2 4 C. x 3 y 1 2
2 2
B. x 42 y 22 4 D. x 1 y 1 10
2 2
20. What is the equation of a line that passes through the point 1, 2 and parallel to the vector 1, 1
?
A. 2x y 1 B. x y 1 0 C. x 2 y 3 D. y 2x 1 0
21. Which of the following points is the reflection of 1, 1 on the line y x 2 ?
A. 1, 1 B. 3, 3 C. 7, 1 D. 1, 7
22. If l is the line passing through 0, 2 and parallel to V i 3 j , which one of the following is
26. The image of a figure with vertices A1, 2, B3, 6, C 1, 2, and D 2, 2 after reflection
across the x axis is:
A. A' 1, 2, B' 3, 6, C ' 1, 2, and D' 2, 2
B. A' 1, 2, B' 3, 6, C ' 1, 2, and D' 2, 2
C. A' 1, 2, B' 3, 6, C ' 1, 2, and D' 2, 2
D. A' 1, 2, B' 3, 6, C ' 1, 2, and D' 2, 2
27. If T is a translation that takes 1, 2 to 3, 2 , then which of the following is NOT true?
y y
2 2 2 2
A. x 3 1 3 1 1 C. x 3 1 3 1 1
B. x 3 1 y 3 1 D. x 3 1 y 3 1
2 2 2 2
1 1
31. Which of the following is NOT true about transformation?
A. Rotation is a rigid motion. C. Rotations move triangles into congruent triangles.
B. Reflection is not a rigid motion. D. Translations move angles to congruent angles.
32. The image of the circle x 2 y 2 x 2 y 0 when it is reflected about the line L is
x 2 y 2 2x y 0 . The equation of L is equal to:
1
A. y x B. y 2x C. y x D. y x
2
Main Contents:
13.1. SEQUENCE
13.2. ARTHMETIC SEQUENCE AND GEOMETRIC SEQUENCE
13.3. THE SIGMA NOTATION AND PARTIAL SUMS
13.4. INFINITE SERIES
13.5. APPLICATION OF SEQUENCE AND SERIES IN DAILY LIFE
. / . /
. / . /
. / . /
. / . /
n
Therefore the first five terms of n . / are 1, - , - and
Example 2: The monthly rent of a machine, Birr 200, is to be paid at the end of each month. If it is not
paid at the end of the month, the amount due will increase Birr 3 per day. What will be the first five terms
of the amount to be paid after a delay of n days?
Solution The amount to be paid after n days delay forms the general term (in Birr) n 200 + 3n Thus
=200+3 = 203
=200+3(2) =200+6 = 206
=200+3(3) =200+9 = 209
=200+3(4) =200+12 = 212
=200+3(5) =200+15 = 215
∴ The first five terms of the amount to be paid after a delay of n days are 203, 206, 209, 2012 and 215
Fibonacci and Mulatu Sequences
Fibonacci sequence
Fibonacci sequence is defined as follows:
i n
n { i n
n n rn
Mulatu sequence
Professor Mulatu Lemma introduced a sequence of the form:
i n
n { i n
n n rn
Example: List the first eight terms of
a. Fibonacci sequence and b. Mulatu‟s sequences
Solution: the first eight terms are
. . . .
. . . .
. . . .
n n (n ) (n )
Example 1: Find the general term of the sequence n , when the first term is 5 and common difference 4.
Solution: Given n
From theorem 1.1 we have n (n )
So n (n ) n n
Example 2: What is the 31st term of the sequence 1, 4, 7, 10...?
Solution: The given sequence is arithmetic sequence with n
The nth term is n (n )
Then ( ) ( )
Further on arithmetic sequence
Example 1: When the third term is 10, and the sixth term is 1,
a. Find the general term of sequence n
b. Find
Solution: a. Applying arithmetic sequence formula and substituting existing values yields:
{ Subtracting these two equations
We obtain
( )
Therefore, the general term becomes n (n )
n (n )( )
n n
n n
b. sin n n t n
( )
Example 2: Determine whether the sequences with the following general terms are arithmetic
a. n n b. n n
Solution: To solve such types of problem, we have to show the difference between successive terms is
constant
a. n n
n (n )
n
n
That is n n n ( n ) n n
Since, the difference between successive term is constant, it is arithmetic sequence.
b. n n
n (n )
(n n )
n n n n
n n n n ( n ) n n n n
Since, the difference between successive terms is not constant, it is not arithmetic sequence
Arithmetic Mean between two numbers
The term(s) of arithmetic sequence that lie between two given terms are called arithmetic mean.
Example 1: given that x is arithmetic sequence, find x
Solution: Since it is arithmetic sequence, the difference between two consecutive terms is constant.
x x
x x
x
x
Example 2: the first and sixth terms of an arithmetic sequence are 4 and 29. Find the values of four terms
between them.
Solution: Let the four terms be
So 4, , 29 form an arithmetic sequence.
Then n (n )
Thus n (n )
( )
( )
( )
( )
√
Example: When is a geometric sequence, find ( )
Solution: As the ratio between the consecutive terms is the same,
√
Example: Find the term of the geometric sequence whose term is 5 and is
Solution:
( )
n
Sn (2a (n 1)d)
2
5
S5 (2x3 (5 1)4) 55
2
So, the sum of the first 5 terms of the sequence is 55
13.3.2. Sum of geometric Sequences
Definition: A geometric sequence is a series of numbers where each term after the first is found by
multiplying the previous term by a fixed, non-zero number called the common ratio. The sum of the first
nn terms of a geometric sequence is called the geometric series.
The formula to find the sum of the first nn terms of a geometric sequence is:
a(1 rn )
Sn
1 r
where:
Sn is the sum of the first nn terms,
a is the first term,
r is the common difference,
n is the number of terms.
Example:
Let's find the sum of the first 4 terms of the geometric sequence 2,6,18,54,…2, 6, 18, 54,….
1. Identify the first term (aa): a=2
2. Identify the common ratio (r): =6/2=3
3. Number of terms (n): n=4
a(1 rn)
Using the formula: Sn 1 r
2(1 34 ) 2(1 81)
S4 80
1 2 1 2
So, the sum of the first 4 terms of the geometric sequence 2,6,18,542, 6, 18, 54 is 80.
13.4. INFINITE SERIES
Definition: An infinite series is the sum of the terms of an infinite sequence. In other words, it's a sum in
which the terms continue indefinitely.
The general form of an infinite series is given by:
n
Where are the terms of the series.
Example
r r r
where:
If | | (the common ratio is between -1 and 1), the series converges and the sum is given by:
Solution:
Here, the first term and the common ratio
Since | | , the series converges.
The sum of the infinite geometric series is: : .
Thus, the sum of the series is 6
2. Find the sum of the infinite geometric series:
Solution:
Here, the first term and Since | | the common ratio the series diverges.
Therefore, this infinite series does not have a sum, as it grows without bound.
3. Find the sum of the infinite geometric series:
Solution:
Here, the first term and the common ratio
Since | | , the series converges.
The sum of the infinite geometric series is:
Thus sum of the infinite geometric series is: :
Thus, the sum of the series is
4. Find the 20th term of the arithmetic sequence where the first term and the common difference
d=3
Solution:
In an arithmetic sequence, the general formula for the n-th term is: n (n )
A. , so A is a geometric sequence.
Solution: The list of a numbers between 20 and 200 which are a multiple of 3 are 21, 24, 27, …198
Then the sequence is an arithmetic sequence with common difference is 3.
n
n ( (n ) ) , where n n , In this case to determine
n ( ( ) ) ( )
11. If An n 1 is an arithmetic sequence such that its 1st term A1 = -5 and its 5th term A5 =15, then its 11th term A11 is
equal to:
A. 40 B. 50 C. 45 D. 55
Solution: n (n ) ( )
Then A11= A1+(11-1)d A11
Therefore the correct answer is C
12. What is the sum of all multiples of 4 that are between 30 and 301?
A. 12,882 B. 11,288 C. 6,288 D. 6,882
Solution: The list of a numbers between 30 and 301which are a multiple of 32 are 21, 24, 27, …300
Then the sequence is an arithmetic sequence with common difference is 4.
n
n ( (n ) ) , where n n , In this case to determine
n ( ( ) ) ( ) ( )
Therefore the correct answer is B
13. The nth term of the sequence: 1, -4, 9, -16, . . . is:
A. an = (-2)n B. an = (-1)n n2 C. an = (-1)2n n2 D. an = (-1)n-1 n2
Solution:
Generally: ( )
Therefore, the correct answer is D
10
A. 0 B. 15 C. D. 5
3
n
Solution: In this sequence 2 some of the lists of terms are
5 3
n 0
n
Therefore 2 =15 the correct answer is B
5 3
n 0
Main Contents:
14.1. REVISION ON LINEAR GRAPHS
14.2. GRAPHICAL SOLUTIONS OF SYSTEMS OF LINEAR INEQUALITIES
14.3. MAXIMUM AND MINIMUM VALUES
14.4. REAL LIFE LINEAR PROGRAMMING PROBLEMS
Key terms
Summary
Review Exercise
Many real-life problems involve finding the optimum (maximum or minimum) value of a function under certain
conditions. In particular, linear programming is a field of mathematics that deals with the problem of finding the
maximum or minimum value of a given linear function, known as the objective function, subject to certain
conditions expressed as linear inequalities known as constraints. The objective function may be profit, cost,
production capacity or any other measure of effectiveness, which is to be obtained in the best possible or optimal
manner. The constraints may be imposed by different resource limitations such as market demand, labour time,
production capacity, etc
Example 1: The slope of a line passing through the points P 3, 2 and Q 1, 3 is given by
y2 y1 3 2 5
m .
x2 x1 1 3 4
NOTE:
1. Two non-vertical lines 1 and 2 with slopes m1 and m2 , respectively, are parallel if and only if they have the
same slope; i.e. m1 m2 .
2. Two lines 1 and 2 with slopes m1 and m2 , respectively, are perpendicular if and only if m1 m2 1 .
An equation of a line is an equation in two variables x and y such that a point P x, y is on if and only if
x and y satisfy the equation.
Recall that if a line has slope m and passes through a point P x1 , y1 , then the point slope form of equation of
is given by y y1 m x x1
If the line passes through 0, 0 , its equation is y mx .
If the y intercept of a line with slope m is 0, b , then its equation in the slope-intercept form is y mx b
2 3 23
Example 2: If a line passes through P 1, 3 and Q 2, 2 , then its slope m 5.
2 1 1
Its equation in slope-intercept form is
y 2 5 x 2 5x 10 or y 2 5x 8 slope 5, y intercept 0, 8 .
Exercise 14.1
1. Determine the equation of the line
a. that has slope 4 and passes through P 1, 3 . D that passes through the points P 1, 2 and Q 4, 1
b. d.
Observe that in a the left half plane x 2 contains the points on the line x 2 and hence the line is a bold
(unbroken) line; whereas in b the right half plane x 2 does not include the points on the line x 2 (broken
line).
A non-vertical line divides the plane into two regions which can be called upper and lower half planes.
Example 2: Consider the graph of the linear equation 2x y 3 and the related linear inequalities 2x y 3 and
2x y 3 . First graph the line 2x y 3 by plotting two points on the line. To identify which half plane
belongs to which inequality, test a point that does not lie on the line (usually the origin).
Test 0, 0 ;2 0 0 0 3 . Observe the broken line for 2x y 3 and solid line for 2x y 3 .
A system of linear inequalities is a collection of two or more linear inequalities to be solved simultaneously. A
graphical solution of a system of linear inequalities is the graph of all ordered pairs x, y that satisfy all the
inequalities. Such a graph is called the solution region (or feasible region).
Example 5: Draw the solution region of the system of the following linear inequalities.
2
y 3 x 1 2
y x 1
a. b. 3
y 1 x 2 x 3
3
Solution:
2 1
a. Draw the two lines 1 :y x 1 and 2 : y x 2 and identify their point of intersection
3 3
P 3, 1 . The solution region, which is the intersection of the two half planes, is shaded in the following
figure.
Definition 14.1: A point of intersection of two or more boundary lines of a solution region is called a vertex (or
a corner point) of the region.
Example 6: Solve the following system of linear inequalities.
2 x y 22
x y 13
2 x 5 y 50
x0
y 0
Solution: The last two inequalities x 0 and y 0 are known as non-negative inequalities (or non-negative
requirements). They indicate that the solution region is in the first quadrant of the plane.
Draw the lines 1 : 2 x y 22, 2 : x y 13 and 3 : 2 x 5 y 50 .
To determine the solution region test the point O 0, 0 which is not in any of these 3 lines, and find the
intersection of all half planes to get the shaded region in the following figure.
This solution region has five corner points. The vertices O 0, 0 , P 0, 10 and Q 11, 0 can be easily
determined. To find the other two vertices R and S solve simultaneously the following two pairs of equations:
1 : 2 x y 22 2 : x y 13
and
2 : x y 13 3 : 2 x 5 y 50
to get S 9, 4 to get R 5, 8
Observe that the point of intersection of 1 and 3 is not a corner point of the solution region.
Definition 14.2: A solution region of a system of linear inequalities is said to be bounded, if it can be enclosed
by a rectangle, otherwise it is unbounded. Thus the solution region of Example 6 is bounded, while that of
Example 4 is unbounded.
Solution:
i. Consider 2 x 3 y 3 . We observe that the shaded region and the origin lie on opposite side of this line
and 0, 0 , satisfies 2 x 3 y 3 . Therefore, we must have 2 x 3 y 3 as linear inequality corresponding
to the line 2 x 3 y 3 .
ii. Consider 3x 4 y 18 We observe that the shaded region and the origin lie on the same side of this line
and 0, 0 satisfies 3x 4 y 18 . Therefore, 3x 4 y 18 is the linear inequality corresponding to the
line 3x 4 y 18 .
iii. Consider 7 x 4 y 14 . It is clear from the figure that the shaded region and the origin lie on the same
side of this line and 0, 0 satisfies the inequality 7 x 4 y 14 .
iv. Consider x 6 y 3 . It may be noted that the shaded portion and origin lie on the same side of this line
and 0, 0 satisfies x 6 y 3 .
v. Also the shaded region lies in the first quadrant only. Therefore, x 0, y 0 .
Hence, in view of (i), (ii), (iii), (iv) and (v) above, the linear inequalities corresponding to the given
2 x 3 y 3
3 x 4 y 18
solution set are : 7 x 4 y 14
x 6 y 3
x, y 0
Exercise 14.2
1. Find a graphical solution for each of the following.
x y 2 x 0 x, y 0
x 3 y 6 x 3 y 6 x y 2 y 0 2 x 3 y 60
a. b. c. d. e.
4 x 3 y 3 4 x y 1 x 2 y 8 x y 8 2 x y 28
x 4 3 x 5 y 30 4 x y 48
2. Find the linear inequalities for which the shaded region in the given figure is the solution set.
a. b.
From the above figure you can observe that as the value of Z increases, the lines are moving upwards and the line
for Z 15 is outside the feasible region. The maximum possible value of Z will be obtained if we draw a line
between Z 10 and Z 15 parallel to them that just "touches" the feasible region.
49 1 9
Comparing the values of Z , you get the maximum value of Z obtained at , .
4 2 4
We also have the minimum value Z 0 at 0, 0 .
Example 2: Use the graphical method to solve the following linear programming problem.
x y 3
Maximize R 4 x 11y Subject to : 2 x y 4 .
x 0
y 0
Solution: We need to graph the system of inequalities to produce the feasible set. We will start by rewriting each
inequality as an equation, and then number the equation for each line.
The x and y intercepts for line x y 3 are 3, 0 and 0, 3 , respectively. Since the inequality x y 3
contains an equal sign, a solid line can be drawn through those two intercepts. We need to choose a test point to
substitute into the original inequality to determine which half-plane to shade. We will choose the point 0, 0 :
x y 3
00 3
03
The point 0, 0 satisfies the inequality, so we will shade the half-plane containing 0, 0 .
We can see from the diagram that the feasible set is bounded, so this problem will have an optimal solution.
Next, we need to find the vertices (corner points) of the feasible set. By observing the graph we see that the
vertices are 0, 0 , 0, 3 , 1, 2 and 2, 0 .
Finally, we substitute each corner point into the objective function to determine the optimal solution.
The x and y intercepts for line x 3 y 9 are 9, 0 and 0, 3 , respectively. Since the inequality x 3 y 9
contains an equal sign, a solid line can be drawn through those two intercepts. We need to choose a test point to
substitute into the original inequality to determine which half-plane to shade. We will choose the point
0, 0 0 9 . The point 0, 0 does not satisfies the inequality, so we will shade the half-plane does not
contains 0, 0 .
3 20
The vertices of the feasible set are 0, 5 , , and 9, 0
7 7
.Next, we substitute each point into the objective function to determine the optimal solution.
The minimum value appears to be 18, which occurs at 9, 0 . Since the feasible set is unbounded, this may or may
not represent the minimum. Let us test some points in the feasible set near the suspected optimal solution of 9, 0
For test point 8, 1 : S 2 x 7 y 2 8 7 1 23
For test point 9, 1 : S 2 x 7 y 2 9 7 1 25
For test point 10, 0 : S 2 x 7 y 2 10 7 0 20
The above is not a formal proof since we have only chosen a few points, but we could go on testing and would not
find any points in the feasible set which yield a function value less than 18. The minimum value is 18, which
occurs at 9, 0 .
Example 4: Solve the following linear programming problem. Find the maximum value of the objective function
x 2 y 4
x y 1
Z 3x 2 y , subject to the following constraints: .
x 0
y 0
Solution: From the constraints you sketch the feasible region shown in the following figure. The vertices of this
region are 0, 0 , 1, 0 , 2, 1 and 0, 2 .
3
Thus, the maximum value of Z is 34 , and occurs when x 5 and y .
2
x 2 y 8
Example 7: Maximize Z 2 x 5 y subject to: .
6 x 4 y 36
0 x 7
Solution: The feasible region is illustrated in the following figure. Since it is unbounded, we are not assured by
Theorem 14.1 that the objective function attains a maximum value. In fact, it is easily seen that since the feasible
region contains points for which both x and y are arbitrarily large and positive, the objective function can be
made arbitrarily large and positive. This problem has no optimal solution. Instead, we say the problem has an
unbounded solution.
Here vertices 2, 4 and 10, 0 give the minimum value S 20 so that the solution is not unique. In fact every
point on the line segment through 2, 4 and 10, 0 gives the same minimum value of S 20 . From this
example we can observe that
i. an optimization problem can have infinite solutions.
ii. not all optimization problems have a solution, since the above problem does not have a maximum value for Z.
Exercise 14.3
Find the maximum and minimum values of
a. Z 2x 3 y , x0 x0
x0 y0 y0
y0 subject to: x 2y 4 subject to: 2 x 3 y 6
subject to:
2 y x 16 3x y 7 3x 2 y 9
x y 10 x 2 y 7 x 5 y 20
b. Z 2x 3 y , d. Z 4 x 5 y , f. Z 3x 4 y ,
subject to: subject to: x0
x0 x0 y0
y0 y0
subject to: x 2 y 14
3 x 7 y 42 2 x 2 y 10 3x y 0
x 5 y 22 x 2y 6 x y 2
c. Z 4x 2 y , e. Z 4x 3 y ,
To solve the maximization problem geometrically, we first find the vertices by finding the points of intersection
of the border lines of S, to get O 0, 0 , A 600, 0 , B 1050, 150 and C 800, 400 .
Then a solution can be obtained from the table below:
35
Telegram channel https://t.me/johnson201485 8
Thus the maximum profit is Birr 2000 and it occurs when the monthly production consists of 800 units of product
I and 400 units of product II. (Observe that the minimum profit is Birr 0 which occurs at the vertex O 0, 0 .
Example 2: A manufacturer of tents makes a standard model and an expedition model for national distribution.
Each standard tent requires 1 labour-hour from the cutting department and 3 labour-hours from the assembly
department. Each expedition tent requires 2 labour-hours from cutting and 4 labour hours from assembly. The
maximum labour-hours available per day in the cutting department and the assembly department are 32 and 84,
respectively. If the company makes a profit of Birr 50.00 on each standard tent and Birr 80 on each expedition
tent, how many tents of each type should be manufactured each day to maximize the total daily profit? (Assume
that all tents produced can be sold.)
Solution: The information given in the problem can be summarized in the following table.
Then we assign decision variables as follows: Let x number of standard tents produced per day
y number of expedition tents produced per day
The objective of management is to decide how many of each tent should be produced each day in order to
maximize profit P 50 x 80 y . Both cutting and assembly departments have time constraints given by
1 x 2 y 32 .................. .cutting dept. constraint
3 x 4 y 84 .................. assembly dept. constraint
where x 0 and y 0................. non-negative constraints
x 2 y 32
The Linear Programming problem is then to maximize P 50 x 80 y , subject to : 3 x 4 y 84
x, y 0
To get a graphical solution, we have the feasible region S shown in the figure below. The vertices are at
0, 0 , 28, 0 , 20, 6 and 0, 16 . The maximum value of profit can be obtained from the following table.
Thus the maximum profit of Birr 1,480 is attained at (20, 6); i.e. the manufacturer should produce 20 standard and
6 expedition tents each day to maximize profit.
Example 3: A patient in a hospital is required to have at least 84 units of drug A and 120 units of drug B each
day. Each gram of substance M contains 10 units of drug A and 8 units of drug B, and each gram of substance N
contains 2 units of drug A and 4 units of drug B. Suppose both substances M and N contain an undesirable drug
C, 3 units per gram in M and 1 unit per gram in N. How many grams of each substance M and N should be mixed
to meet the minimum daily requirements and at the same time minimize the intake of drug C? How many units of
drug C will be in this mixture?
Solution: Let us summarize the above information as:
35
Telegram channel https://t.me/johnson201485 9
Let x number of grams of substance M used
y number of grams of substance N used
Our objective is to minimize drug C from 3x y . The constraints are the minimum requirements of
10x 2 y 84 ................ from drug A
and 10x 2 y 84 ................ from drug B
Since both M and N must be non- negative x 0, y 0 .
our optimization problem is to
10 x 2 y 84
Minimize C 3x y subject to: 8 x 4 y 120
x, y 0
The sketch of the feasible region S is given in the figure below.
The minimum intake of drug C is 34 units and it is attained at an intake of 4 grams of substance M and 22 grams
of substance N.
We can summarize the steps in solving real life optimization problems geometrically as follows.
Step 1: Summarize the relevant information in the problem in table form.
Step 2: Form a mathematical model of the problem by introducing decision variables and expressing the objective
function and the constraints using these variables.
Step 3: Graph the feasible region and find the corner points.
Step 4: Construct a table of the values of the objective function at each vertex.
Step 5: Determine the optimal value(s) from the table.
Step 6: Interpret the optimal solution(s) in terms of the original real life problem.
Exercise 14.4
Solve each of the following real life problems:
a. A farmer has Birr 1,700 to buy sheep and goats. Suppose the unit price of sheep is Birr 300 and the unit
price of goats is Birr 200.
i. If he decided to buy only goats, what is the maximum number of goats he can buy?
ii. If he has bought 2 sheep what is the maximum number of goats he can buy with the remaining
money?
iii. Can the farmer buy 4 sheep and 3 goats? 2 sheep and 5 goats? 3 sheep and 4 goats?
36
Telegram channel https://t.me/johnson201485 0
b. A company produces two types of tables; Tables A and Table B. It takes 2 hours of cutting time and 4
hours of assembling to produce Table A. It takes 10 hours of cutting time and 3 hours of assembling to
produce Table B. The company has at most 112 hours of cutting labour and 54 hours of assembly labour
per week. The company's profit is Birr 60 for each Table A produced and Birr 170 for each Table B
produced. How many of each type of table should the company produce in order to maximize profit?
The officers of a high school senior class are planning to rent buses and vans for a class trip. Each bus can
transport 36 students, requires 4 supervisors and costs Birr 1000 to rent. Each van can transport 6 students,
requires 1 supervisor, and costs Birr150 to rent. The officers must plan to accommodate at least 420 students.
Since only 48 parents have volunteered to serve as supervisors, the officers must plan to use at most 48
supervisors. How many vehicles of each type should the officers rent in order to minimize the transportation
costs? What is the minimum transportation cost?
c. A calculator company produces a scientific calculator and a graphing calculator. Long-term projections
indicate an expected demand of at least 100 scientific and 80 graphing calculators each day. Because of
limitations on production capacity, no more than 200 scientific and 170 graphing calculators can be made
daily. To satisfy a shipping contract, a total of at least 200 calculators much be shipped each day. If each
scientific calculator sold results in a Birr 2 loss, but each graphing calculator produces a Birr 5 profit, how
many of each type should be made daily to maximize net profits?
1. The angle of inclination of a line L is the angle measured from the x axis to L in the counter clock wise
direction.
y2 y1
2. The slope of a line passing through P x1 , y1 and Q x2 , y2 is m tan , for x1 x2 .
x2 x1
3. If a line has slope m and passes through P x1 , y1 , the slope-point form of its equation is given by
y y1 m x x1 .
4. An equation of a line can be reduced to the form ax by c, a, b, c with a 0 or b 0 .
5. A line divides the plane into two half-planes.
6. A system of linear inequalities is a collection of two or more linear inequalities to be solved simultaneously.
7. A graphical solution is the collection of all points that satisfy the system of linear inequalities.
8. A vertex (or corner point) of a solution region is a point of intersection of two or more boundary lines.
9. A solution region is said to bounded, if it can be enclosed in a rectangle.
10. A number M f c for c in I is called the maximum value of f on I, if M f x for all x in I.
11. A number m f d for d in I is called the minimum value of f on I, if m f x for all x in I.
12. A value which is either a maximum or a minimum value is called an optimal (or extreme) value.
13. An optimization problem involves maximizing or minimizing an objective function subject to constraints.
14. If an optimal value of an objective function exists, it will occur at one or more of the corner points of the
feasible region.
15. In solving real life linear programming problems, assign variables called decision variables.
36
Telegram channel https://t.me/johnson201485 1
REVIEW EXERCISES ON UNIT 14
4. Find the optimal solution of the following real life linear programming problems.
Ahadu company produces two models of radios. Model A requires 20min of work on assembly line I and 10
min of work on assembly line II. Model B requires 10 min of work on assembly line I and 15 min of work on
assembly line II. At most 22 hrs of assembly time on line I and 25 hrs of assembly time on line II are available per
week. It is anticipated that Ahadu company will realize a profit of Birr 10 on model A and Birr 14 on model B.
How many radios of each model should be produced per week in order to maximize Ahadu's profit?
a. A farming cooperative mixes two brands of cattle feed. Brand X costs Birr 25 per bag and contains 2
units of nutritional element A, 2 units of nutritional element B, and 2 units of element C. Brand Y costs
Birr 20 per bag and contains 1 unit of nutritional element A, 9 units of element B, and 3 units of element
C. The minimum requirements of nutrients A, B and C are 12, 36 and 24 units, respectively. Find the
number of bags of each brand that should be mixed to produce a mixture having a minimum cost.
5. In laying out a car park it is decided, in the hope of making the best use of the available parking space
7200 sq. ft , to have some spaces for small cars, the rest for large cars. For each small space 90sq. ft. is
allowed, for each large space 120sq. ft . Every car must occupy a space of the appropriate size. It is reliably
estimated that of the cars wishing to park at any given time, the ratio of small to large will neither less than
2:3 nor greater than 2 :1 . Find the number of spaces of each type in order to maximize the number of cars
that can be parked.
36
Telegram channel https://t.me/johnson201485 2
PRACTICE QUESTIONS ON UNIT 14
CHOOSE THE BEST ANSWER FROM THE GIVEN ALTERNATIVES
1. Which of the following point is not lie in the half-plane 2 x 3 y 12 0 ?
A. 2, 3 B. 2, 3 C. 2, 1 D. 1, 2
x 2 y 120
x y 60
2. The maximum value of the object function Z 5x 10 y , subject to the constraints x 2 y 0 is equal
x 0
y 0
to:
A. 800 B. 300 C. 600 D. 400
x 4 y 24
3 x y 21
3. Maximize Z 5x 10 y, , subject to: .
x y 9
x, y 0
a. 20 at 1, 0 B. 30 at 0, 6 C. 37 at 4, 5 D. 33 at 6, 3
2. The region represented by 2 x 3 y 5 0 and 4 x 3 y 2 0 is:
A. Not in first quadrant C. Bounded in first quadrant
B. Unbounded in first quadrant D. It is in second quadrant
3. A shopkeeper wants to purchase two articles A and B of cost price Birr 4 and 3 respectively. He thought that
he may earn 30 praise by selling article A and 10 praise by selling article B. He has not to purchase total
article worth more than Birr 24. If he purchases the number of articles of A and B, x and y respectively,
then linear constraints are:
A. x 0, y 0, 4x 3 y 24 C. x 0, y 0, 30 x 40 y 24
B. x 0, y 0, 30x 10 y 24 D. x 0, y 0, 4 x 3 y 24
4. A factory produces two products A and B. In the manufacturing of product A, the machine and the carpenter
requires 3 hour each and in manufacturing of product B, the machine and carpenter requires 5 hour and 3 hour
respectively. The machine and carpenter work at most 80 hour and 50 hour per week respectively. The profit
on A and B is Birr 6 and 8 respectively. If profit is maximum by manufacturing x and y units of A and B
type product respectively, then find the constraints for the function 6 x 8 y
A. x 0, y 0, 5x 3 y 80, 3x 2 y 50 C. x 0, y 0, 5x 3 y 80, 3x 2 y 50
B. x 0, y 0, 3x 5 y 80, 2 x 3 y 50 D. x 0, y 0, 3x 5 y 80, 3x 3 y 50
5. Which one of the following is the vertex of a feasible region by the linear constraints
3x 4 y 18, 2x 3 y 3 and x, y 0 ?
A. 0, 3 B. 0, 2 C. 4.8, 0 D. 0, 5
6. A toy manufacturer produces two types of dolls; a basic version doll A and a deluxe version doll B. Each doll
of type B takes twice as long to produce as one doll of type A. The company have time to make a maximum
of 2000 dolls of type A per day, the supply of plastic is sufficient to produce 1500 dolls per day and each type
requires equal amount of it. The deluxe version, i.e. type B requires a fancy dress of which there are only 600
per day available. If the company makes a profit of Birr 3 and Birr 5 per doll, respectively, on doll A and B,
then the number of each should be produced per day in order to maximize profit, is equal to:
A. 800, 500 B. 500, 600 C. 450, 450 D. 1000, 500
36
Telegram channel https://t.me/johnson201485 3
7. The corner point of the feasible region determined by the system of linear constraints are
0, 0 , 0, 40 , 20, 40 , 60, 20 , 60, 0 . The objective function is Z 4x 3 y .
Compare the quantity in column A and column B
Column A Column B
Maximum of Z 325
A. The quantity in column A is greater
B. The two quantities are equal
C. The quantity in column B is greater
D. The relationship cannot be determined based on the given information.
8. A feasible solution to linear programming:
A. Must be a corner point of feasible region.
B. Must optimize the value of the objective function.
C. Need not satisfy all of the constraints, only some of them.
D. Must satisfy all of the problem‟s constraints simultaneously.
9. Which of the following represent the shaded region?
36
Telegram channel https://t.me/johnson201485 4
14. In Mathematics test there are multiple choice and workout part questions to be answered. The relevant data is
given in the following table:
Type of questions Time taken to solve Marks Number of questions
Multiple choice 5 minute 3 10
Workout 10 minute 5 14
The total mark is 100. Students can solve all questions. To secure maximum marks, a student solves x multiple
choice and y workout questions in three hours, then linear constraints except x, y 0 are:
A. 5x 10 y 180, x 10, y 14 C. x 10 y 180, x 10, y 14
B. 5x 10 y 180, x 10, y 14 D. 5x 10 y 180, x 10, y 14
15. What is the maximum value of the function Z 3x y subject to: 2 x 3 y 1, x y 3 , for x 0 and
y0?
9 3
A. 7 B. 9 C. D.
2 2
3x y 6
x y 1
16. Consider the following linear programming model: objective function Z 3x 5 y , subject to .
x 2y 6
x 0, y 0
Which of the following is the minimum value of Z ?
A. 5 C. It has no minimum value
B. 3 D. 0
17. Consider a line with equation y mx b . Which of the following is true?
A. The line intersects the x axis, if m 0 and b 0 . C. The line passes through the origin.
B. The line intersects the y axis, if m 0 and b 0 . D. The line has no slope.
18. A problem to be solved using linear programming method is given as follows:
“ A coffee packer blends coffee from Harar and from Sidama to prepare two kinds products, viz., “Super”
and “Deluxe” brands. Each kilogram of Super coffee contains 0.5kg of Harar coffee and 0.5kg of Sidama
coffee, whereas each kilogram of Deluxe coffee contains 0.25kg of Harar coffee and 0.75kg of Sidama
coffee. The packer has 120kg of Harar coffee and 160kg of Sidama coffee in store. Moreover, the plan is to
get 20 Birr profit on each kilogram of Super coffee and a profit of 30 Birr on each kilogram of Deluxe coffee.
In order to maximize profit the packer needs to know the amount (in kg) of each brand of coffee that should
be blended.”
If x the amount of Super coffee in kilograms and
y the amount of Deluxe coffee in kilograms, then which one of the following standard forms of linear
programming models should be used to solve the above problem?
A. Maximize P 30x 20 y C. Maximize P 20x 30 y
0.5 x 0.75 y 120 0.75 x 0.25 y 120
subject to: 0.5 x 0.25 y 160 subject to: 0.5 x 0.5 y 160
x 0, y 0 x 0, y 0
B. Maximize P 20x 30 y D. Maximize P 30x 20 y
0.5 x 0.25 y 120 0.5 x 0.5 y 120
subject to: 0.5 x 0.75 y 160 subject to: 0.25 x 0.75 y 160
x 0, y 0 x 0, y 0
Main Contents:
15.1. BASIC MATHEMATICAL CONCEPTS IN BUSINESS
15.2. COMPOUND INTEREST AND DEPRECIATION
15.3. SAVING, INVESTING, AND BORROWING MONEY
15.4. TAXATION
Key terms
Summary
Review Exercises
A. Ratio
Consider the explanation given below :
Suppose the number of students and teachers in a given school are 3900 and 75, respectively. From this we can
make the statement that
i) “the ratio of teachers to students in the school is 1 to 52” or
ii) we can say that “the ratio of students to teachers in the school is 52 to 1”.
This tells us that for every 52 students in the school there corresponds one teacher.
DEFINITION 15.1
A RATIO is a comparison of two quantities that are measured in the same unit.
The numbers appearing in a ratio are called TERMS of the ratio and they must be expressed in the same unit of
measurement.
For instance, if the ratio of boys to children is 2 to 5, then this is to mean that there are two boys for every 5
children. So, if there are 50 children in attendance, then there are 20 boys.
Thus, the ratio of boys to children is written as 2 : 5. This ratio is part-to-whole ratio.
The terms of this ratio are 2 and 5.
Ratios can be written in three ways:
A ratio can be expressed in one of two ways:
i. part-to-whole ratio or ii. part-to-part ratio
Example 1. Suppose there are 20 students in a class, of which 12 are girls, as shown in the figure below.
Determine
a. Ratio of number of girls to number of boys and ratio of boys to girls.
b. Ratio of number of girls to number of whole students in the class
Example 2. The following table gives the number of teachers in a given school according to their education
level and sex.
a. What is the ratio of female diploma holders to the number of teachers in the school?
b. What is the ratio of male degree holder to all degree holder teachers in the school?
c. What is the ratio of diploma holders to degree holders in the school?
Solution:
a. The first question is asking the part-to-whole ratio, hence it is 16:100 or 4:25.
b. The second question is asking the part-to-whole ratio of male teachers, hence it is 46:72 or 23:36.
c. The third question is asking the part-to-part ratio, hence it is 42:58 or 21:29.
Example 3. What is the ratio of 1.6 meters to 180 centimetres?
Solution: To compare two measurements in different units you must change one of the units of measurement to
the other unit.
If you change 1.6 meters to centimeters, we have conversion rate 1 m = 100 cm. hence,
1.6 meters = 1.6 100 cm = 160 cm.
REMARK. People commonly form a group and involve on a given business activity according to their
individual contribution for the business. In this case, their individual profit is allocated according to the ratio of their
investment.
To allocate, multiply each term of the ratio by the value of the single part, i.e.
3 × 1,250 = Birr 3,750; 4 × 1,250 = Birr 5,000;
8 × 1,250 = Birr 10,000 and 5 × 1,250 = Birr 6,250.
Therefore, the allocation will be Birr 3,750, Birr 5,000, Birr 10,000 and Birr 6,250, respectively.
Example 5. A sum of money was divided between Aster, Ali, and Mesfin in the ratio ,
respectively. Aster has received Birr 3504. How much money was there to start with?
Solution: First write the terms of the ratio as the common denominator. Thus the common denominator for the
ratio is L.C.M(5, 3, 1) = 15.
TOTAL AMOUNT =
Exercise 15.1
1. A profit of Birr 19,560 is to be divided between four partners in the ratio of 3 : 2 : 1 : 6. How
much should each receive?
2. Allocate Birr 800 among three workers in the ratio of
B. Rate
Frequently we want to compare two different types of measurements, such as miles to gallons and and money per
hours. To make this comparison, we use a rate. For instance, to make a beam or a column of residential building,
cement, sand and gravel are mixed in the ratio , respectively. In this case cement is measured in quintals while
sand and gravel are measured using a cubic meter box. Hence the ratio involves different units of
measurement and this will lead us to the following definitions.
DEFINITION 11.2: A rate is a comparison of two or more quantities expressed in different units of
measurement.
Like ratios we usually write rates as fractions. We put the first given in the numerator and the second amount in the
denominator. When rates are simplified, the units remain in the numerator and denominator.
Example 6. The distance from Addis Ababa to Adama is 100 km. Ahmed travelled by minibus from Addis
Ababa to Adama early in the morning and it took him 1 hour and 20 minutes. What is the rate of speed of his
journey?
Solution. : The rate of speed of his journey is the ratio of the distance travelled and the time it took. Since the
distance is 100 km and the time taken is , the rate is:
Example 7. A wild dog can run 30 miles in 1 hour. How many feet is this per second?
Solution. You need to convert to
In dealing with business, production, population, and so on, it is common to describe by what amount a quantity has
increased or decreased based on some starting or fixed level. This will lead us to the rate of change of a given
quantity given by the relation:
The rate of change will be a rate of increase if the amount of change is positive and a rate of decrease if the amount
of change is negative.
Example 8. The price of a quintal of cement in Addis Ababa in September 2008 was Birr 220, and ten months
later, on July 2009, its price was Birr 370. What is the rate of increase in the price of one quintal of cement from
September 2008 to July 2009?
Solution. We are given that: the original price = Birr 220 and the new price = Birr 370.
Hence change in price = Birr 370 – Birr 220 = Birr 150
0.682
Example 9. Aster has invested 20,000 Birr in a fruit wholesaler. A year later the audit report on the business
indicated that there was 16,200 Birr as a balance. Find the rate of decrease that resulted in one year.
Solution. Since the balance indicated that there is a decrease from the amount of capital invested, we have a
( )
decrease rate. 0.19
The negative sign indicates that there is a decrease in the investment which is a loss.
Exercise 15.2
1. A carpenter‟s daily production of school chairs increased from 20 units to 40 units. At the same time his income
(or revenue) increased from 1600 Birr to 2400 Birr. What is the rate of change of income per unit?
2. A steel company has imported 35 tons of raw material from South Africa in 1995. In 2008 the company
imported 54 tons of raw material from the same country. What is the rate of change of amount imported?
C. Proportion
DEFINITION 15.3
In the proportion a : b = c : d , the product of the extremes is equal to the product of the means; that is, is
equivalently represented as a × d = b × c .
For three quantities a, b and c such that which is equivalent to b2 = a × c , b is called the MEAN
PROPORTIONAL between a and c .
Proportions and rates allow us to solve many applications.
Example 10. You are making cookies. A recipe calls for 29 grams of sugar and makes 2 dozen cookies. You
want to make 6 dozen cookies for a week. How much grams of sugar you will need?
Solution. We can use a proportion to figure out how much grams of sugar you will need.
Our first step is to write the rate 29 grams of sugar for 2 dozen cookies in fraction form.
You want to make 6 dozen cookies but do not know how much sugar we need. We will use this information to write
a proportion.
Notice the units are lined up. Both rates have grams in the numerator and dozens in the denominator. If we think of
these rates as equivalent fractions, we have
Example 11. A secretarial pool (15 secretaries in all) on one floor of a corporate complex has access to 11
telephones. If on a different floor, there are 23 secretaries, approximately what number of telephones should be
available?
Solution Let x be the number of telephones available on the other floor. Then we
have the proportion 15 : 11 = 23 : x, that is,
Hence,
Example 12. If z is proportional to y and to the square of x and z = 80 when x = 2 and y = 5, then find the
equation that relates the variables x, y and z.
Solution. We are given that which is equivalent to , where k is a constant of proportionality.
To determine the constant of proportionality, put the given values of the variables in .
z = 80 when x = 2 and y = 5 implies that ( )( ) this gives
Therefore the equation that relates the three variables is .
Example 13. The power (P) of an electric current varies jointly as the resistance (R) and the square of the
current (I). Given that the power is 12 watts when the current is 0.5 amperes and the resistance is 40 ohms, find the
power if the current is 2 amperes and the resistance is 20 ohms
Solution. , that is, , where k is a constant of proportionality. Putting the given values in the
equation and solving for k, we have ( )( )
.
( )( )
Hence the relationship between the three variables is and the required power is
( )( ) watts.
Exercise 15.3
1. The ratio of the lengths of corresponding sides of two similar decagons is 1: 2. If the perimeter of the
smaller decagon is 76 cm, what is the perimeter of the larger decagon?
2. A cookie recipe requires 4 cups of flour to make 5 dozen cookies. If Amy needs to make 15 dozen cookies,
how many cups of flour will she need?
3. The president of the student body estimated that 2 out of every 3 students at school would attend the Spring
Festival. If there are 1,140 students at this school, according to the estimate, how many students will not
attend the Spring Festival?
C. Percentage
The concept of percentage is an extension of the material we have covered about fractions and ratio. To allow easy
comparisons of fractions we need to use the same denominator. As such, percentages use 100 as the denominator. In
other words, the „whole‟ is divided into 100 equal parts. The word “per cent” means per 100. Therefore 27% is .
DEFINITION 15.5 A PERCENTAGE is the numerator of a fraction whose denominator is 100. The term
percent is denoted by % which means “per one hundred”.
Commercial Discount
In business activities, it is common to offer a sales discount due to clearance of available stock, changing the
business activity, approaching expiry date, and so on. In such cases the discount of an item is described in terms of
percentage. For example, you may have 20% discount, 30% discount, and so on.
If p is the original price of an item and r is the percentage of discount, then the amount of discount is given by:
Markup: In order to make a profit, any institution or company must sell its products for more than the
product costs the company to make or buy. The difference between a product selling price and its cost is called
MARKUP.
Markup = Selling price – Cost price
Example 18. If the price of cement is Birr 250 per quintal and you sell it for Birr 330 per quintal, find the
markup per quintal.
Solution Markup = Selling price – Cost price
= Birr 330 per quintal – Birr 250 per quintal = Birr 80 per quintal
REMARK. Markup is usually expressed in terms of percentage with respect to selling price
and cost price.
i) Markup with respect to selling price is given by
Ordinary annuity
Many people are not in a position to deposit a large amount of money at a time in an account. Most people save
money by depositing relatively small amount at different times. If a depositor makes equal deposits at regular
intervals, he/she is contributing to an ANNUITY. The deposits may be made weekly, monthly, yearly, or any other
period of time.
If we deal with annuities in which the deposits (or payment) are made at the end of each of the deposit (or payment)
intervals, which coincides with the compounding period of interest, then this type of annuity is called ORDINARY
ANNUITY.
Suppose a person deposit an amount R at the end of each of an „m‟ deposit (or payment) interval periods, in an
account that pays interest per year compounded m-times a year over t years. Then he made a total of
deposits, one at the end of each interest payment period over t years.
The total future value will be calculated as follow:
The last deposit (the deposit) has no interest, so stays at Birr R. Thus,
The deposit has interest calculated for only one period, and it will accumulate to
( ) ( )
where R is the periodic payment and is the interest rate per period.
The payment has interest computed for two periods, and it will accumulate for the first period A = R (1 + i)
and for the second period as the amount for the first period serve as a principal to the second period
( ) ( ) .
The payment has interest computed for three periods, and it will accumulate for the first period A = R (1 + i)
and for the second period as the amount for the first period serve as a principal to the second period
( ) ( ) .
Continuing this process the first payment has interest computed for periods, and will accumulate to the
amount ( )
Thus, the amount of the ordinary annuity will be the sum S of the amount accumulated from each deposit made, that
is,
( ) ( ) ( ) ( ) ……………….. (*)
Multiplying both sides of equation (*) by ( ), we obtain
( ) ( ) ( ) ( ) ( ) ( ) …….. (**)
Subtract (*) from (**) and simplifying the difference give us ( ) ( )
,( ) -
,( ) -
This implies that
,( ) -
Thus, the future value S of an ordinary annuity is given by
where R is the periodic payment, is the interest rate per period and n is the number of periods.
The amount of interest of an ordinary annuity is .
In the above ordinary annuity formula, and , where which is the interest rate per year, m is the
number of times interest is compounded per year and t is time in years.
Example 29. A machine costing Birr 35,000 is estimated to have a useful lifetime of 8 years and a salvage value of
Birr 3,000. What is the accumulated depreciation at the end of 5 years? Find the book value of the asset at that time,
using the fixed instalment method (where book value = cost – accumulated depreciation)
Solution. We have the cost = Birr 35,000, salvage value = Birr 3,000 and useful life = 8 years.
The depreciation charge per year is r i ti n Birr 4,000
n
Hence the accumulated depreciation increases by Birr 4,000 every year.
The accumulated depreciation at the end of 5 years will be :
years × depreciation charge per year = 5 × 4,000 = Birr 20,000.
The book value of the asset at the end of 5 years will be :
Book value = cost – accumulated depreciation = 35,000 – 20,000 = Birr 15,000.
Example 30. A business buys a machine for Birr 250,000. It is expected to have a useful lifetime of 5 years, after
which time it will have a scrap value of Birr 50,000. Calculate the annual depreciation charge.
Solution. In this question, we have the cost = Birr 250,000, scrap value = Birr 50,000 and useful life = 5 years.
The depreciation charge per annum is r i ti n n
Birr 40,000
Hence the accumulated depreciation increases by Birr 40,000 every year.
The yearly depreciation and book value are shown in the following table.
Cost Yearly Accumulated Book
Year
depreciation depreciation value
Year 1 250,000 40,000 40,000 210,000
Year 2 250,000 40,000 80,000 170,000
Year 3 250,000 40,000 120,000 130,000
Year 4 250,000 40,000 160,000 90,000
Year 5 250,000 40,000 200,000 50,000
Therefore the machine will have a scrap value of Birr 50,000 after 5 years time.
Reducing balance method: The reducing balance method (or declining-balance method) yields a
declining periodic depreciation charge over the estimated life of the asset. Of the several variant techniques the most
common is to apply double straight-line depreciation rate, computed by:
Example 31. Using the double reducing balance method of depreciation, determine the book value at the end of
the second year of an item that was bought on May 5 for Birr 30,000 and that has a salvage value of Birr 5,000 and
an estimated useful life of 40 years.
Solution. The depreciation rate per year = = 0.05
The depreciation for the first full year is 30,000 × 0.05 = Birr 1,500.
Hence the depreciation per month is Birr 1,500 per year 12 month per year = Birr 125 per month.
Since the item is bought on May 5, it is close to May 1. Hence at the end of the first year the depreciation is
Birr 125 per month × 8 months = Birr 1,000.
The book value at the end of the first year is 30,000 – 1000 = Birr 29,000.
Therefore, the depreciation for the second year is 29,000 × 0.05 = Birr 1450, and
the book value at the end of the second year is Birr 29,000 – Birr 1,450 = Birr 27,550.
Exercise 15.7
1. A dump truck is purchased for $110,000 and has an estimated salvage value of $10,000 at the end of the
recovery period. Prepare a depreciation schedule for the dump truck using the fixed instalment method with a
recovery period of five years.
2. New equipment was obtained at a cost of Birr 100,000 on January 5. The equipment has estimated lifetime of 5
years and an estimated residual value of Birr 8,000.
i. Determine the annual depreciation for each of the five years of the estimated useful life of the equipment.
ii.The accumulated depreciation at the end of each year.
iii. The book value of the equipment at the end of each year by using
a. the fixed instalment method. b. the double reducing balance method.
3. A dump truck is purchased for $110,000 and has an estimated salvage value of $10,000 at the end of the
recovery period. Prepare a depreciation schedule for the dump truck using the double reducing balance method with
a recovery period of five years.
4. A business buys a machine for Birr 200,000. It is expected to have a useful life of 10 years. After which time it
will have a scrap value of Birr 0. Calculate the annual depreciation charge.
15.3. SAVING, INVESTING AND BORROWING MONEY
What is Money?
It is very difficult to give a precise definition of money because various authors have defined money differently.
However we may define money in terms of functions it performs, i.e. “Money is that what money does" or
"Anything which is generally accepted as a medium of exchange in the settlement of all transactions
including debt and acts as a measure and store of value".
Functions of money: Money performs the following four important functions
I. Primary Functions of Money
a. Money as a medium of exchange: the most important function of money is to serve as a
medium of exchange.
Turnover Tax (TOT): To enhance fairness in commercial dealings and to make a complete coverage of
the tax system, a turnover tax is imposed on those persons who are not required to register for VAT, but supply
goods and services in the country. As a result, persons who are engaged in the supply of goods and rendering of
service (which are taxable) and who are not required to register for VAT have to pay turnover tax on the value of
goods they supply or on the value of services they render. TOT is computed as per the proclamation No 308/2002.
The TOT rate is
On goods sold locally : 2%
On services rendered locally :
Contractors, grain mills, tractors, and combine harvesters : 2%
Others : 10%
Items like diplomatic and consular missions, personal effects, grants and gifts to Ethiopia, fire fighting
instruments and appliances, trade samples, defense and public security equipment, materials for handicapped
and similar items are exempted from customs duty.
Example 39. Kent tobacco importing company paid cost of purchase $ 120,000.00, insurance premium and
freight costs are, respectively, $12,000.00 and $8,000.00.The exchange rate is currently $1=12.50 Birr. Compute the
customs duty. (Note:- Tobacco is taxed at 35%.)
Solution. CIF = (120,000 + 12,000 + 8,000) × Birr 12.50 = 140,000 × Birr 12.50 = Birr 1,750,000.
Custom Duty = Birr 1,750,000 × 35% = Birr 612,500.
Exercise 15.10
1. Find the income tax of the following employees of Nyala Insurance sh. Company.
a. W/ro Mebrat with monthly salary of Birr 850.
b. Ato Tesfu with monthly salary of Birr 2,390.
c. Dr. Gebru with monthly salary of Birr 5,400.
2. Buna Bank declared to pay 20% dividend to its share holders. Find the dividend earned and
tax to be paid by the following share holders.
a. Mesfin with Birr 300,000 worth of shares.
b. Askale with Birr 100,000 worth of shares.
c. W/ro Almaz with Birr 450,000 worth of shares.
KEY TERMS
annually jointly proportional rate
base liquidity ratio
book value markup reducing-balance method
commercial discount mean proportion restrictions
compound interest ordinary annuity safety
compound proportion percentage salvage value/residual value
depreciation present value semi-annually
earnings principal simple interest
fixed-installment method proportion simple proportion
future value proportionality constant taxes
interest quarterly terms
SUMMARY
1. A ratio is a comparison of two or more quantities expressed in the same unit of measurement.
2. A rate is a comparison of two or more quantities expressed in different unit of measurement.
3. A ratio can be a rate.
n n n n n n
4.
n n n n
Main Contents:
If you are driving a car, your speed at any moment is the rate at which your position is changing with
respect to time. The speed is the derivative of the position function.
When you look at the graph of a function ( ) you can imagine a tangent line touching the graph at a particular
point. The derivative at that point is the slope of that tangent line. The slope tells you how steep the curve is at that
point.
If the tangent line has a positive slope, the function is increasing at that point.
If the tangent line has a negative slope, the function is decreasing at that point.
If the tangent line is horizontal (slope = 0), the function has a local maximum, minimum, or a point of
inflection at that point.
The second derivative indicates whether the function is concave up (like a bowl) or concave down
(like a dome).
The third derivative and higher-order derivatives are used in advanced analysis, but typically, the
first and second derivatives are the most important.
16.2. APPLICATIONS OF DERIVATIVES
16.2.1. Finding Tangent Lines:
Derivatives are used to find the equation of a tangent line to a curve at a given point. The slope of the
tangent line at is the value of the derivative ( )
Example: For ( ) , the slope at is ( ) The equation of the tangent line is:
( ) ( )( )
Substituting the values: ( )
16.2.2. Optimization(Finding Extrema (Maximum and Minimum): Derivatives are used in
optimization problems to find maximum or minimum values of a function. By setting the derivative equal
to zero ( ( ) ) we can find the critical points, which are potential maxima, minima, or points of
inflection.
16.2.4. Engineering and Science: Derivatives are also used to describe physical phenomena, such as heat
transfer, fluid dynamics, and electrical circuits. They allow engineers and scientists to model and analyze
rates of change in systems.
Conclusion: The derivative is an essential concept in calculus that gives us a way to understand how
functions behave, how things change, and how we can optimize and control processes in various fields.
Whether you're studying the motion of a car, the growth of a population, or the design of a bridge,
derivatives provide the tools to analyze change effectively.
16.3. INTRODUCTION TO INTEGRATION
Integration is a fundamental concept in calculus that involves finding the total accumulation of a quantity.
It is often interpreted as the area under a curve or the accumulation of quantities over an interval.
Mathematically, the integral of a function ( ) over an interval , -is denoted as:
∫ ( )
∫ ( ) ( ) ( )
This theorem allows us to evaluate definite integrals by finding the antiderivative of the integrand and
computing its values at the boundaries of the interval.
Application Example: Area Under a Curve
Consider the function ( ) over the interval [1, 3]. To find the area under the curve from to
, we compute the definite integral:
∫ [ ] ( ) ( )
Therefore, the area under the curve ( ) over the interval from to is square units
v r v u ∫ [ ] [ ] (. / . /)
Supplementary question
DIRECTION: ALL OF THESE QUESTIONS ARE FROM THE TWO GRADE LEVELS AND
MULTIPLE CHOICES; SO, SOLVE THE QUESTIONS USING DEFINITIONS AND RULES ON
YOUR TEXT BOOK, CHOOSE THE CORRECT ANSWER FROM THE GIVEN
ALTERNATIVES AND PREPARE YOUSELF FOR SSLE
UNIT ONE: FURTHER ON RELATIONS AND FUNCTIONS
1. Which of the following function is power function?
A. f(x) = xx B. f(x) = 2x C. f(x) = 4x-3/4 D. f(x) = (6x +1)6
1
2. Let f(x) = , the domain of f(x) = ______. A. * + B. C. D.
| x | x
3. Which of the following is a one-to-one correspondence function?
A. f:[0, ) [0, )defined by f(x) = |x - 1| C. f: defined by f(x) = ex
B. f:[1, ) [0, )defined by f(x) = (x – 1)2 D. f:[1, ) [0, )defined by f(x) = (x – 1)2+1
4. Which of the following is rational expression?
√
A. B. C. D. 6x 2 5 x
x
5. Let f(x) = , g(x) = √ and assume that b = a + 1, then the value of f(a-1) + g(a+1)
x2
√ √ √ √
A. B. C. D.
x | x |
6. If x<0, then the simplest form of f(x) = is equal to:
x
A. 2x B. 2 C. -2 D. 0
x2 1
7. If f(x) = and g(x) = 2 , then f(g(x)) is equal to:
x2 x
x
A. x 2 B. x 2 C. x D.
x
x
8. If f(x) = ln 2 , for x>1, then which one of the following is the inverse off?
x 1
e 2
x
ex 2 ex x
A. G(x) = x B. G(x) = x C. G(x) = x 2 D. G(x) = x 1
e 3 e 1 e 1 e 2
9. What is the value of |x| + 2x if x<0?
A. -3x B. 3x C. –x D. x
1
10. If f(x) = x , then which of the following is equal to f-1(x)?
e 1
1 1
A. n(1 x) n( x) B. e-x+1 C. n D. x
x 1 e 1
11. Given f(x) = In(x-1) and g(x) = 1 2 x , which one of the following is the domain of f g ?
1 1
A. {x R : x 1} B. {x R : x } C. {x R : x 0} D. {x R : x }
2 2
A. ( ) B. ( ) C. ( ) D. ( )
A. *( )+ B. *( )+ C. *( )+ D. *( )+
76. Let ( ) and ( ) , then the values of and respectively such that
is a diagonal matrix.
A. B. C. D.
A. 3 B. -3 C. 0 D. 5
1 y
2 0 x
78. Let A = and B = 0 3 Such that A + 2Bt = 0
2y x y 4
1 x 2
Then which of the following is the value of y?
13
A. 0 B. C. -8 D. any real number
2
2 0 0
A = 1 5 0 and |B| =
1
79. Let A and B be 3 x 3 matrices such that Which one of the
1 10
0 1
2
following is equal to |2ABT|?
A. 1 B. 4 C. 100 D. 400
x y xz 1
80. What is the solution set of the following system of equations x 2y x 2
2 x 2 y 4 z 2
A. {(0,1,0)} C. {(-3k,k+1,k|k ( , )}
B. , ) D. {(3k,k-1,k|k ( , )}
1 2
81. Suppose A = . If X is a 2x2 matrix such that AX – AT = 2A , then what is the value ofX?
2 3
3 1 3 3 3 6 3 0
A. B. C. D.
1 3 3 3 6 9 0 3
82. Suppose that A and B are 3x3 matrices, I is the identity matrix of order 3 such that AB = 2I. If det
B = |B| = 6. What is det (AT)?
1 4
A. B. C. 12 D. 48
3 3
x 3x 2 z 6
95. The solution of the system of linear equation of 2 x 4 y 3z 8 is:
3x 6 y 8 z 5
A. X = -1, y = -3, z = -2 C. x = 1, y = -3, z = 2
B. X = -1, y = -3, z = 2 D. x = 1, y = 3, z = -2
96. If is perpendicular to the n, what is the cosine of the angle between and ?
| | | | | | | |
A. B. C. D.
| | | | | | | |
97. Suppose = (3, x) and = (x, y-2) are vectors, then what is the value of „x‟ so that
3
A. 30 B. 24 C. 20 D. 25
138.The value of x and y that gives the minimum value of the objective function z = 4x + 2y subject to
given constraints
A. x = 1, y = 4 B. x = 2, y = 1 C. x = 4, y = 0 D. x = 0, y = 4
139. The possible values of the point which minimizes the value of the objective function
Z= -2x+y Subject to
are:
A. (0, 0) B. (7, 4) C. (7,0) D. (5,2)